Maryville 611 Patho Exam 4

¡Supera tus tareas y exámenes ahora con Quizwiz!

What part of the kidney controls renal blood flow, glomerular filtration, and renin secretion? a. Macula densa b. Visceral epithelium c. Juxtaglomerular apparatus (JGA) d. Filtration slits

ANS: C Control of renal blood flow, glomerular filtration, and renin secretion occur at the JGA. Together, the juxtaglomerular cells and macula densa cells form the JGA. The control of renal blood flow, glomerular filtration, and renin secretion is not directed by any of the other options. Page 1322

Creatinine is primarily excreted by glomerular filtration after being constantly released from what type of tissue? a. Nervous system b. Kidneys c. Muscle d. Liver

ANS: C Creatinine is constantly released from only muscle tissue to be excreted by glomerular filtration. Page 1366

During the latent period of a herpes virus infection, where in the host cell is the genome of the virus maintained? a. Mitochondria b. Lysosomes c. Nucleus d. Cytoplasm

ANS: C During the latent period, the genome for the virus is maintained in the host cell nucleus without causing the death of the cell. Page 933

Which gastric hormone inhibits acid and pepsinogen secretion, as well as decrease the release of gastrin? a. Bombesin b. Histamine c. Somatostatin d. Acetylcholine

ANS: C Of the options available, only somatostatin inhibits the secretion of acid and pepsinogen and decreases gastrin release. Page 1397 | Table 40-1

Priapism has been associated with the abuse of what substance? a. Marijuana b. Alcohol c. Cocaine d. Heroin

ANS: C Of the options available, priapism has been associated with cocaine use. Page 890

Open-angle glaucoma occurs because of: a. Decreased production of aqueous humor b. Increased production of vitreous humor c. Obstructed outflow of aqueous humor d. Excessive destruction of vitreous humor

ANS: C Open-angle glaucoma occurs because of an obstruction of the outflow of aqueous humor at trabecular meshwork or Schlemm canal. The remaining options fail to accurately describe the cause of open-angle glaucoma. Page 511 | Table 16-5

Atheromatous plaques are most commonly found: a. In larger veins b. Near capillary sphincters c. At branches of arteries d. On the venous sinuses

ANS: C Over 20 to 30 years, atheromatous plaques (stenotic lesions) tend to form at branchings and curves in the cerebral circulation, not at any of the other options provided. Page 598

Which process makes it possible for ureters to be transplanted successfully? a. Compensatory hypertrophy b. Erythropoietin secretion c. Peristalsis d. Collateral circulation

ANS: C Peristalsis is the process which makes it possible for ureters to be transplanted successfully. This process is maintained even when the ureter is denervated. The remaining options are not relevant to transplant success. Page 1325

Which hormone relaxes the myometrium and prevents lactation until the fetus is born? a. Gonadotropin-releasing hormone (GnRH) b. Follicle-stimulating hormone (FSH) c. Progesterone d. Estrogen

ANS: C Progesterone is sometimes called the hormone of pregnancy. Its effects in pregnancy include (1) maintenance of the thickened endometrium; (2) relaxation of smooth muscle in the myometrium, which prevents premature contractions and helps the uterus expand; (3) thickening of the myometrium, which prepares it for the muscular work of labor; (4) prevention of lactation until the fetus is born; and (5) prevention of additional maturation of ova by way of suppressing FSH and LH, thereby stopping the menstrual cycle. Page 778

Where in the male body does spermatogenesis occur? a. Epididymis b. Rete testes c. Seminiferous tubules d. Vas deferens

ANS: C Spermatogenesis takes place only in the seminiferous tubules of the testes (see Figure 23-14). Pages 784-785

The collateral blood flow to the brain is provided by the: a. Carotid arteries b. Basal artery c. Circle of Willis d. Vertebral arteries

ANS: C The arterial circle (circle of Willis) (see Figure 15-20) is a structure credited with the ability to compensate for reduced blood flow from any one of the major contributors (collateral blood flow). The remaining options are not considered sources of collateral blood flow. Page 467

The brain receives approximately what percentage of the cardiac output? a. 80% b. 40% c. 20% d. 10%

ANS: C The brain receives approximately 20% of the cardiac output, or 800 to 1000 ml of blood flow per minute. Page 454

Tremors at rest, rigidity, akinesia, and postural abnormalities are a result of the atrophy of neurons in the brain's: a. Caudate that produces serotonin b. Putamen that produces gamma-aminobutyric acid c. Substantia nigra that produces dopamine d. Hypothalamus that produces acetylcholine

ANS: C The hallmark characteristics of Parkinson disease (PD) are a result of a loss of dopaminergic-pigmented neurons in the substantia nigra pars compacta with dopaminergic deficiency in the putamen portion of the striatum (the striatum includes the putamen and caudate nucleus). The remaining options are not characteristics of PD. Pages 565-566

The most common clinical manifestation of portal hypertension is what type of bleeding? a. Rectal b. Duodenal c. Esophageal d. Intestinal

ANS: C The vomiting of blood from bleeding esophageal varices is the most common clinical manifestation of portal hypertension. Page 1453

What is the most common opportunistic infection associated with acquired immunodeficiency syndrome (AIDS)? a. Non-Hodgkin lymphoma b. Kaposi sarcoma c. Toxoplasmosis d. Cytomegalovirus

ANS: C Toxoplasmosis is the most common opportunistic infection and occurs in approximately one third of individuals with AIDS. Cytomegalovirus encephalitis is common in those with AIDS but is often not diagnosed while the person is alive. Other neoplasms associated with human immunodeficiency virus (HIV) include systemic non-Hodgkin lymphoma and metastatic Kaposi sarcoma. Page 617

Which medication is used to reverse the effects of malignant hyperthermia? a. Propranolol b. Diazepam c. Dantrolene sodium d. Sodium carbonate

ANS: C Treatment includes the withdrawal of the provoking agents and the administration of dantrolene sodium (a skeletal relaxant that inhibits calcium release during muscle contraction). The other options are not effective in the treatment of malignant hyperthermia. Pages 500-501

What term is used to identify the movement of fluids and solutes from the tubular lumen to the peritubular capillary plasma? a. Tubular secretion b. Ultrafiltration c. Tubular reabsorption d. Tubular excretion

ANS: C Tubular reabsorption is the movement of fluids and solutes from the tubular lumen to the peritubular capillary plasma. This selection is the only option that correctly identifies the process. Page 1328

What is the primary site for uncomplicated local gonococci infections in men? a. Epididymis b. Lymph nodes c. Urethra d. Prostate

ANS: C Uncomplicated local infections are observed primarily as urethral infections in men. Page 921

Which statement is true regarding urodilatin? a. Urodilatin inhibits sodium chloride and water reabsorption in the medullary part of the collecting duct. b. It inhibits antidiuretic hormone (ADH) to prevent water reabsorption in the medullary part of the collecting duct. c. Urodilatin is stimulated by a rise in blood pressure and an increase in extracellular volume. d. It is stimulated by a fall in blood pressure and a decrease in extracellular volume.

ANS: C When the circulating volume and increased blood pressure are increased, the distal tubule and collecting duct produces urodilatin (a natriuretic peptide). Urodilatin inhibits sodium and water reabsorption from the medullary part of collecting duct, thereby producing diuresis. Page 1328 | Table 37-1

How high does the plasma glucose have to be before the threshold for glucose is achieved? a. 126 mg/dl b. 150 mg/dl c. 180 mg/dl d. 200 mg/dl

ANS: C When the plasma glucose reaches 180 mg/dl, as occurs in the individual with uncontrolled diabetes mellitus, the threshold for glucose is achieved. Page 1330

Which renal change is found in older adults? a. Sharp decline in glomerular filtration rate b. Sharp decline in renal blood flow c. Decrease in the number of nephrons d. Decrease in urine output

ANS: C With aging, the number of nephrons decreases. The other options are not necessarily related to aging. Page 1336

Dementia is commonly characterized by the deterioration in which abilities? (Select all that apply.) a. Sociability b. Balance c. Memory d. Speech e. Decision making

ANS: C, D, E Dementia is the progressive failure (an acquired deterioration) of many cerebral functions that include impairment of intellectual function with a decrease in orienting, memory, language, executive attentional functions, and alterations in behavior. Loss of the need for social contact and impaired balance are not associated with dementia, although a person with such a diagnosis may exhibit these deficiencies. Pages 545-546

What are the most common side effects of selective serotonin reuptake inhibitors (SSRIs)? (Select all that apply.) a. Orthostatic hypotension b. Dry mouth c. Sleep disturbances d. Agitation e. Nausea

ANS: C, E Common side effects of SSRIs include sleep disturbances (e.g., insomnia) and nausea. The remaining options are not common side effects of SSRIs. Page 652

Which conditions related to the bladder would result from the effects of lesions of the sacral segments below S1? (Select all that apply.) a. Frequency b. Urge incontinence c. Bladder distension d. Urgency e. Urinary retention

ANS: C, E Lesions that involve the sacral micturition center (below S1; may also be termed cauda equina syndrome) or peripheral nerve lesions result in detrusor areflexia (acontractile detrusor), a lower motor neuron disorder. The result is an acontractile detrusor or atonic bladder with retention of urine and distention. The other options are associated with neurologic lesions that occur between C2 and S1 Page 1345

Which water-soluble vitamins are dependent on sodium for absorption? (Select all that apply.) a. Pantothenic acid b. Vitamin B1 c. Vitamin C d. Vitamin B12 e. Folic acid

ANS: C, E Of the options available, only vitamin C and folic acid are water-soluble vitamins that are dependent on sodium for absorption. Page 1406 | Table 40-2

How is gonorrhea transmitted from a pregnant woman to her fetus? a. Unbound in the blood via the placenta b. Attached to immunoglobulin G (IgG) via the placenta c. Across amniotic membranes by the direct inoculation with the fetal scalp electrodes during labor monitoring d. Predominately through infected cervical and secretions during the birth process

ANS: D A pregnant woman can transmit gonorrhea to her fetus during the birth process. The infection passes from mother to child predominately through infected cervical and vaginal secretions. The transmission is not associated with the other options. Page 920

What causes the vasomotor flushes (hot flashes) that are associated with declining ovarian function with age? a. Decreased estrogen levels b. Absence of estrogen c. Increased estrogen levels d. Rapid changes in estrogen levels

ANS: D A rapid change in estrogen levels (withdrawal or increase), rather than low estrogen levels, induces hot flashes. Pages 794-795

The major sleep center is located in which section of the brain? a. Thalamus b. Brainstem c. Frontal lobe d. Hypothalamus

ANS: D A small group of hypothalamic nerve cells, the suprachiasmatic nucleus (SCN), controls the timing of the sleep-wake cycle and coordinates this cycle with circadian rhythms (24-hour rhythm cycles) in areas of the brain and other tissues. The remaining options do not fulfill this objective. Pages 502-503

Pinkeye is characterized by inflammation of which structure? a. Eyelids b. Sebaceous glands c. Meibomian glands d. Conjunctiva

ANS: D Acute bacterial conjunctivitis (pinkeye) is an inflammation of the conjunctiva (mucous membrane covering the front part of the eyeball). The other structures are not affected by this inflammation. Page 507

Hepatic fat accumulation is observed in which form of cirrhosis? a. Biliary b. Metabolic c. Postnecrotic d. Alcoholic

ANS: D Alcoholic cirrhosis is a complex process that begins with fatty infiltration (hepatic steatosis). Fat deposition (deposition of triglycerides) within the liver hepatocytes is primarily caused by increased lipogenesis and decreased fatty acid oxidation by hepatocytes. This selection is the only option that accurately identifies the correct form of cirrhosis. Pages 1460-1461

The most critical aspect in correctly diagnosing a seizure disorder and establishing its cause is a. Computed tomographic (CT) scan b. Cerebrospinal fluid analysis c. Skull x-ray studies d. Health history

ANS: D Although the history may be supplemented with the remaining options, it remains the pivotal tool for establishing the cause of a seizure disorder. Page 555

Anemia of chronic renal failure can be successfully treated with which element? a. Intrinsic factor b. Vitamin B12 c. Vitamin D d. Erythropoietin

ANS: D Anemia of chronic renal failure can be successfully treated with erythropoietin. Reduced erythropoietin secretion and reduced red cell production are evident in anemia resulting from chronic renal failure. The other options are not relevant to this condition. Page 1365 | Page 1369 | Table 38-13

When caring for a person who has experienced pain for 3 days, anxiety is likely to produce which physical signs that a nurse would expect to find? a. Fever and muscle weakness or reports of fatigue b. Irritability and depression or reports of constipation c. Decreased blood pressure or reports of fatigue d. Increased heart rate and respiratory rate with diaphoresis

ANS: D Anxiety is common in acute pain states and is usually apparent in the alterations of vital signs and can include elevation of blood sugar levels, decreases in gastric acid secretion and intestinal motility, and a general decrease in blood flow to the viscera and skin. Nausea occasionally occurs. The other symptoms are not generally associated with an anxiety response to acute pain. Pages 491-492

Symptoms of benign prostatic hyperplasia (BPH) are a result of which pathophysiologic condition? a. Infection of the prostate b. Obstruction of the urethra c. Ischemia of the urethra d. Compression of the urethra

ANS: D BPH becomes problematic as prostatic tissue compresses the urethra, where it passes through the prostate. Of the selections available, only this option accurately describes the pathophysiologic condition behind the symptoms of BPH. Pages 897-899

Which infection has clinical manifestations that include the sudden onset of malaise, low back pain, and perineal pain with high fever and chills, dysuria, nocturia, and urinary retention? a. Orchitis b. Balanitis c. Epididymitis d. Bacterial prostatitis

ANS: D Bacterial prostatitis can exhibit common manifestations that include a sudden onset of malaise, low back and perineal pain, high fever (up to 40° C [104° F]), and chills, as well as dysuria, inability to empty the bladder, nocturia, and urinary retention. Myalgia and arthralgia also may occur. This selection is the only option that exhibits the symptoms described. Pages 899-900

By which method is the organism that causes syphilis best identified? a. Acid-fast stain b. Gram-stained slide c. In vitro culture d. Darkfield microscopy

ANS: D Because Treponema pallidum cannot be cultured in vitro, early definitive diagnosis of primary or secondary syphilis depends on darkfield microscopy of a specimen taken from a chancre, regional lymph node, or other lesion. The remaining options are not relevant. Page 925

What group is most at risk of spinal cord injury from minor trauma? a. Children b. Adolescents c. Adults d. Older adults

ANS: D Because of preexisting degenerative vertebral disorders, older adults are particularly at risk for minor trauma, resulting in serious spinal cord injury, especially from falls. The risk to the other age groups is less than that of the older adult. Page 634

The equivalent to the female gonad is the male: a. Epididymis b. Spermatic cord c. Vas deferens d. Testes

ANS: D Between 6 to 7 weeks' gestation, the male embryo will differentiate under the influence of TDF. TDF stimulates the male gonads to develop into the two testes. The ovaries, the female gonads, are the primary female reproductive organs. Page 769

The function of arachnoid villi is to: a. Produce cerebrospinal fluid b. Provide nutrients to the choroid plexuses c. Transmit impulses within the meninges d. Absorb cerebrospinal fluid into the cerebral venous sinuses

ANS: D CSF is reabsorbed by means of a pressure gradient between the arachnoid villi and the cerebral venous sinuses. The remaining options do not accurately describe the function of arachnoid villi. Page 466

What process allows the kidney to respond to an increase in workload? a. Glomerular filtration b. Secretion of 1,25-dihydroxyvitamin D3 c. Increased heart rate d. Compensatory hypertrophy

ANS: D Compensatory hypertrophy allows the kidney to respond to an increase in workload throughout life. The remaining options are not relevant to accommodating an increased workload. Page 1336

Cryptorchidism can be defined as which of the following? a. Normal developmental state of the testes b. Abnormal state in which the testes are overdeveloped c. Lack of scrotum d. Testicular maldescent

ANS: D Cryptorchidism is a condition of testicular maldescent, the only option that accurately defines cryptorchidism. Page 892

Which condition is considered a clinical cause of amenorrhea? a. Disorder in the endometrium b. Obstruction of the fallopian tubes c. Lack of physical exercise d. Failure to ovulate

ANS: D Depressed ovarian hormone levels, which are associated with a variety of clinical disorders, also cause amenorrhea by preventing ovulation. This option is the only answer that accurately identifies a clinical cause of cycle irregularities Page 807

Diagnostic criteria for a persistent vegetative state include: a. Absence of eye opening b. Lack of subcortical responses to pain stimuli c. Roving eye movements with visual tracking d. Return of autonomic functions such as gastrointestinal function

ANS: D Diagnostic criteria for vegetative state (VS) include the return of professed vegetative (autonomic) functions, including sleep-wake cycles and normalization of respiratory and digestive system functions. Only the correct option appropriately describes the diagnostic criteria for a VS. Page 534

What is the most immediate result of a small intestinal obstruction? a. Vomiting b. Dehydration c. Electrolyte imbalances d. Distention

ANS: D Distention begins almost immediately, as gases and fluids accumulate proximal to the obstruction. Within 24 hours, up to 8 L of fluid and electrolytes enters the lumen in the form of saliva, gastric juice, bile, pancreatic juice, and intestinal secretions. Copious vomiting or sequestration of fluids in the intestinal lumen prevents their reabsorption and produces severe fluid and electrolyte disturbances. Pages 1431-1432

Which statement is false concerning how abdominal pain is produced? a. Chemical mediators, such as histamine, bradykinin, and serotonin, produce abdominal pain. b. Edema and vascular congestion produce abdominal pain by stretching. c. Ischemia, caused by distention of bowel obstruction or mesenteric vessel thrombosis, produces abdominal pain. d. Low concentrations of anaerobes, such as Streptococci, Lactobacilli, Staphylococci, Enterobacteria, and Bacteroides, produce abdominal pain.

ANS: D Low concentrations of anaerobes are not typically a cause of abdominal pain. Page 1426

Which spinal tract carries the most nociceptive information? a. Archeospinothalamic b. Paleospinothalamic c. Dorsal spinothalamic d. Lateral spinothalamic

ANS: D Most nociceptive information travels by means of ascending columns in the lateral spinothalamic tract (also called the anterolateral funiculus). The other tract options do not carry the most nociceptive information. Page 487

What effect do natriuretic peptides have during heart failure when the heart dilates? a. Stimulates antidiuretic hormones. b. Inhibits antidiuretic hormones. c. Stimulates renin and aldosterone. d. Inhibits renin and aldosterone.

ANS: D Natriuretic peptides inhibit renin and aldosterone during heart failure when the heart dilates. These make up a group of peptide hormones, including atrial natriuretic peptide (ANP), secreted from myocardial cells in the atria and brain natriuretic peptide (BNP) secreted from myocardial cells in the cardiac ventricles. When the heart dilates during volume expansion or heart failure, ANP and BNP inhibit sodium and water absorption by kidney tubules, inhibit the secretion of renin and aldosterone, vasodilate the afferent arterioles, and constrict the efferent arterioles. The result is increased urine formation, leading to decreased blood volume and blood pressure. Page 1327

What is the cause of functional dysphagia? a. Intrinsic mechanical obstruction b. Extrinsic mechanical obstruction c. Tumor d. Neural or muscular disorders

ANS: D Neural or muscular disorders that interfere with voluntary swallowing or peristalsis cause functional dysphagia. This selection is the only option that accurately identifies a cause of functional dysphagia. Page 1428

Obesity is defined as a body mass index (BMI) greater than what measurement? a. 22 b. 25 c. 28 d. 30

ANS: D Obesity is an energy imbalance, with caloric intake exceeding energy expenditure, and is defined as a BMI greater than 30. Page 1446

Which cells in the stomach secrete histamine? a. Oxyntic b. Chief c. D d. Enterochromaffin-like

ANS: D Of the available options, only enterochromaffin-like cells secrete histamine. Page 1398

What type of vomiting is caused by the direct stimulation of the vomiting center by neurologic lesions involving the brainstem? a. Retch b. Periodic c. Duodenal d. Projectile

ANS: D Of the available options, only projectile vomiting is caused by the direct stimulation of the vomiting center by neurologic lesions, such as increased intracranial pressure, tumors, or aneurysms involving the brainstem. Page 1424

A notable complication of panic disorder is: a. Avolition b. Anhedonia c. Alogia d. Agoraphobia

ANS: D Of the available options, the only notable complication of panic disorder is the development of agoraphobia or phobic avoidance of places or situations where escape or help is not readily available. Page 654

What type of cyst develops when an ovarian follicle is stimulated but no dominant follicle develops and completes the maturity process? a. Follicular b. Corpus luteal c. Corpus albicans d. Benign ovarian

ANS: D Only benign cysts of the ovary are produced when a follicle or a number of follicles are stimulated but no dominant follicle develops and completes the maturity process. Page 820

Which hormone promotes the development of the lobular ducts in the breasts? a. Progesterone b. Prolactin c. Oxytocin d. Estrogen

ANS: D Only estrogen promotes the increase in the size of the breasts by the formation of a mass of tissue under the areola, which increases the size and pigmentation of the areola and contributes to the development of the lobular ducts. Page 783

Parkinson and Huntington diseases are associated with defects in which area of the brain? a. Thalamus b. Medulla oblongata c. Cerebellum d. Basal ganglia

ANS: D Parkinson and Huntington diseases are conditions associated with defects of the basal ganglia. No current research supports the role of any of the other options in these diseases. Page 457

What term is used to identify a fibrotic condition that causes lateral curvature of the penis during erection, which is associated with a local vasculitis-like inflammatory reaction and decreased tissue oxygenation? a. Phimosis b. Lateral phimosis c. Lateral paraphimosis d. Peyronie disease

ANS: D Peyronie disease (bent nail syndrome) is a fibrotic condition of the tunica albuginea of the penis, resulting in varying degrees of curvature and sexual dysfunction (see Figure 25-2). Although the exact cause is unknown, a local vasculitis-like inflammatory reaction occurs and decreased tissue oxygenation results in fibrosis and calcification. Peyronie disease is the only term used to identify the pathophysiologic condition described. Pages 889-890

What is a long-term complication of rewarming as a treatment for hypothermia? a. Acidosis b. Dysrhythmias c. Shock d. Renal failure

ANS: D Rewarming can result in long-term complications that include congestive heart failure, hepatic and renal failure, abnormal erythropoiesis, myocardial infarction, pancreatitis, and neurologic dysfunctions. Short-term complications of rewarming include acidosis, rewarming shock, and dysrhythmias. Page 501

In which stage of syphilis would the following clinical manifestations be found: destructive skin, bone and soft tissue lesions, aneurysms, heart failure, and neurosyphilis? a. Primary b. Secondary c. Latent d. Tertiary

ANS: D Stage IV, tertiary syphilis, is the only stage during which significant morbidity and mortality occur, including destructive skin, bone, and soft-tissue lesions (see Box 26-2). Page 924

A right hemisphere embolic CVA has resulted in left-sided paralysis and reduced sensation of the left foot and leg. Which cerebral artery is most likely affected by the emboli? a. Middle cerebral b. Vertebral c. Posterior cerebral d. Anterior cerebral

ANS: D Symptoms of an embolic stroke in only the right anterior cerebral artery would include left-sided contralateral paralysis or paresis (greater in the foot and thigh) and mild upper extremity weakness with mild contralateral lower extremity sensory deficiency with loss of vibratory and/or position sense and loss of two-point discrimination. Page 600 | Table 18-5

Which structure synthesizes clotting factors and the vitamin K necessary for hemostasis? a. Colon b. Spleen c. Gallbladder d. Liver

ANS: D The liver has hemostatic functions. It synthesizes prothrombin, fibrinogen, and factors I, II, VII, IX, and X, all of which are necessary for effective clotting. Vitamin K, a fat-soluble vitamin, is essential for the synthesis of other clotting factors. Because bile salts are needed for the absorption of fats, vitamin K absorption depends on adequate bile production in the liver. Impairment of vitamin K absorption diminishes the production of clotting factors and increases the risk of bleeding. The other structures mentioned are not associated with synthesizing of clotting factors and vitamin K. Page 1412

Reflex activities concerned with heart rate, blood pressure, respirations, sneezing, swallowing, and coughing are controlled by which area of the brain? a. Pons b. Midbrain c. Cerebellum d. Medulla oblongata

ANS: D The medulla oblongata makes up the myelencephalon and is the lowest portion of the brainstem. Reflex activities, such as heart rate, respiration, blood pressure, coughing, sneezing, swallowing, and vomiting, are controlled only in this area. Page 460

What is the most common cause of uncomplicated urinary tract infections? a. Staphylococcus b. Klebsiella c. Proteus d. Escherichia coli

ANS: D The most common infecting microorganisms are uropathic strains of E. coli (80% to 85%). Page 1350

Where is the usual site of cervical dysplasia or cancer in situ? a. Squamous epithelium of the cervix meets the cuboidal epithelium of the vagina. b. Columnar epithelium of the cervix meets the squamous epithelium of the uterus. c. Squamous epithelium of the cervix meets the columnar epithelium of the uterus. d. Columnar epithelium of the cervix meets the squamous epithelium of the vagina.

ANS: D The point at which the columnar epithelium of the cervix meets the squamous epithelium of the vagina is called the transformation zone or the squamous-columnar junction. The transformation zone is especially susceptible to the oncogenic human papillomavirus (HPV), which leads to cervical dysplasia and, ultimately, cervical cancer; these are the cells sampled during a Papanicolaou (Pap) test. Page 775

What is the primary source of physiologic iron? a. Transferrin from plasma b. Pepsin form pepsinogen c. Bile from bilirubin d. Heme from animal protein

ANS: D The primary source of iron is heme from animal protein. The other options are not considered primary sources of physiologic iron. Page 1405

In glomerulonephritis, what damages the epithelial cells resulting in proteinuria? (Select all that apply.) a. Ischemia b. Lysosomal enzymes c. Compression from edema d. Activated complement e. Altered membrane permeability

ANS: D, E Activated complement, inflammatory cytokines, oxidants, proteases, and growth factors attack epithelial cells, alter membrane permeability, and cause proteinuria. None of the other options are responsible for this process. Pages 1353-1357

Which hormones are involved in the relaxation of the stomach's fundus during swallowing? (Select all that apply.) a. Progesterone b. Glucagon c. Motilin d. Gastrin e. Cholecystokinin

ANS: D, E Swallowing causes the fundus to relax (receptive relaxation) to receive a bolus of food from the esophagus. Relaxation is coordinated by efferent, nonadrenergic, noncholinergic vagal fibers and, of the available options, is facilitated only by gastrin and cholecystokinin, two polypeptide hormones secreted by the gastrointestinal mucosa. Page 1397

Varicocele

Abnormal dilation of the vein within the spermatic cord

Identify populations at risk for neurological pathophysiology

Adolescents and young adults 15-35 year old, greater than 70. Men 1.5x as likely to sustain a TBI, persons living in high crime area, military, professional sports players, order of spinal cord trauma (MVA, falls, acts of violence (GSW, recreational sports).

Hypoglossal nerve

Affects control over the motor fibers to the muscles of tongue and sensory impulses from the tongue to the brain.

Describe the pathophysiology of alcoholic cirrhosis.

Alcoholic cirrhosis is a complex process that starts with fatty infiltrates. Fatty infiltration can occur without the development of hepatitis or cirrhosis. Fat deposition is caused by increased lipogenesis and decreased fatty acid oxidation by hepatocytes. Alcoholic cirrhosis is caused by the toxic effects of alcohol metabolism on the liver, immunologic alterations, lipoid peroxidation, and malnutrition. Alcohol can also induce the formation of autoantibodies targeting heptic cells. Cellular damage initiates an inflammatory response. Cytokines related to alcoholic liver disease include TNF-alpha, IL-6, IL-8, and IL-18. Lastly, Kupffer cells produce transforming growth factor beta, which contributes to fibrosis.

Hepatic fat accumulation is seen in which form of cirrhosis?

Alcoholic cirrhosis-fat deposition with in the hepatocytes is caused primarily by increased lipogenesis and decreased fatty acid oxidation by hepatocytes. Cessation of alcohol intake reverses fat accumulation.

Spontaneous speech is absent.

Alogia

Condition is characterized by a deficit in spontaneous or goal-directed activities.

Avolition

A _____ cyst develops when an ovarian follicle is stimulated, but no dominant follicle develops and completes the maturity process.

Benign

The mucosal secretions of the cervix secrete which immunoglobulin? a. IgA b. IgE c. IgG d. IgM

ANS: A Mucosal secretions from the cervix contain enzymes and antibodies—predominantly IgA. Page 775

Which age group should be targeted for testicular cancer education and screening? a. 15 to 35 year olds b. 20 to 45 year olds c. 30 to 55 year olds d. 45 to 70 year olds

ANS: A Overall, testicular cancers are rare, yet they are the most common form of cancer in young men between the ages of 15 and 35 years. Page 895

Saliva contains which immunoglobulin (Ig)? a. IgA b. IgE c. IgG d. IgM

ANS: A Saliva contains only IgA, which helps prevent infection. Page 1395

Testosterone is believed to have a role in: (Select all that apply.) a. Male-patterned baldness b. Libido levels c. Acne development d. Altered cholesterol metabolism e. Thinning of the larynx

ANS: A, B, C, D Testosterone is associated with all the options except thinning of the larynx; it actually stimulates the growth of the larynx's cartilage. Page 789

Which statements are true regarding parietal pain? (Select all that apply.) a. Parietal pain arises from the parietal peritoneum. b. It is generally more localized than visceral pain. c. Parietal pain is usually less intense than visceral pain. d. Nerve fibers that travel to the spinal cord are involved in parietal pain. e. Parietal pain corresponds to dermatomes T6 and L1.

ANS: A, B, D, E Parietal pain arises from the parietal peritoneum and is more localized and intense than visceral pain. Nerve fibers from the parietal peritoneum travel with peripheral nerves to the spinal cord, and the sensation of pain corresponds to skin dermatomes T6 and L1. Page 1426

The size of benign uterine tumors, such as leiomyomas, is thought to be caused by the influence of which hormone? (Select all that apply.) a. Progesterone b. Estrogen c. Luteinizing hormone d. Gonadotropin-stimulating hormone e. Growth factors

ANS: A, B, E The cause of uterine leiomyomas is unknown, although their size appears to be related to only estrogen, progesterone, growth factors, angiogenesis, and apoptosis. Page 822

Which condition poses the highest risk for a cerebrovascular accident (CVA)? a. Insulin-resistant diabetes mellitus b. Hypertension c. Polycythemia d. Smoking

ANS: B Hypertension is the single greatest risk factor for stroke. The other options are recognized risk factors but do not carry the intensity of hypertension. Page 598

What type of diarrhea is a result of lactase deficiency? a. Motility b. Osmotic c. Secretory d. Small-volume

ANS: B Malabsorption related to lactase deficiency, pancreatic enzyme or bile salt deficiency, small intestine bacterial overgrowth, and celiac disease cause osmotic diarrhea. None of the other options are associated with lactase deficiencies. Page 1425

How many stools per day are considered the upper limits of normal? a. Two b. Three c. Five d. Seven

ANS: B More than three stools per day is considered abnormal. Page 1428

Which enzyme breaks down protein-forming polypeptides in the stomach? a. Acetylcholine b. Pepsin c. Gastrin d. Secretin

ANS: B Of the options available, only pepsin, a proteolytic enzyme, breaks down protein-forming polypeptides in the stomach. Page 1399

Which vitamin facilitates the absorption of iron by the epithelial cells of the duodenum and jejunum? a. B6 b. C c. E d. B12

ANS: B Of the options available, only vitamin C reduces ferric iron to ferrous iron, which is the form more easily absorbed by the epithelial cells of the duodenum and jejunum. Page 1405

Which description characterizes visceral pain? a. Is sharp and well-defined when transmitted by A-delta (A) fibers. b. Is perceived as poorly localized and is transmitted by the sympathetic nervous system. c. Arises from connective tissue, muscle, bone, or skin. d. Is perceived as dull, aching, and poorly localized when transmitted by C fibers.

ANS: B Of the options provided, only visceral pain refers to pain in internal organs and the abdomen and is transmitted by sympathetic afferents. Visceral pain is poorly localized because of fewer nociceptors in the visceral structures. Page 492

Which hepatitis virus is known to be sexually transmitted? a. A b. B c. C d. D

ANS: B Only hepatitis B virus (HBV) is known to be sexually transmitted. Page 940

Clinical manifestations of Parkinson disease are caused by a deficit in which of the brain's neurotransmitters? a. Gamma-aminobutyric acid b. Dopamine c. Norepinephrine d. Acetylcholine

ANS: B Parkinson disease is a commonly occurring degenerative disorder involving deficits of dopamine, not of any of the other options. Pages 565-566

In the mouth and stomach, salivary -amylase initiates the digestion of which nutrients? a. Proteins b. Carbohydrates c. Fats d. Fiber

ANS: B Salivary -amylase is an enzyme that initiates only carbohydrate digestion in the mouth and stomach. Page 1395

Which structure regulates eating behavior and energy metabolism? a. Anterior pituitary b. Hypothalamus c. Posterior pituitary d. Parietal lobe

ANS: B The arcuate nucleus (ARC) in the hypothalamus has two sets of neurons with opposing effects that interact to regulate and balance food intake and energy metabolism. This selection is the only option that regulates eating behavior and energy metabolism. Pages 1447-1448

The Skene glands are located on either side of which structure? a. Introitus b. Urinary meatus c. Clitoris d. Vestibule

ANS: B The ducts of the Skene glands (also called the lesser vestibular or paraurethral glands) are related only to the urinary meatus. Page 772

Which statement is true regarding the state of the intestinal tract at birth? a. The intestinal tract is colonized by Escherichia coli. b. The intestinal tract is sterile. c. Clostridium welchii is present in the intestinal tract but in very small numbers. d. Streptococcus colonization in the intestinal tract has begun.

ANS: B The intestinal tract is sterile at birth but becomes colonized with Escherichia coli, Clostridium welchii, and Streptococcus within a few hours. Page 1409

Which statements are true regarding prostate cancer? (Select all that apply.) a. It ranks second to lung cancer as being most common among American men. b. A familial history of prostate cancer is a risk factor. c. Dietary habits seem to play a role in its development. d. African-American men have an increased risk for its development. e. Being over 65 years of age increases the risk for developing prostate cancer.

ANS: B, C, D, E Prostate cancer is the most commonly diagnosed non-skin cancer in American men, and the incidence varies greatly worldwide. Possible causes include a genetic predisposition, environmental and dietary factors, inflammation, and alterations in levels of hormones (e.g., testosterone, dihydrotestosterone, estradiol) and growth factors. Incidence is greatest among northwestern European and North American men (particularly African Americans) older than 65 years of age. Pages 900-910

Which clinical manifestation is characteristic of cluster headaches? (Select all that apply.) a. Preheadache aura b. Severe unilateral tearing c. Gradual onset of a tight band around the head d. Significant unilateral, temporal pain e. Pain lasting from 30 to 120 minutes

ANS: B, C, E The cluster headache attack usually begins without warning and is characterized by severe, unilateral tearing, burning, periorbital, and retrobulbar or temporal pain lasting 30 minutes to 2 hours. Neither preheadache aura nor significant unilateral, temporal pain is clinical manifestation characteristic of cluster headaches. Pages 608-609

Which hormones are involved in regulating gastric motility by making the threshold potential of muscle fibers more negative? (Select all that apply.) a. Estrogen b. Secretin c. Somatostatin d. Gastrin e. Motilin

ANS: B, D, E Gastrin and motilin (small intestine hormones) and the vagus nerve increase contraction by making the threshold potential of muscle fibers less negative. Sympathetic activity and secretin (another small intestine hormone) are inhibitory and make threshold potential more negative. The rate of peristalsis is mediated by pacemaker cells that initiate a wave of depolarization (basic electrical rhythm), which moves from the upper part of the stomach to the pylorus. Neither estrogen nor somatostatin inhibits gastric motility as described in the question. Page 1397

When a woman's uterus is assessed as protruding through the entrance of the vagina to the hymen, which grade of prolapse does this indicate? a. 0 b. 1 c. 2 d. 3

ANS: C A grade 2 prolapse reaches the hymen (see Figure 24-10). Page 818 | Box 24-10

Microinfarcts resulting in pure motor or pure sensory deficits are the result of which type of stroke? a. Embolic b. Hemorrhagic c. Lacunar d. Thrombotic

ANS: C A lacunar stroke (lacunar infarct) is a microinfarct smaller than 1 cm in diameter. Because of the subcortical location and small area of infarction, these strokes may have pure motor and sensory deficits. The other options would not result in the complications described. Page 599

Pain that warns of actual or impending tissue injury is referred to as what? a. Chronic b. Psychogenic c. Acute d. Phantom

ANS: C Acute pain is a protective mechanism that alerts the individual to a condition or experience that is immediately harmful to the body. The remaining options do not fulfill this objective. Pages 491-492

Which antibiotics are considered "major culprits" in causing nephrotoxic acute tubular necrosis (ATN)? a. Penicillin and ampicillin b. Vancomycin and bacitracin c. Gentamicin and tobramycin d. Cefazolin and cefepime

ANS: C Although numerous antibiotics can produce nephrotoxic ATN, the aminoglycosides (gentamicin, tobramycin) are the major culprits. Page 1361

A herniation of which disk will likely result in motor and sensory changes of the lateral lower legs and soles of the feet? a. L2-L3 b. L3-L5 c. L5-S1 d. S2-S3

ANS: C Clinical manifestations of posterolateral protrusions include radicular pain exacerbated by movement and straining (medial calf suggests L5; lateral calf suggests S1 root compression). Herniation of any of the other vertebrae will not result in the described symptoms. Pages 595-596 | Figure 18-14

Cognitive operations cannot occur without the effective functioning of the brain's: a. Pons b. Medulla oblongata c. Reticular activating system d. Cingulate gyrus

ANS: C Cognitive cerebral functions require a functioning reticular activating system (RAS). Cognitive operations are not managed by any of the other options. Page 528

Persistent beliefs are contrary to the educational and cultural background of the individual.

Delusions

Define diffuse cortical dysfunction.

Disease processes (encephalitis) may produce diffuse bilateral cortical dysfunction. Results from the effects of head rotation-the brain experiences shearing stresses resulting in axonal damage ranging from concussion to a severe diffuse axonal injury (DAI) state. Several categories of diffuse brain injury exist, mild concussion, classic concussion, mild DAI, moderate DAI, and severe DAI. Severity of the diffuse injury correlates with the direction and velocity of rotation, how much shearing force was applied to the brainstem. Supratentorial disorders produce a decreased level of arousal by one of three mechanisms: 1) Diffuse bilateral cortical dysfunction, 2) bilateral subcortical dysfunction, 3) localized hemispheric dysfunction. Disease processes may produce diffuse bilateral cortical dysfunction (encephalitis, extracerebral disorders, neoplasms, closed head injury, dementia, and Alzheimer's). May occur in either the cerebral cortex or the underlying white matter. There may be CNS involvement, focal pathologic findings, and obstructive hydrocephalus (CVA, brainstem trauma).

Enhances insulin release, lipolysis, and ketogenesis

Enteroglucagon

Movement of organisms from the urethra into the bladder with infection and retrograde spread to the kidney

Female sexual trauma

Oculomotor nerve

Fibers emerge from the midbrain, exit from the skull, and extend to the eye.

Trochlear nerve

Fibers emerge from the posterior midbrain and exit from the skull to run to the eye

Kidney stones in the upper part of the ureter would produce pain referred to the:______.

Flank, radiating to the groin

Crystals

Form in concentrated acidic or alkaline urine

Define autonomic hyperreflexia.

Individuals most likely to be affected have lesions at the T6 level or above. Autonomic hyperreflexia is characterized by paroxysmal HTN (up to 300 systolic), a pounding HA, blurred vision, sweating above the level of the lesion with flushing of the skin, nasal congestion, nausea, piloerection caused by piolmeter spasm, and bradycardia.

Abducens nerve

Innervates muscles that move the eye laterally

Vagus nerve

Is made up of parasympathetic motor fibers that supply the smooth muscles of the abdominal organs.

Olfactory nerve

Is purely sensory and carries impulses for the sense of smell.

Obstruction and stasis of urine, contributing to bacteremia and hydronephrosis; irritation of epithelial lining with entrapment of bacteria

Kidney stones

_____ are benign uterine tumors that develop from smooth muscle cells in the myometrium and are commonly called uterine fibroids.

Leiomyomas

How does an intestinal obstruction at the pylorus or high in the small intestine causes metabolic alkalosis?

Metabolic alkalosis develops initially as a result of excessive loss of hydrogen ions that normally would be reabsorbed from the gastric juice. Pylorus obstruction is the narrowing or blocking of the opening between the pylorus and the duedum. Vomiting becomes a cardinal sign of obstruction, usually of copious amounts several hours after eating. This prolonged vomiting leads to dehydration, which is accompanied by hypokalemic and hypochloremic metabolic alkalosis caused by loss of K and gastric acid.

The desire to eat is stimulated by what hormone or protein?

Molecules that stimulate eating are called orexins. One set of neurons in the hypothalamus contains the arcuate nucleus ARC) contains 2 sets of neurons: one set stimulates eating and decreases metabolism, while the other does the opposite-inhibits eating and increases metabolism. The group of neurons that stimulates eating and has an anabolic effect (decreasing metabolism) produces neuropeptide Y and agouti-related protein. These neurons activate 2nd order neurons locally, in the hypothalamus, to increase the appetite and decrease metabolism. Further, orexins, also known as hypocretins, are a family of peptides that act to stimulate eating.

Encephalitis

Mosquito-borne viral infection

Pain that is the result of muscle spasms, tenderness, and stiffness and leads to muscle guarding that limits muscle motion

Myofascial pain syndrome

Impairment to the bladder, interfering with normal bladder contraction and causing residual urine and ascending infection

Neurogenic bladder

Pain that is thought to be caused by trauma or disease of nerves and leads to abnormal processing of sensory information by the peripheral and central nervous systems

Neuropathic pain

Cryptococcus neoformans

Opportunistic infection

Decreases pancreatic and enzyme secretions.

Pancreatic polypeptide

What is the most commonly occurring small intestinal obstruction?

The most common cause of SBO is related to adhesions, 50-70%. Peritoneal irritation from surgery or trauma leads to formation of fibrin and adhesions that attach to intestine omentum, or peritoneum and can cause traction and obstruction, most common in small intestine.

Define the types of cerebral edema.

Vasogenic edema is caused by the increased permeability of the capillary endothelium of the brain after injury to the vascular structure. Cytotoxic edema, toxic factors directly affect the cellular elements of the brain parenchyma, causing failure of the active transport system.

Chronic reflux of urine up the ureter and into the kidney during micturition, contributing to bacterial infection

Vesicoureteral reflux

The adult intestine processes approximately _____ L of luminal content per day.

9

The human zygote has a total of how many chromosomes? a. 23 b. 25 c. 46 d. 50

ANS: C A 23-chromosome female gamete, the ovum, and a 23-chromosome male gamete, the spermatozoon (sperm cell), unite to form a 46-chromosome zygote that is capable of developing into a new individual. Page 768

A peptic ulcer may occur in all of the following areas except the: a. Stomach b. Duodenum c. Jejunum d. Esophagus

ANS: C A peptic ulcer is a break, or ulceration, in the protective mucosal lining of the lower esophagus, stomach, or duodenum. This type of ulcer is not associated with the jejunum. Page 1435

What are the clinical manifestations of testicular cancer? a. Firm, nontender testicular mass b. Painful, mobile, firm testicular mass c. Painful fluid-filled testicular mass d. Soft, nontender testicular mass

ANS: A Of the options available, a firm, painless testicular enlargement is commonly identified as the first sign of testicular cancer. Pages 895-896

Which water-soluble vitamin is absorbed by passive diffusion? a. Vitamin B6 b. Vitamin B1 c. Vitamin K d. Folic acid

ANS: A Of the options available, only Vitamin B6 is the water-soluble vitamin absorbed by passive diffusion. Page 1406 | Table 40-2

Which benign breast tumor affects postmenopausal women and is characterized by the principal lactiferous ducts becoming dilated and filled with cellular debris? a. Mammary duct ectasia b. Intraductal papilloma c. Phyllodes tumor d. Fibroadenoma

ANS: A Of the options available, only mammary duct ectasia is associated with the age and the identified pathologic characteristics (see Table 24-9). Page 838 | Table 24-9

What is the first sign of puberty in girls? a. Breast enlargement b. Growth of pubic hair c. Menstruation d. Vaginal discharge

ANS: A Of the options available, the first sign of puberty in girls is usually thelarche or breast development. Page 802

Using a fan to reduce body temperature is an example of which mechanism of heat loss? a. Evaporation b. Radiation c. Convection d. Conduction

ANS: C Only convection causes the transfer of heat through currents of gases or liquids. Page 497

Tactile agnosia is related to injury of which area of the brain? a. Frontotemporal b. Parietal c. Temporal d. Broca area

ANS: B Tactile agnosia (astereognosis) is the inability to recognize objects by touch as a result of damage to the parietal lobe. Tactile agnosia is not related to an injury to any of the other options. Page 542 | Table 17-9

Which clinical manifestations of a urinary tract infection may be demonstrated in an 85-year-old individual? a. Confusion and poorly localized abdominal discomfort b. Dysuria, frequency, and suprapubic pain c. Hematuria and flank pain d. Pyuria, urgency, and frequency

ANS: A Older adults with cystitis may demonstrate confusion or vague abdominal discomfort or otherwise be asymptomatic. Page 1351

Hallucinations, delusions, and thought disorders occur with alterations to which part of the brain? a. Temporal lobe b. Parietal lobe c. Hypothalamus d. Cerebral cortex

ANS: A Only temporal lobe alterations may be responsible for the production of positive schizophrenic symptoms, such as hallucinations, delusions, and thought disorders. Page 642

A Sims-Huhner test is of particular interest to a patient experiencing which condition? a. Infertility b. Possible HIV infection c. Venereal disease d. Perimenopause

ANS: A A Sims-Huhner test evaluates the ability of sperm to penetrate and maintain motility in cervical mucus 2 to 4 hours after coitus approximately 1 day before ovulation. The enzyme-linked immunosorbent assay (ELISA) detects the presence of antibodies to human immunodeficiency virus (HIV). Venereal Disease Research Laboratory (VDRL) is a test for nonspecific venereal diseases. Estradiol levels are associated with erratic or intermittent menstruation. Page 792 | Table 23-3

The onset of schizophrenia can be triggered by which prenatal occurrence? a. Viral infection b. Maternal depression c. Maternal smoking d. Exposure to toxic waste

ANS: A A leading hypothesis for the cause of schizophrenia suggests that the illness results from neurodevelopmental defects that occur in fetal life. Several early environmental factors have been suggested to increase the risk of developing schizophrenia, including viral infection during pregnancy, prenatal nutritional deficiencies, and perinatal complications, such as birth defects and neonatal hypoxia. No current research supports the theory that any of the other options are prenatal triggers of schizophrenia. Page 642

The body compensates for a rise in intracranial pressure by first displacing the: a. Cerebrospinal fluid b. Arterial blood c. Venous blood d. Cerebral cells

ANS: A A rise in intracranial pressure necessitates an equal reduction in the volume of the other contents. The most readily displaced content of the cranial vault is cerebrospinal fluid (CSF), not any of the other options. Pages 555-556

Why does a person who has a spinal cord injury experience faulty control of sweating? a. The hypothalamus is unable to regulate body heat as a result of damage to the sympathetic nervous system. b. The thalamus is unable to regulate body heat as a result of damage to the sympathetic nervous system. c. The hypothalamus is unable to regulate body heat as a result of damage to the parasympathetic nervous system. d. The thalamus is unable to regulate body heat as a result of damage to spinal nerve roots.

ANS: A A spinal cord injury results in disturbed thermal control because the hypothalamus is unable to regulate a damaged sympathetic nervous system. This damage causes faulty control of sweating and radiation through capillary dilation. The other options do not appropriately describe the process that causes faulty control of sweating. Page 592

Which clinical finding is considered a diagnostic indicator for an arteriovenous malformation (AVM)? a. Systolic bruit over the carotid artery b. Decreased level of consciousness c. Hypertension with bradycardia d. Diastolic bruit over the temporal artery

ANS: A A systolic bruit over the carotid in the neck, the mastoid process, or (in a young person) the eyeball is almost always diagnostic of an AVM. The other options are not as indicative as the systolic bruit. Page 605

The concentration of the final urine is determined by antidiuretic hormone (ADH), which is secreted by which gland? a. Posterior pituitary b. Thyroid c. Parathyroid d. Anterior pituitary

ANS: A ADH, which is secreted from the posterior pituitary gland, controls the concentration of the final urine. ADH is not secreted by any of the other options. Pages 1332-1333

Multiple sclerosis and Guillain-Barré syndrome are similar in that they both: a. Result from demyelination by an immune reaction. b. Cause permanent destruction of peripheral nerves. c. Result from inadequate production of neurotransmitters. d. Block acetylcholine receptor sites at the myoneuronal junction.

ANS: A Acute inflammatory demyelinating polyneuropathy (AIDP) accounts for most occurrences of Guillain- Barré syndrome (GBS). Multiple sclerosis (MS) is an autoimmune disorder diffusely involving degeneration of CNS myelin and loss of axons. Only the correct option accurately describes the similarity between MS and GBS Page 618 | Pages 622-623

What local complication of a gonococcal infection is diagnosed in approximately 10% of affected women? a. Acute salpingitis b. Cystitis c. Vaginitis d. Cervicitis

ANS: A Acute salpingitis, or pelvic inflammatory disease (PID), is the most common local complication in women. Approximately 10% of women with untreated cervical gonorrhea develop PID. Page 921

The common property among the three types of medications used to treat depression is that they: a. Increase neurotransmitter levels within the synapse. b. Increase neurotransmitter levels in the presynapse. c. Decrease neurotransmitter levels in the postsynapse. d. Decrease neurotransmitter levels within the synapse.

ANS: A All available antidepressants share the common property, albeit through different mechanisms, that increasing monoamine neurotransmitter levels within the synapse is the basis for their antidepressant effects. The processes by which antidepressants affect depression make the remaining options untrue. Page 648

Which drug may be prescribed orally for outbreak management of herpes simplex viral (HSV) infections? a. Acyclovir (Zovirax) b. 5-Fluorouracil (5-FU) c. Zidovudine (AZT) (Retrovir) d. Bichloroacetic acid (BCA)

ANS: A Although no curative treatment for HSV infection is known, only oral acyclovir, valacyclovir, penciclovir, and famciclovir are used for primary and periodic outbreaks and to prevent recurrences. Page 934

When treating individuals with depression, the result produced by electroconvulsive therapy (ECT) is believed to be an alteration in: a. The monoamine systems b. Serotonin c. Norepinephrine d. The limbic system

ANS: A Although the mechanism of action of ECT is not clear, the procedure is known to produce alterations in only the monoamine systems. Page 652

Which hormone is required for water to be reabsorbed in the distal tubule and collecting duct? a. Antidiuretic hormone b. Aldosterone c. Cortisol d. Adrenocorticotropin hormone

ANS: A Antidiuretic hormone is required for water to be reabsorbed in the distal tubule and collecting duct. The later, straight segment of the distal tubule and the collecting duct are permeable to water as controlled by antidiuretic hormone. The other options are not involved in this process. Page 1331

Goodpasture syndrome is an example of which of the following? a. Antiglomerular basement membrane disease b. Acute glomerulonephritis c. Chronic glomerulonephritis d. Immunoglobulin A (IgA) nephropathy

ANS: A Antiglomerular basement membrane disease (Goodpasture syndrome) is associated with immunoglobulin G (IgG) antibody formation against pulmonary capillary and glomerular basement membranes. Goodpasture syndrome is not an example of any of the other options. Page 1355 | Table 38-5

In adults, most intracranial tumors are located: a. Infratentorially b. Supratentiorially c. Laterally d. Posterolaterally

ANS: A Approximately 70% to 75% of all intracranial tumors diagnosed in adults are located supratentorially (above the tentorium cerebella). The other options are not primary locations for intracranial tumors in adults. Page 626

Which statement is false concerning the pathophysiologic process of alcoholic cirrhosis? a. Inflammation and damage leading to cirrhosis begin in the bile canaliculi. b. Alcohol is transformed to acetaldehyde, which promotes liver fibrosis. c. Mitochondrial function is impaired, decreasing oxidation of fatty acids. d. Acetaldehyde inhibits export of proteins from the liver.

ANS: A Biliary cirrhosis differs from alcoholic cirrhosis in that the damage and inflammation leading to cirrhosis begin in bile canaliculi and bile ducts, rather than in the hepatocytes. The other options provide true information regarding the pathophysiologic process of alcoholic cirrhosis. Pages 1460-1462

The existence of regular, deep, and rapid respirations after a severe closed head injury is indicative of neurologic injury to the: a. Lower midbrain b. Pontine area c. Supratentorial d. Cerebral area

ANS: A Central reflex hyperpnea, which is a sustained deep and rapid but regular respiratory pattern that is the result of central nervous system (CNS) damage or disease, involves the lower midbrain and upper pons. This neurologic injury is observed after increased intracranial pressure and blunt head trauma. Damage to any of the other areas listed would not produce this breathing pattern. Page 530 | Table 17-4

What is a cause of chronic antral gastritis? a. Helicobacter pylori bacteria b. Development of autoantibodies to gastric H+/K+ ATPase c. Pernicious anemia d. Reflux of bile and alkaline pancreatic secretions

ANS: A Chronic antral gastritis generally involves only the antrum and is more common than fundal gastritis. It is caused by H. pylori bacteria or the chronic use of alcohol, tobacco, and nonsteroidal antiinflammatory drugs. None of the other options are associated with the cause of chronic antral gastritis. Page 1435

Considering the pathophysiologic characteristics of primary amenorrhea, what anatomic structure is involved in compartment II? a. Ovary b. Anterior pituitary c. Hypothalamus d. Vagina

ANS: A Compartment II disorders involve only the ovary. Page 805

A blunt force injury to the forehead would result in a coup injury to which region of the brain? a. Frontal b. Temporal c. Parietal d. Occipital

ANS: A Coup injuries occur directly below the point of impact. Objects striking the front of the head usually produce only coup injuries (contusions and fractures) because the inner skull in the occipital area is smooth. A coup injury is not nearly as likely when other portions of the brain are affected. Page 583

Which symptom is characteristic of bulimia nervosa? a. Recurrent episodes of binge eating with fears of not being able to stop eating. b. Fear of becoming obese, despite progressive weight loss. c. Perception that the body is fat when it is actually underweight. d. Absence of three consecutive menstrual periods.

ANS: A Diagnosis of bulimia is based on, among other findings, recurrent episodes of binge eating during which the individual fears not being able to stop. The remaining options are characteristic of anorexia nervosa. Page 1450 | Box 41-5

Which hormones help diminish the febrile response? a. Arginine vasopressin (AVP), melanocyte-stimulating hormone-alpha (-MSH), and corticotropin-releasing factor b. Adrenocorticotropic hormone (ACTH), thyroid-stimulating hormone, and thyroxine (T4) c. Antidiuretic hormone, growth hormone, and aldosterone d. None; hormones only facilitate the increase of core body temperature.

ANS: A During fever, AVP, -MSH, and corticotropin-releasing factors are released from the brain, and antiinflammatory cytokines (e.g., IL-1, IL-10) can act as endogenous cryogens or antipyretics to help diminish the febrile response. The other options are not hormones that diminish the febrile response. Page 498

Electroconvulsive therapy (ECT) is used to treat depression: a. In pregnant women b. By enhancing the effect of antidepressants c. As a first-line treatment before medications d. That is unipolar

ANS: A ECT is used to treat depression when individuals fail to respond to antidepressants or when they are severely depressed, pregnant, suicidal, or psychotic. The other options are not true regarding the conditions of this treatment. Page 652

Which microorganism is sexually transmitted, primarily by homosexual men, through infected feces? a. Shigellosis b. Cytomegalovirus (CMV) c. Giardia lamblia d. Entamoeba histolytica

ANS: A Only the Shigella infection, termed shigellosis, is transmitted by contact with infected feces particularly among homosexual men. Page 940 | Table 26-3

Food enters the stomach via which sphincter? a. Cardiac b. Upper esophageal c. Gastric d. Fundal

ANS: A Each end of the esophagus is opened and closed by a sphincter. The upper esophageal sphincter (cricopharyngeal muscle) prevents entry of air into the esophagus during respiration. The lower esophageal sphincter (cardiac sphincter) prevents regurgitation from the stomach. The lower esophageal sphincter is located near the esophageal hiatus—the opening in the diaphragm where the esophagus ends at the stomach. The cardiac sphincter is the only option that fulfills the function described in the question. Page 1395

Which abnormal laboratory value is found in glomerular disorders? a. Elevated creatinine concentration b. Low blood urea nitrogen (BUN) c. Elevated immunoglobulin A (IgA) d. Low serum complement

ANS: A Elevated creatinine concentration is an abnormal laboratory value found in glomerular disorders. Reduced glomerular filtration rate during glomerular disease is evidenced by elevated plasma urea, creatinine concentration, or reduced renal creatinine clearance. Glomerular disorders are not associated with the other options. Page 1357

When do penile erections begin? a. Before birth b. Shortly after birth c. Shortly before puberty d. After puberty

ANS: A Erections begin in utero and continue throughout life, but ejaculation does not occur until sperm production begins at puberty. Page 787

Diffuse axonal injuries (DAIs) of the brain often result in: a. Reduced levels of consciousness b. Mild but permanent dysfunction c. Fine motor tremors d. Visual disturbances

ANS: A Focal brain injuries account for more than two-thirds of head injury deaths; DAIs accounts for less than one third. However, more severely disabled survivors, including those in an unresponsive state or reduced level of consciousness, have DAIs. The other options do not appropriately complete the stem. Page 582

It is true that Guillain-Barré syndrome (GBS): a. Is preceded by a viral illness. b. Involves a deficit in acetylcholine. c. Results in asymmetric paralysis. d. Is an outcome of HIV.

ANS: A GBS is considered to be an autoimmune disease triggered by a preceding bacterial or viral infection. None of the other options are true of GBS. Pages 622-623

When a patient has small, vesicular lesions that last between 10 and 20 days, which sexually transmitted infection is suspected? a. Genital herpes b. Chancroid c. Syphilis d. Chlamydia

ANS: A If symptoms occur, the individual may have small (1 to 2 mm), multiple, vesicular lesions that are generally located on the labia minora, fourchette, or penis. They may also appear on the cervix, buttocks, and thighs and are often painful and pruritic. These lesions usually last approximately 10 to 20 days. The other options do not demonstrate these symptoms. Page 933

What is the function of the mucus secreted by the Bartholin glands? a. Enhancement of the motility of sperm b. Lubrication of the urinary meatus and vestibule c. Maintenance of an acid-base balance to discourage proliferation of pathogenic bacteria d. Enhancement of the size of the penis during intercourse

ANS: A In response to sexual stimulation, the Bartholin glands secrete mucus that serves only to lubricate the inner labial surfaces, as well as to enhance the viability and motility of sperm. Page 772

In women, what is the usual site of original gonococcal infection? a. Endocervical canal b. Vagina c. Fallopian tube d. Labia majora

ANS: A In women, the endocervical canal (inner portion of the cervix) is the usual site of original gonococcal infection, although urethral colonization and infection of Skene or Bartholin glands also are common. The other options are not usually associated with gonococcal infections. Page 921

Which virus is a precursor for developing cervical intraepithelial neoplasia (CIN) and cervical cancer? a. Human papillomavirus (HPV) b. Epstein-Barr virus (EBV) c. Herpes simplex II virus (HSV) d. Cytomegalovirus (CMV)

ANS: A Infection with high-risk (oncogenic) types of HPV (predominantly 16 and 18) is a necessary precursor to the development of precancerous dysplasia of the cervix that leads to invasive cancer. The other options are not precursors to CIN and cervical cancer. Page 825

What is the most common infratentorial brain disease process that results in the direct destruction of the reticulating activation system (RAS)? a. Cerebrovascular disease b. Demyelinating disease c. Neoplasms d. Abscesses

ANS: A Infratentorial disorders produce a decline in arousal through a direct destruction of the RAS and its pathways. The most common cause of direct destruction is cerebrovascular disease, but demyelinating diseases, neoplasms, granulomas, abscesses, and head injury also may cause brainstem destruction by tissue compression. Page 528

Loud snoring, a decrease in oxygen saturation, fragmented sleep, chronic daytime sleepiness, and fatigue are clinical manifestations of which sleep disorder? a. Obstructive sleep apnea b. Upper airway resistance syndrome c. Somnambulism d. Narcolepsy

ANS: A Obstructive sleep apnea is characterized by repetitive increases in resistance to airflow within the upper airway with loud snoring, gasping, intervals of apnea lasting from 10 to 30 seconds, fragmented sleep, and chronic daytime sleepiness and fatigue, as well as a decrease in oxygen saturation. The remaining options do not exhibit the signs and symptoms listed in the stem. Page 504

What data confer the link between bipolar disorders and schizophrenia? a. Individuals with bipolar disorder who exhibit psychotic behaviors have deficits in reelin expression linked to genetic loci located on chromosome 22. b. Individuals with schizophrenia who exhibit psychotic behaviors have deficits in serotonin linked to genetic loci located on chromosome 16. c. Individuals with bipolar disorder who exhibit psychotic behaviors have deficits in gamma-aminobutyric acid (GABA) linked to genetic loci located on chromosome 20. d. Individuals with schizophrenia who exhibit psychotic behaviors have deficits in reelin expression linked to genetic loci located on chromosome 18.

ANS: A Interestingly, loci on chromosomes 18 and 22 have been linked to bipolar disorder and schizophrenia. Individuals with bipolar disorder, who may exhibit psychotic behavior, have deficits in reelin expression linked to genetic loci located on chromosome 22, which confers susceptibility to schizophrenia. The remaining options do not appropriately describe the link between bipolar disorder and schizophrenia. Pages 647-648

What is the normal intracranial pressure (in mm Hg)? a. 5 to 15 b. 7 to 20 c. 12 to 14 d. 80 to 120

ANS: A Intracranial pressure is normally 5 to 15 mm Hg or 60 to 180 cm water (H2O). The remaining options reflect increased intracranial pressure. Pages 555-556

By what mechanism does intussusception cause an intestinal obstruction? a. Telescoping of part of the intestine into another section of intestine, usually causing strangulation of the blood supply b. Twisting the intestine on its mesenteric pedicle, causing occlusion of the blood supply c. Loss of peristaltic motor activity in the intestine, causing an adynamic ileus d. Forming fibrin and scar tissue that attach to the intestinal omentum, causing obstruction

ANS: A Intussusception is the telescoping of part of the intestine into another section of intestine, usually causing strangulation of the blood supply. This selection is the only option that accurately describes how intussusception causes an intestinal obstruction. Page 1431 | Table 41-2

Massage therapy relieves pain by closing the pain gate with the stimulation which fibers? a. A b. A c. B d. C

ANS: A Massaging stimulates different A fibers to close the pain gate. The remaining options do not fulfill this objective. Page 485

Which type of precocious puberty causes the child to develop some secondary sex characteristics of the opposite sex? a. Mixed b. Incomplete c. Isosexual d. Homosexual

ANS: A Mixed precocious puberty (i.e., virilization of a girl or feminization of a boy) causes the child to develop some secondary sex characteristics of the opposite sex. This selection is the only option that accurately identifies the type of precocious puberty described. Page 887

Which type of precocious puberty causes the child to develop some secondary sex characteristics of the opposite sex? a. Mixed b. Incomplete c. Isosexual d. Homosexual

ANS: A Mixed precocious puberty, which is virilization of a girl or feminization of a boy, causes the child to develop some secondary sex characteristics of the opposite sex. This option is the only answer that accurately identifies the type of precocious puberty described. Page 804

Which statement is consistent with dumping syndrome? a. Dumping syndrome usually responds well to dietary management. b. It occurs 1 to 2 hours after eating. c. Constipation is often a result of the dumping syndrome. d. It can result in alkaline reflux gastritis.

ANS: A Most individuals with the dumping syndrome respond well to dietary management. None of the other options is associated with the dumping syndrome. Page 1440

Multiple sclerosis is best described as a(an): a. Central nervous system demyelination, possibly from an immunogenetic virus b. Inadequate supply of acetylcholine at the neurotransmitter junction as a result of an autoimmune disorder c. Depletion of dopamine in the central nervous system as a result of a virus d. Degenerative disorder of lower and upper motor neurons caused by viral-immune factors

ANS: A Multiple sclerosis (MS) is an autoimmune disorder diffusely involving the degeneration of central nervous system (CNS) myelin and loss of axons. MS is described as occurring when a previous infectious insult to the CNS has occurred in a genetically susceptible individual with a subsequent abnormal immune response in the CNS. The other options do not adequately describe MS. Pages 618-619

Which statement is false regarding the risk of transmission of the herpes simplex virus (HSV) from mother to fetus? a. Neonatal infection of HSV rarely occurs in the intrapartum or postpartum period. b. The risk is higher in women who have a primary HSV infection. c. The risk is higher in women who experience ruptured membranes more than 6 hours before delivery. d. The risk is higher when internal fetal monitoring devices are used.

ANS: A Neonatal infections can occur in utero or, more commonly, during the intrapartum or postpartum period. The other options are accurate statements. Page 933

What medical term is used to identify a functional urinary tract obstruction caused by an interruption of the nerve supply to the bladder? a. Neurogenic bladder b. Obstructed bladder c. Necrotic bladder d. Retrograde bladder

ANS: A Neurogenic bladder is a general term for bladder dysfunction caused by neurologic disorders. The types of dysfunction are related to the sites in the nervous system that control sensory and motor bladder function (see Figure 38-3). None of the other options correctly identify the described condition. Pages 1344-1345

Which neurotransmitters inhibit pain in the medulla and pons? a. Norepinephrine and serotonin b. Gamma-aminobutyric acid (GABA) and aspartate c. Glutamate and tumor necrosis factor-alpha d. Neurokinin A and nitric oxide

ANS: A Norepinephrine and serotonin (5-hydroxytryptamine) contribute to pain modulation (inhibition) in the medulla and pons. The remaining options do not fulfill this objective. Page 490

Which immunoglobulin is contained in breast milk? a. IgA b. IgE c. IgG d. IgM

ANS: A Not only does breast milk composition change over time to meet the changing digestive capabilities and nutritional requirements of the infant, but it also contains immune cells, specific immunoglobulins, especially IgA, and nonspecific antimicrobial factors, such as lysozymes and lactoferrin, that protect the infant against infection, allergies, and asthma. Page 784

Which gastric cells secrete hydrochloric acid and intrinsic factor? a. Parietal b. Chief c. G d. H

ANS: A Of the available options, only the parietal cells (oxyntic cells) secrete hydrochloric acid and intrinsic factor. Page 1398

Which pain theory proposes that a balance of impulses conducted from the spinal cord to the higher centers in the central nervous system (CNS) modulates the transmission of pain? a. GCT b. Pattern theory c. Specificity theory d. Neuromatrix theory

ANS: A Only the gate control theory (GCT) explains that a balance of impulses conducted to the spinal cord, where cells in the substantia gelatinosa function as a spinal gate, regulates pain transmission to higher centers in the CNS. Page 485

Which factor increases the risk for ovarian cancer after the age of 40 years? a. Use of fertility drugs b. Oral contraceptive use c. Multiple pregnancies d. Prolonged lactation

ANS: A Ovarian cancer in women older than 40 years of age is associated with early menarche, late menopause, nulliparity, and the use of fertility drugs. The other options are not necessarily related to women older than the age of 40 years. Page 831

How can abdominal pain that is visceral in nature best be described? a. Abdominal pain that is visceral in nature is diffused, vague, poorly localized, and dull. b. It travels from a specific organ to the spinal cord. c. The pain lateralizes from only one side of the nervous system. d. Abdominal pain is associated with the peristalsis of the gastrointestinal tract.

ANS: A Pain is usually felt near the midline in the epigastrium (upper midabdomen), midabdomen, or lower abdomen. The pain is poorly localized, is dull rather than sharp, and is difficult to describe. None of the other options accurately describe this type of pain. Page 1426

What is the term that denotes the duration of time or the intensity of pain that a person will endure before outwardly responding? a. Tolerance b. Perception c. Threshold d. Dominance

ANS: A Pain tolerance is the duration of time or the intensity of pain that an individual will endure before initiating overt pain responses. The other options are not related to the duration or intensity of pain endured before the pain is recognized. Page 491

Which pathway carries sensory information toward the central nervous system (CNS)? a. Ascending b. Descending c. Somatic d. Efferent

ANS: A Peripheral nerve pathways can be afferent (ascending) pathways that carry sensory impulses toward the CNS. The remaining options do not carry sensory information to the CNS. Page 448

Exposure to which substance protects the mucosal barrier of the stomach? a. Prostaglandins b. Aspirin c. Helicobacter pylori d. Regurgitated bile

ANS: A Prostaglandins and enterogastrones, such as gastric inhibitory peptide, somatostatin, and secretin, inhibit acid secretion. This selection is the only option that accurately identifies a substance that protects the mucosal barrier of the stomach. Page 1399

Pyelonephritis is usually caused by which type of organism? a. Bacteria b. Fungi c. Viruses d. Parasites

ANS: A Pyelonephritis is usually caused by the bacteria Escherichia coli, Proteus, or Pseudomonas. Pages 1351-1352

Heat loss from the body via radiation occurs by: a. Emanations of electromagnetic waves b. Transfer of heat through currents of liquids or gas c. Dilation of blood vessels bringing blood to skin surfaces d. Direct heat loss from molecule-to-molecule transfer

ANS: A Radiation refers to heat loss through electromagnetic waves. None of the other options accurately describes heat loss via radiation. Page 497

Renal cell carcinoma, classified as clear cell tumors, arises from epithelial cells in which structure? a. Proximal tubule b. Distal tubule c. Nephron d. Glomerulus

ANS: A Renal cell carcinoma, classified as clear cell tumors according to cell type and extent of metastasis, arises from the proximal tubular epithelium. These tumors are not associated with the other options. Page 1347

What is the major concern regarding the treatment of gonococci infections? a. Development of antibiotic resistance b. Changes in virulence c. Changes in pathogenicity d. Mutations into different strains

ANS: A Several types of drug-resistant strains have been identified; they are penicillinase-producing Neisseria gonorrhoeae (PPNG), which is resistant to penicillin; tetracycline-resistant N. gonorrhoeae (TRNG), which is resistant to tetracycline; chromosomal control of mechanisms of resistance of N. gonorrhoeae (CMRNG), which is resistant to penicillin and tetracycline; and increasingly a fluoroquinolone-resistant N. gonorrhoeae (QRNG). The other options are not major concerns. Page 922

a. Sodium b. Phosphate c. Potassium d. Chloride

ANS: A Sodium passes through the tight junctions and is actively transported across cell membranes. Sodium and glucose share a common active transport carrier (sodium-glucose ligand transporter 1 [SGLT1]). This statement is not true of any other option. Page 1402

What stimulates the desire to eat? a. Agouti-related protein (AgRP) b. Alpha-melanocyte-stimulating hormone (-MSH) c. Cocaine- and amphetamine-regulated transcript (CART) d. Peptide YY (PYY)

ANS: A Specific neurons produce neuropeptide Y (NPY) and AgRP, which stimulates eating and decreases metabolism (anabolic). Page 1448

Regarding the formation of renal calculi, what function does pyrophosphate, potassium citrate, and magnesium perform? a. They inhibit crystal growth. b. Pyrophosphate, potassium citrate, and magnesium stimulate the supersaturation of salt. c. They facilitate the precipitation of salts from a liquid to a solid state. d. Pyrophosphate, potassium citrate, and magnesium enhance crystallization of salt crystals to form stones.

ANS: A Stone or crystal growth inhibiting substances, including potassium citrate, pyrophosphate, and magnesium, are capable of crystal growth inhibition. They are not capable of the functions stated by the other options. Page 1343

Up to how many liters of fluid per hour may be lost by sweating? a. 2 b. 4 c. 6 d. 8

ANS: A Sweating may cause as much as 2.2 L of fluid per hour to be lost. Page 497

Hypothyroidism, edema, hyperlipidemia, and lipiduria characterize which kidney disorder? a. Nephrotic syndrome b. Acute glomerulonephritis c. Chronic glomerulonephritis d. Pyelonephritis

ANS: A Symptoms of nephrotic syndrome include edema, hyperlipidemia, lipiduria, vitamin D deficiency, and hypothyroidism. These symptoms do not support the other options. Page 1359

Which glycoprotein protects against urolithiasis and is a ligand for lymphokines? a. Uromodulin b. Nephrin c. Urodilatin d. Cystatin

ANS: A Tamm-Horsfall glycoprotein, also known as uromodulin, is the most abundant urinary protein, protects against bacterial adhesion and urolithiasis, and is a ligand for lymphokines. This statement is not true of the other options. Page 1331

Antipsychotic drugs cause tardive dyskinesia by mimicking the effects of increased: a. Dopamine b. Gamma-aminobutyric acid c. Norepinephrine d. Acetylcholine

ANS: A The antipsychotic drugs cause denervation hypersensitivity, which mimics the effect of too much dopamine. None of the other options produce such an affect. Page 562

Where in the CNS does a person's learned pain response occur? a. Cerebral cortex b. Frontal lobe c. Thalamus d. Limbic system

ANS: A The cognitive-evaluative system overlies the individual's learned behavior concerning the experience of pain and can modulate the perception of pain and is mediated only through the cerebral cortex. Page 487

The glomerular filtration rate is directly related to which factor? a. Perfusion pressure in the glomerular capillaries b. Diffusion rate in the renal cortex c. Diffusion rate in the renal medulla d. Glomerular active transport

ANS: A The filtration of the plasma per unit of time is known as the glomerular filtration rate (GFR), which is directly related to only the perfusion pressure in the glomerular capillaries. Page 1326

A blunt force injury to the forehead would result in a contrecoup injury to which region of the brain a. Frontal b. Temporal c. Parietal d. Occipital

ANS: A The focal injury produces a contrecoup (on the pole opposite the site of impact) injury. The frontal portion of the brain is opposite of the site of impact. Objects striking the back of the head usually result in both coup and contrecoup injuries because of the irregularity of the inner surface of the frontal bones. A contrecoup injury is not nearly as likely when other portions of the brain are affected. Page 583

Both oligodendroglia and Schwann cells share the ability to: a. Form a myelin sheath b. Remove cellular debris c. Transport nutrients d. Line the ventricles

ANS: A The function of oligodendroglia (oligodendrocytes) is to deposit myelin within the central nervous system (CNS). Oligodendroglia are the CNS counterpart of Schwann cells. The remaining options are not reflective of the common function of these structures. Pages 449-450

What provides the best estimate of the functioning of renal tissue? a. Glomerular filtration rate b. Hourly urine output c. Serum blood urea nitrogen and creatinine d. The specific gravity of the solute concentration of the urine

ANS: A The glomerular filtration rate provides the best estimate of the level of functioning of renal tissue. The other options are not used to assess renal tissue function. Page 1334

What is the role of the normal intestinal bacterial flora? a. Metabolizing bile salts, estrogens, and lipids b. Breaking down proteins into amino acids c. Facilitating the motility of the colon d. Metabolizing aldosterone and insulin

ANS: A The intestinal bacteria play a role in the metabolism of bile salts, contributing to the intestinal reabsorption of bile and the elimination of toxic bile metabolites. These bacteria also play a role in the metabolism of estrogens, androgens, and lipids, as well as in the conversion of unabsorbed carbohydrates to absorbable organic acids, the synthesis of vitamin K2, and the metabolism of various nitrogenous substances and drugs. This selection is the only option that accurately describes the role of normal intestinal bacterial flora. Page 1409

What part of the brain provides the emotional response to pain? a. Limbic system b. Parietal lobe c. Thalamus d. Hypothalamus

ANS: A The limbic and reticular tracts are involved in alerting the body to danger, initiating arousal of the organism, and emotionally processing the perceived afferent signals, not just as stimuli, but also as pain. The remaining options do not fulfill this objective. Page 487

The majority of the small percentage of ovarian cancers that are associated with a known pattern of inheritance are associated with: a. Susceptibility of the BRCA1 gene b. Mutations of the BRCA2 gene c. Hereditary nonpolyposis colorectal cancer (HNPCC) syndrome d. Low progesterone levels

ANS: A The majority (approximately 90%) of ovarian cancers are sporadic and not associated with a known pattern of inheritance. Of the 5% to 10% that have a familial component, the majority are associated with the breast cancer susceptibility gene 1 (BRCA1) and a smaller number with mutations of the BRCA2 or mismatched repair genes (HNPCC syndrome). Low progesterone levels are not associated with ovarian cancers. Page 831

Which of the following is not considered a cause of galactorrhea? a. Proliferation of the lactiferous ducts of the breast b. Hypothyroidism, resulting from a decrease in thyroid-releasing hormone c. Excess prolactin secretion from the pituitary d. Drugs such as high-dose oral contraceptives and phenothiazines

ANS: A The most common cause of galactorrhea is nonpuerperal hyperprolactinemia, or excessive amounts of prolactin. A variety of exogenous agents (such as drugs) and disorders can trigger one of these three mechanisms, thereby causing hyperprolactinemia. Hypothyroidism causes increased secretion of hypothalamic thyroid-releasing hormone, which stimulates the release of prolactin from the pituitary. The proliferation of lactiferous breast ducts is not associated with galactorrhea. Pages 836-837

Which is a characteristic lesion of secondary syphilis? a. Condylomata lata b. Gummas c. Chancroid d. Donovan bodies

ANS: A The only secondary syphilis lesion is the condylomata lata. Pages 924-925

The only surface inside the nephron where cells are covered with microvilli to increase the reabsorptive surface area is called the: a. Proximal convoluted tubules b. Distal tubules c. Ascending loop of Henle d. Descending loop of Henle

ANS: A The only surface inside the nephron where the cells are covered with microvilli (a brush border) is called the proximal convoluted tubules. This proximal convoluted tubular lumen consists of one layer of cuboidal cells with a surface layer of microvilli that increases the reabsorptive surface area. Page 1322

How does the epididymis become infected? a. The pathogenic microorganisms ascend the vasa deferentia from an already infected urethra or bladder. b. The pathogenic microorganisms are attached to sperm that travel through the genital tract. c. The pathogenic microorganisms from the tunica vaginalis are transported to the epididymis. d. The pathogenic microorganisms from the prostate fluid ascend to the epididymis.

ANS: A The pathogenic microorganisms usually reach the epididymis by ascending the vasa deferentia from an already infected urethra or bladder. Of the selections available, this is the only option that accurately describes how the epididymis becomes infected. Page 897

The formation of water-soluble molecules to facilitate the absorption of the by-products of lipid hydrolysis is accomplished by: a. Micelles b. Phospholipase c. Chylomicrons d. Colipase

ANS: A The products of lipid hydrolysis must be made water soluble if they are to be efficiently absorbed from the intestinal lumen. The formation of water-soluble molecules known as micelles (see Figure 40-14) accomplishes this process. This selection is the only option that accurately identifies the molecule needed to facilitate lipid hydrolysis. Page 1404

What structure in the male lies posterior to the urinary bladder? a. Seminal vesicles b. Prostate glands c. Cowper glands d. Parabladder glands

ANS: A The seminal vesicles are a pair of glands, each measuring approximately 4 to 6 cm long, which lie behind the urinary bladder and in front of the rectum. None of the other structures lie in this location. Page 788

What are the expected changes in sleep patterns of older adults? a. Older adults experience difficulty falling asleep with less time spent in REM sleep. b. They experience sound sleep during the night with approximately 50% of the time spent in REM sleep and dreaming. c. Older men commonly experience interrupted sleep patterns later in life than do older women. d. Older adults awaken often but with a rapid return to sleep; they awaken refreshed but often later in the morning.

ANS: A The sleep pattern of the older adult differs from the younger adult in that total sleep time is decreased, and the older individual takes longer to initiate and maintain sleep. Older adults tend to go to sleep earlier in the evening and awaken more frequently during the night and earlier in the morning. Rapid eye movement (REM) and slow-wave sleep decreases. The alteration in sleep pattern typically appears approximately 10 years later in women than it does in men. Page 504

The gate in the GCT of pain is located in the: a. Substantia gelatinosa b. Marginal layer c. Nucleus proprius d. Dorsolateral tract of Lissauer

ANS: A The synaptic connections between the cells of the primary- and secondary-order neurons located in the substantia gelatinosa and other Rexed laminae function as a pain gate. The remaining options do not act in this function. Page 487

The membrane that separates the brain's cerebellum from its cerebrum is the: a. Tentorium cerebelli b. Falx cerebri c. Arachnoid membrane d. Falx cerebelli

ANS: A The tentorium cerebelli is a membrane that separates the cerebellum below from the cerebral structures above. The remaining options do not perform the function described in the stem. Page 464

Which treatment is used for trichomoniasis? a. Topical application of 5-Fluorouracil (5-FU) b. Topical application of acyclovir c. Systemic metronidazole d. Systemic tetracycline

ANS: A The treatment of choice for trichomoniasis is a single 2-gram dose of metronidazole (Flagyl) or tinidazole. The other options are not applicable. Page 938

Based on an understanding of the physiologic process of nociceptors, the nurse expects which surgical procedure to create more pain? a. Repair of several crushed fingers b. External fixation of a dislocated shoulder c. Cyst removal on the internal surface of an ovary d. Repair of a ruptured spleen

ANS: A The variable nature and distribution of nociceptors affect the relative sensitivity to pain in different areas of the body; the tips of the fingers have more nociceptors than the skin on the back, and all skin has many more nociceptors than the internal organs including bone. Pages 485-486

The sudden apparent arousal in which a child expresses intense fear or another strong emotion while still in a sleep state characterizes which sleep disorder? a. Night terrors b. Insomnia c. Somnambulism d. Enuresis

ANS: A Three types of parasomnias include arousal disorders such as confusional arousals, sleepwalking (somnambulism), and night terrors (dream anxiety attacks). The remaining options do not involve a sense or expression of fear or any other strong emotion. Page 505

What event is most likely to occur to the brain in a classic cerebral concussion? a. Brief period of vital sign instability b. Cerebral edema throughout the cerebral cortex c. Cerebral edema throughout the diencephalon d. Disruption of axons extending from the diencephalon and brainstem

ANS: A Transient cessation of respiration can occur with brief periods of bradycardia, and a decrease in blood pressure occurs, lasting 30 seconds or less. Vital signs stabilize within a few seconds to within normal limits. The other options do not accurately describe an event associated with a classic cerebral concussion. Page 588

Uncal herniation occurs when: a. The hippocampal gyrus shifts from the middle fossa through the tentorial notch into the posterior fossa. b. The diencephalon shifts from the middle fossa straight downward through the tentorial notch into the posterior fossa. c. The cingulate gyrus shifts under the falx cerebri. d. A cerebellar tonsil shifts through the foramen magnum.

ANS: A Uncal herniation (i.e., hippocampal herniation, lateral mass herniation) occurs when the uncus or hippocampal gyrus (or both) shifts from the middle fossa through the tentorial notch into the posterior fossa. This shift results in the compression of the ipsilateral third cranial nerve (CN), impairing parasympathetic function. This impairment is carried on in the periphery of the nerve, then in the contralateral third CN, and finally in the mesencephalon, inducing coma. The other options do not appropriately describe when uncal herniation occurs. Page 557 | Box 17-4

Which urine characteristics are indicative of acute tubular necrosis (ATN) caused by intrinsic (intrarenal) failure? a. Urine sodium >30 mEq/L b. Urine osmolality >500 mOsm c. Fractional excretion of sodium (FENa) <1% d. Urine sediment has no cells, some hyaline casts

ANS: A Urine sodium >30 mEq/L is the only option indicative of ATN. Page 1363 | Table 38-11

Where in the brain is the vomiting center located? a. Hypothalamus b. Medulla oblongata c. Pons d. Midbrain

ANS: B The vomiting center of the brain lies in the medulla oblongata. The other locations listed are not related to vomiting. Page 1424

Vomiting is associated with central nervous system (CNS) injuries that compress which of the brain's anatomic locations? a. Vestibular nuclei in the lower brainstem b. Floor of the third ventricle c. Any area in the midbrain d. Diencephalon

ANS: A Vomiting, yawning, and hiccups are complex reflexlike motor responses that are integrated by neural mechanisms in the lower brainstem. Vomiting often accompanies CNS injuries that involve the vestibular nuclei. The remaining options will not trigger vomiting when compressed. Page 533

Which statement is incorrect regarding gallbladder function? a. Within 30 minutes of eating, the gallbladder forces bile into the stomach. b. Cholinergic branches of the vagus nerve mediate gallbladder contraction. c. Cholecystokinin provides hormonal regulation of gallbladder contraction. d. The sphincter of Oddi controls the flow of bile from the gallbladder.

ANS: A Within 30 minutes after eating, the gallbladder begins to contract and the sphincter of Oddi relaxes, forcing bile into the duodenum through the major duodenal papilla. The other options are correct statements regarding the functioning of the gallbladder. Page 1413

Which statements are true regarding the ileogastric reflex? (Select all that apply.) a. The purpose of ileogastric reflex is to inhibit gastric motility. b. Ileum distension triggers the ileogastric reflex. c. The ileogastric reflex causes the relaxation of the ileocecal sphincter. d. Increased gastric secretion triggers the ileogastric reflex. e. The ileogastric reflex stimulates an increase in ileal motility.

ANS: A, B The ileogastric reflex inhibits gastric motility when the ileum becomes distended. The gastroileal reflex, which is activated by an increase in gastric motility and secretion, stimulates an increase in ileal motility and the relaxation of the ileocecal sphincter. Page 1407

Why are children more susceptible to heat stroke than are adults? (Select all that apply.) a. Children produce more metabolic heat when exercising. b. They have more surface area-to-mass ratio. c. Children have less sweating capacity. d. They an underdeveloped hypothalamus. e. Children have an overdeveloped ability to perceive heat.

ANS: A, B, C Children are more susceptible to heat stroke than are adults because (1) they produce more metabolic heat when exercising, (2) they have a greater surface area-to-mass ratio, and (3) their sweating capacity is less than that of adults. The remaining options are not true of a child. Page 500

A nurse caring for an older adult patient would expect which functional changes to occur? (Select all that apply.) a. Increased risk for falls b. Increased risk for falls c. Loss of muscle in the arms and legs d. Decrease in the need for social interaction e. Increased agitation and frustration levels

ANS: A, B, C Functional changes with aging include skeletal muscle atrophy, progressive deficits in taste and smell, and a decrease in neuromuscular control with changes in gait and posture. Neither increased agitation and frustration nor a decreased need for social interaction is considered an expected change resulting from the aging process. Page 478 | Box 15-5

What are considered risk factors for developing bladder and kidney cancers? (Select all that apply.) a. Cigarette smoking b. Hypertension c. Exposure to aniline dyes d. Below normal body weight e. Male gender

ANS: A, B, C Risk factors for renal cancer include cigarette smoking, obesity, and hypertension. The risk of primary bladder cancer is greater among people who smoke or those who are exposed to metabolites of aniline dyes or other aromatic amines or chemicals and with heavy consumption of phenacetin. Gender is not a recognized risk factor. Pages 1347-1349

What evidence does the nurse expect to see when a patient experiences trauma to the hypothalamus? (Select all that apply.) a. Uneven expression of mood b. Unstable blood glucose levels c. Poor regulation of body temperature d. Visual disturbances such as blurred vision e. Nausea, vomiting, and symptoms of gastroesophageal reflux disease

ANS: A, B, C The hypothalamus forms the base of the diencephalon. Hypothalamic function controls autonomic nervous system (ANS) function, regulation of body temperature, endocrine function (e.g., unstable glucose levels), and regulation of emotional expression. Visual and gastrointestinal symptoms would not likely be related to hypothalamus function. Page 459 | Box 15-3

Which hormones are produced by the kidney? (Select all that apply.) a. Renin b. Erythropoietin c. 1,25-dihydroxyvitamin D3 d. Calcitonin e. Aldosterone

ANS: A, B, C The kidney also has an endocrine function, secreting the hormones renin, erythropoietin, and 1,25-dihydroxyvitamin D3 for the regulation of blood pressure, erythrocyte production, and calcium metabolism, respectively. Neither calcitonin nor aldosterone is produced by the kidney. Page 1319

Heat exhaustion results in: (Select all that apply.) a. Profuse sweating b. Profound vasodilation c. A need to ingest warm liquids d. Permanent damage to the hypothalamus e. An increased risk for future heat exhaustion

ANS: A, B, C Internally high temperatures cause the appropriate hypothalamic response of profound vasodilation and profuse sweating. The individual should be encouraged to drink warm fluids to replace fluid lost through sweating. Heat exhaustion is a result of prolonged high core or environmental temperatures that are unique to each incidence. Page 500

What is a recognized treatment for the symptoms often associated with pelvic organ prolapse? (Select all that apply.) a. Pessary b. Kegel exercises c. Estrogen therapy d. Surgical repair e. Bearing down exercises

ANS: A, B, C, D A common first-line treatment is a pessary, which is a removable mechanical device that holds the uterus in position. The pelvic fascia may be strengthened through Kegel exercises (repetitive isometric tightening and relaxing of the pubococcygeal muscles) or by estrogen therapy in menopausal women. Maintaining a healthy body mass index, preventing constipation, and treating chronic cough may help as well. Surgical repair with or without a hysterectomy is the treatment of last resort. Bearing down would likely exacerbate the problem. Page 818

Which statements are true regarding chronic gastritis? (Select all that apply.) a. Chronic gastritis tends to occur in older adults. b. It causes thinning and degeneration of the stomach wall. c. Chronic gastritis results in chronic inflammation and mucosal atrophy. d. Mucosal atrophy is a common outcome of chronic gastritis. e. Epithelial metaplasia is often observed with chronic gastritis.

ANS: A, C, D, E Chronic gastritis tends to occur in older adults and causes chronic inflammation, mucosal atrophy, and epithelial metaplasia. Neither thinning nor degeneration of the stomach wall is associated with chronic gastritis. Page 1434

Which statements are true regarding the female menstrual cycle? (Select all that apply.) a. Initial cycles may dramatically vary in length. b. By adulthood, the commonly accepted cycle average is 28 (27 to 30) days. c. The length of a cycle varies among women. d. Up to 8 years before menopause, the intervals of the menstrual cycle begin to lengthen. e. Menopause is achieved when a woman is without a period for 2 years.

ANS: A, B, C, D At first, cycles are anovulatory and may vary in length from 10 to 60 days or longer. As adolescence proceeds into adulthood, regular patterns of menstruation and ovulation are established at intervals ranging from 25 to 35 days. The length of the menstrual cycle varies considerably among women. The commonly accepted cycle average is 28 (27 to 30) days, with rhythmic intervals of 21 to 35 days considered normal. Approximately 2 to 8 years before menopause, cycles begin to lengthen again. Menopause is defined as the cessation of menstrual flow for 1 year. Page 778

Which statements are true regarding the hepatitis B infection? (Select all that apply.) a. Hepatitis B poses a significant risk for chronic liver disease. b. Hepatocellular cancer is a common comorbid condition. c. Universal vaccination of infants and children is recommended. d. The preventive method of choice is immunization. e. Universal vaccination of sexually active adults is recommended.

ANS: A, B, C, D Hepatitis B infection poses significant health risks including chronic liver disease and hepatocellular cancer. Immunization against hepatitis B is the most effective means of preventing transmission. Universal vaccination of infants and children is recommended, as well as vaccination of high-risk adults. Page 941

Which factors contribute to sensorineural hearing loss? (Select all that apply.) a. Ménière disease b. Aging c. Diabetes mellitus d. Noise exposure e. Outer ear trauma

ANS: A, B, C, D Impairment of the organ of Corti or its central connections causes a sensorineural hearing loss. The hearing loss may be gradual or sudden. Conditions that commonly cause sensorineural hearing loss include congenital and hereditary factors, noise exposure, aging, Ménière disease, ototoxicity, and systemic disease (e.g., syphilis, Paget disease, collagen diseases, diabetes mellitus). Outer ear trauma is not a typical cause of sensorineural hearing loss. Page 517

Significant numbers of individuals with depression have problems related to: (Select all that apply.) a. Sleep cycles b. Weight stabilization c. Eating patterns d. Thyroid function e. Cognitive skills

ANS: A, B, C, D Insomnia, loss of appetite and body weight, and reduced interest in pleasurable activities and interpersonal relationships frequently accompany depression. Approximately 20% to 30% of persons with unipolar depression have an altered hypothalamic-pituitary-thyroid (HPT) system. Impaired cognitive skills are not necessarily associated with depression. Pages 648 | Pages 650-651

Which renal disorders are considered causes of intrarenal renal failure? (Select all that apply.) a. Acute glomerulonephritis b. Allograft rejection c. Tumors d. Acute tubular necrosis (ATN) e. Prostatic hypertrophy

ANS: A, B, C, D Intrarenal (intrinsic) acute kidney injury (AKI) may result from ischemic ATN, nephrotoxic ATN, acute glomerulonephritis, vascular disease, allograft rejection, or interstitial disease (drug allergy, infection, tumor growth). Prostatic hypertrophy is not associated with intrarenal renal failure. Pages 1360-1361

A Schwann cell: (Select all that apply.) a. Can form the myelin sheath. b. Is also referred to as a neurolemmocyte. c. Affects the function of the nodes of Ranvier. d. Is located in the peripheral nervous system. e. Is responsible of decreasing conduction velocity.

ANS: A, B, C, D The Schwann cell, or neurolemmocyte, is a glial cell that wraps around and covers axons in the peripheral nervous system. Schwann cells form and maintain the myelin sheath, and the nodes of Ranvier form the spaces on either side of the Schwann cell. If the myelin layer is tightly wrapped many times around the axon, forming nodes of Ranvier, then it increases conduction velocity and the neuron is referred to as myelinated. Page 450

What are the common modes of transmission for the hepatitis B virus (HBV)? (Select all that apply.) a. Needle punctures b. Blood transfusions c. Contact with infected body fluids d. Skin cuts e. Ingestion of infected substances

ANS: A, B, C, D Transmission of HBV can occur through needle puncture, blood transfusion, cuts in the skin, and contact with infected body fluids. Ingestion is not a recognized transmission mode. Page 941

Which clinical manifestations are consistent with cancer of the cecum and ascending colon? (Select all that apply.) a. Mahogany-colored blood mixed with stool b. Anemia c. Pain d. Constipation e. Palpable mass in the lower right quadrant

ANS: A, B, C, E Clinical manifestations consistent with cancer of the cecum and ascending colon include pain, a palpable mass in the lower right quadrant, anemia, and dark red or mahogany-colored blood mixed with the stool. Constipation is not associated with this diagnosis. Page 1471

Which statements are true regarding urethritis? (Select all that apply.) a. A purulent drainage may be present. b. A clear mucus-like discharge may be present. c. Symptoms include urethral tingling and itching or burning on urination. d. A 24-hour urine test is required to diagnose the disorder. e. Treatment includes appropriate antibiotic therapy.

ANS: A, B, C, E Symptoms of urethritis include urethral tingling and itching or a burning sensation on urination (dysuria), frequency, and urgency. The individual may note a purulent or clear mucus-like discharge from the urethra. Nucleic acid detection amplification tests allow easy detection of Neisseria gonorrhoeae and Chlamydia trachomatis in first-void urine. Treatment consists of appropriate antibiotic therapy for infectious urethritis and an avoidance of future chemical or mechanical irritation. Page 888

Which are clinical manifestations of male breast cancer? (Select all that apply.) a. Ulceration present on the breast b. Retraction of breast tissue c. Nipple discharge d. Palpable mass midline of the nipple e. Unilateral solid mass

ANS: A, B, C, E The malignant male breast lesion is usually a unilateral solid mass located near the nipple. Because the nipple is commonly involved, crusting and nipple discharge are typical clinical manifestations. Other findings include skin retraction, ulceration of the skin over the tumor, and axillary node involvement. Page 914

Which structures are parts of the nephron? (Select all that apply.) a. Loop of Henle b. Renal corpuscle c. Proximal convoluted tubule d. Calyx e. Collecting duct

ANS: A, B, C, E The nephron is a tubular structure with subunits that include the renal corpuscle, proximal convoluted tubule, loop of Henle, distal convoluted tubule, and collecting duct, all of which contribute to the formation of final urine. The calyx is not a structure contained in the nephrons. Page 1320

Which statements about the human papillolmavirus (HPV) and vaccine are true? (Select all that apply.) a. Currently, two HPV vaccines have been approved for use in the United States. b. HPV is believed to be responsible for the majority of the diagnosed cases of cervical cancer. c. A form of the vaccine has been approved for use in males to prevent genital warts. d. The administration of the vaccine is a one-dose intramuscular injection. e. The recommended age for vaccination of girls is between 11 and 12 years of age.

ANS: A, B, C, E Two HPV vaccines are currently approved in the United States: (1) quadrivalent HPV recombinant vaccine and (2) bivalent HPV recombinant vaccine. HPV is responsible for 99.7% of cervical cancer cases, 40% of penile cancers, and an estimated 5% of all cancers worldwide. The vaccine is administered by intramuscular injection, and the recommended schedule is a three-dose series with the second and third doses administered 2 and 6 months after the first dose. The recommended age for vaccination of girls is 11 to 12 years. The vaccine can be administered to girls as young as 9 years of age. The quadrivalent vaccine has been approved for males 9 to 26 years of age to prevent genital warts. Page 775 | What's New box

Estrogen has many biological effects on the female body including: (Select all that apply.) a. Maturation of reproductive organs b. Differentiating female physical characteristics c. Postpuberty closure of short bones d. Regulation of the menstrual cycle e. Endometrial regeneration after menstruation

ANS: A, B, D, E Estrogen has numerous biologic effects, many of which involve interactions with other hormones. Estrogen is needed for the maturation of the reproductive organs, development of secondary sex characteristics (differentiating male and female physical characteristics that are not directly related to reproduction), closure of long bones after the pubertal growth spurt, regulation of the menstrual cycle, and endometrial regeneration after menstruation. Page 776

It is true that a fever: (Select all that apply.) a. Is a complex cascade involving several different systems. b. Can be a result of a dysfunctional hypothalamus. c. Should be eliminated as quickly as possible. d. Triggers endocrine responses. e. Is in response to a pyrogen.

ANS: A, B, D, E Fever is a complex, integrated cascade of behavioral, neurologic, and endocrine responses to an immune challenge initiated by endogenous pyrogens or disorders of the hypothalamus. Fever production aids responses to infectious processes through several mechanisms and should be interrupted only when it might present an additional risk to the individual. Pages 498-500

The aging process brings about what changes to human cells? (Select all that apply.) a. Increased neurofibrillary tangles b. Imbalance of neurotransmitters c. Increased neuron production d. Decreased myelin presence e. Altered dendrite structure

ANS: A, B, D, E Principal cellular changes associated with aging include a decrease in the number of neurons, decreased myelin, decreased number of dendritic processes and synaptic connections, intracellular neurofibrillary tangles, and an imbalance in the amount and distribution of neurotransmitters. The aging process does not bring about an increase in neuron production. Page 478 | Box 15-5

In Parkinson disease the basal ganglia influence the hypothalamic function to produce which clinical manifestations? (Select all that apply.) a. Inappropriate diaphoresis b. Gastric retention c. Vomiting d. Diarrhea e. Urinary retention

ANS: A, B, E The basal ganglia influence hypothalamic function (autonomic and neuroendocrine) through pathways connecting the hypothalamus with the basal ganglia and cerebral cortex. Common autonomic symptoms in Parkinson disease include inappropriate diaphoresis, gastric retention, constipation, and urinary retention. Neither vomiting nor diarrhea would be clinical manifestation observed under these circumstances. Pages 567-568

Which are the early (prodromal) clinical manifestations of hepatitis? (Select all that apply.) a. Fatigue b. Vomiting c. Itching d. Splenomegaly e. Hyperalgia

ANS: A, B, E The prodromal (preicteric) phase of hepatitis begins approximately 2 weeks after exposure and ends with the appearance of jaundice. Fatigue, anorexia, malaise, nausea, vomiting, headache, hyperalgia, cough, and low-grade fever are prodromal symptoms that precede the onset of jaundice. Itching and splenomegaly are not associated with the prodromal phase of hepatitis. Page 1459

Which statement is correct regarding the functions of the pancreas? (Select all that apply.) a. Cholecystokinin stimulates the release of pancreatic enzymes. b. Bilirubin and S cells inhibit the secretion of pancreatic enzymes. c. Pancreatic polypeptide is released after eating. d. Acetylcholine (ACh) is liberated from the pancreatic branches of the vagus nerve. e. ACh stimulates the release of pancreatic enzymes.

ANS: A, C, D Enzymatic secretion follows, stimulated by cholecystokinin, which activates ACh from the vagus nerve and the release of ACh from pancreatic stellate cells. Cholecystokinin is released in the duodenum in response to the essential amino and fatty acids already present in chyme. Once in the small intestine, activated pancreatic enzymes inhibit the release of more cholecystokinin and ACh. This feedback mechanism inhibits the secretion of more pancreatic enzymes. Pancreatic polypeptide is released after eating and inhibits postprandial pancreatic exocrine secretion. (Table 40-1 summarizes the hormonal stimulation of pancreatic secretions.) Page 1415

Which statements are true regarding the parasitic infection referred to as scabies? (Select all that apply.) a. Scabies is spread through skin-to-skin contact. b. The crab lice, Phthirus pubis, cause scabies. c. Severe pruritus is its major clinical manifestation. d. Symptoms worsen at night. e. Treatment is provided through oral medication therapy.

ANS: A, C, D Scabies is a common parasitic infection that can be spread by skin-to-skin contact and sexual contact. The scabies mite burrows through the skin, depositing two or three large eggs per day. Intense pruritus, especially at night, is the most pronounced clinical manifestation. Treatment consists of topical application of a pediculicide. Pediculosis pubis (crabs) is commonly transmitted sexually and is caused by the crab louse, P. pubis. Pages 938-939

Which statements are true concerning struvite stones? (Select all that apply.) a. They are more common in women than in men. b. Struvite stones are associated with chronic laxative use in women. c. They grow large and branch into a staghorn configuration in renal pelvis and calyces. d. Struvite stones are closely associated with urinary tract infections caused by urease-producing bacteria, such as Pseudomonas. e. They are more common in men than in women.

ANS: A, C, D Women are at greater risk for developing struvite stones, but the risk is greater because women have an increased incidence of urinary tract infections not because of chronic laxative use. Such stones grow large and branch into a staghorn configuration and are associated with Pseudomonas. Pages 1343-1344

Which statement is true regarding the blood-brain barrier (BBB)? (Select all that apply.) a. The BBB is dependent on astrocytes. b. It uses the meningeal layers of the brain. c. It restricts the flow of large molecules, such as potassium. d. Naturally occurring inflammatory mediators affect the BBB. e. The BBB appears to play a role in slowing down the onset of degenerative brain disease.

ANS: A, C, D, E The BBB is a term used to describe cellular structures that selectively inhibit certain substances in the blood from entering the interstitial spaces of the brain or CSF. This term emphasizes the impermeability of the nervous system to large and potentially harmful molecules. Astrocytes wrap their foot processes around the epithelial cells of brain capillaries, thereby contributing to the formation of the BBB. Tight junctions between capillary endothelial cells form a barrier that regulates the passage of ions (e.g., sodium, potassium) that could interfere with nerve transmission, prevent toxins from entering the brain, and promote transport of nutrients and the removal of metabolites. Hormones, neurotransmitters, and inflammatory mediators can affect BBB permeability. Inhibiting these endogenous chemicals with drug therapy may reduce brain edema and slow the onset of degenerative brain diseases. The statement regarding the meningeal layers of the brain is incorrect. Page 469 | Box 15-4

The clinical manifestations of Parkinson disease include: (Select all that apply.) a. Fragmented sleep b. Drooping eyelids c. Depression d. Muscle stiffness e. Bradykinesia

ANS: A, C, D, E The classic motor manifestations of Parkinson disease (PD) are bradykinesia, tremor at rest (resting tremor), rigidity (muscle stiffness), and postural abnormalities. Nonmotor symptoms associated with PD include hyponosmia, fatigue, pain, autonomic dysfunction, sleep fragmentation, depression, and dementia with or without psychosis. Drooping eyelids are not characteristics of PD. Page 566

Gastric emptying is delayed by the presence of which substances? (Select all that apply.) a. Solids b. Carbohydrates c. Nonisotonic solutions d. Bacteria e. Fats

ANS: A, C, E Of the available options, only solids, fats, and nonisotonic solutions delay gastric emptying. Page 1397

Which statements are true regarding renal circulation? (Select all that apply.) a. The interlobar arteries travel down into the renal columns. b. The arcuate arteries branch to form the interlobar arteries. c. The arcuate arteries arch over the base of the pyramids. d. The interlobar arteries run parallel to the surface of the kidneys. e. The interlobar arteries run between the pyramids.

ANS: A, C, E The interlobar arteries are further subdivisions that travel down the renal columns and between the pyramids. At the cortical medullary junction, interlobar arteries branch into the arcuate arteries that arch over the base of the pyramids and run parallel to the surface of the kidney. Page 1325

Which hormones are natural appetite suppressants? (Select all that apply.) a. Insulin b. Cortisol c. Galanin d. Calcitonin e. Serotonin

ANS: A, D, E Insulin, calcitonin, and serotonin are natural appetite suppressants, whereas cortisol and galanin are natural appetite stimulants. Page 1448 | Box 41-4

A complex partial seizure is described as: a. Alternating of tonic and clonic movements b. Impairment of both consciousness and the ability to react to exogenous stimuli c. Focal motor movement without loss of consciousness d. One seizure followed by another in less than 1 minute

ANS: B A complex partial seizure results is impaired consciousness, as well as the inability to respond to exogenous stimuli. None of the other options accurately describe a complex partial seizure. Page 552

A clinical manifestation caused by damage to the lower pons includes an abnormal: a. Flexion with or without extensor response of the lower extremities b. Extension response of the upper and lower extremities c. Extension response of the upper extremities and flexion response of the lower extremities d. Flaccid response in the upper and lower extremities

ANS: B A flaccid state with little or no motor response to stimuli is characteristic of damage to the pons. None of the other responses are considered a clinical manifestation of damage to the lower pons. Page 534 | Table 17-6

Which assessment finding marks the end of spinal shock? a. Return of blood pressure and heart rate to normal b. Gradual return of spinal reflexes c. Return of bowel and bladder function d. Evidence of diminished deep tendon reflexes and flaccid paralysis

ANS: B A gradual return of spinal reflexes marks the end of spinal shock. The other options are not an indication of the cessation of spinal shock. Pages 570-571

A sudden, explosive, disorderly discharge of cerebral neurons is termed: a. Reflex b.Seizure c.Epilepsy d.Convulsion

ANS: B A sudden, explosive, disorderly discharge of cerebral neurons describes a seizure. This description is not accurate for the other options. Page 550

Characteristics of primary motor neuron atrophy include: a. Loss of sensation in distal, proximal, or midline muscles b. Fasciculations and muscle cramps c. Flaccid paralysis with paresthesia d. Spastic paralysis with increased deep tendon reflexes

ANS: B Fasciculations are particularly associated with primary motor neuron injury, and muscle cramps are common. The other options do not describe characteristics of primary motor neuron atrophy Pages 571-572

How does progressive nephrons injury affect angiotensin II activity? a. Angiotensin II activity is decreased. b. It is elevated. c. Angiotensin II activity is totally suppressed. d. It is not affected.

ANS: B Angiotensin II activity is elevated with progressive nephron injury. This selection is the only accurate identification of the effect of progressive nephron injury on angiotensin II activity. Page 1364 | Page 1366

Since his cerebrovascular accident, a man has been denying his left hemiplegia. What term is used to describe this finding? a. Visual agnosia b. Anosognosia c. Amusia agnosia d. Agraphia agnosia

ANS: B Anosognosia is ignorance or denial of the existence of disease. None of the remaining options describes such denial. Page 542 | Table 17-9

What term describes the loss of the comprehension or production of language? a. Agnosia b. Aphasia c. Akinesia d. Dysphasia

ANS: B Aphasia is the loss of the comprehension or production of language. The remaining options are not terms used to describe this loss of function. Page 539

In children, most intracranial tumors are located: a. Infratentorially b. Supratentiorially c. Laterally d. Posterolaterally

ANS: B Approximately 70% of all intracranial tumors in children are located infratentorially (below the tentorium cerebelli) and not in the locations provided by the other options. Page 626

Which cerebral vascular hemorrhage causes meningeal irritation, photophobia, and positive Kernig and Brudzinski signs? a. Intracranial b. Subarachnoid c. Epidural d. Subdural

ANS: B Assessment findings related to only a subarachnoid hemorrhage include meningeal irritation and inflammation, causing neck stiffness (nuchal rigidity), photophobia, blurred vision, irritability, restlessness, and low-grade fever. A positive Kernig sign, in which straightening the knee with the hip and knee in a flexed position produces pain in the back and neck regions, and a positive Brudzinski sign, in which passive flexion of the neck produces neck pain and increased rigidity, may appear. Pages 605-606

What is the direct action of atrial natriuretic hormone? a. Sodium retention b. Sodium excretion c. Water retention d. Water excretion

ANS: B Atrial natriuretic peptide (ANP) and brain natriuretic peptide (BNP) inhibit the secretion of renin, inhibit angiotensin-induced secretion of aldosterone, vasodilate the afferent and constrict the efferent glomerular arterioles, and inhibit sodium and water absorption by kidney tubules. The other actions are not a result of the atrial natriuretic hormone. Page 1327

What term is used to identify an inflammation of the glans penis? a. Glanitis b. Balanitis c. Priapism d. Hydrocelitis

ANS: B Balanitis is the only term used to identify an inflammation of the glans penis (see Figure 25-4). Page 890

Considering the innervation of the circular muscles of the bladder neck, which classification of drug is used to treat bladder neck obstruction? a. -Adrenergic blocking medications b. -Adrenergic blocking medications c. Parasympathomimetic medications d. Anticholinesterase medications

ANS: B Because the bladder neck consists of circular smooth muscle with adrenergic innervation, detrusor sphincter dyssynergia may be managed by -adrenergic blocking (antimuscarinic) medications. This selection is the only option capable of this specific function. Page 1346

What happens to the vagina's lining at puberty? a. It becomes thinner. b. It becomes thicker. c. It assumes a neutral pH. d. It undergoes atrophy.

ANS: B Before puberty, vaginal pH is approximately 7 (neutral) and the vaginal epithelium is thin. At puberty, the pH becomes more acidic (4 to 5) and the squamous epithelial lining thickens. Cell atrophy is not associated with puberty. Page 773

The initial reproductive structures of the male and female embryos appear the same until which week of gestation? a. Third b. Eighth c. Twentieth d. Thirtieth

ANS: B Between 6 and 7 weeks' gestation, the male embryo differentiates under the influence of testes-determining factor (TDF). In the absence of testosterone, a loss of the wolffian system occurs and the two gonads develop into ovaries at 6 to 8 weeks' gestation. Page 769

During which stage of syphilis do bloodborne bacteria spread to all the major organ systems? a. Primary b. Secondary c. Latent d. Tertiary

ANS: B Bloodborne bacteria spread to all major organ systems during only stage II, secondary syphilis (see Box 26-2). Page 923

What effects do exercise and body position have on renal blood flow? a. Exercise and body position activate renal parasympathetic neurons and cause mild vasoconstriction. b. They activate renal sympathetic neurons and cause mild vasoconstriction. c. Both activate renal parasympathetic neurons and cause mild vasodilation. d. They activate renal sympathetic neurons and cause mild vasodilation.

ANS: B Exercise and change of body position activate renal sympathetic neurons and cause mild vasoconstriction. The other options do not have these effects on renal blood flow. Page 1327

Which statement is false concerning the skeletal alterations caused by chronic renal failure when the glomerular filtration rate (GFR) declines to 25% of normal? a. Parathyroid hormone is no longer effective in maintaining serum phosphate levels. b. The parathyroid gland is no longer able to secrete sufficient parathyroid hormone. c. The synthesis of 1,25-vitamin D3, which reduces intestinal absorption of calcium, is impaired. d. The synthesis of 1,25-vitamin D3, which impairs the effectiveness of calcium and phosphate resorption from bone by parathyroid hormone, is impaired.

ANS: B Bone and skeletal changes develop with alterations in calcium and phosphate metabolism (see Table 38-16). These changes begin when the GFR decreases to 25% or less. The combined effect of hyperparathyroidism and vitamin D deficiency can result in renal osteodystrophies (e.g., osteomalacia, osteitis fibrosa with increased risk for fractures). Other consequences of secondary hyperparathyroidism include soft-tissue and vascular calcification, cardiovascular disease, and, less commonly, calcific uremic arteriolopathy. The other options are true. Page 1368 | Table 38-16

What is the reason breast cancer in men has such a poor prognosis? a. Breast cancer is extremely aggressive in men. b. Treatment is usually delayed as a result of late detection. c. Chemotherapies are not as effective in men. d. Breast tumors tend to be small and hard to isolate.

ANS: B Breast cancer is relatively uncommon in men, but it has a poor prognosis because men tend to delay seeking treatment until the disease is advanced. This selection is the only option that accurately identifies the reason breast cancer in men has a poor prognosis. Page 914

The region responsible for the motor aspects of speech is located in the: a. Wernicke area in the temporal lobe b. Broca area in the frontal lobe c. Wronka area in the parietal lobe d. Barlow area in the occipital lobe

ANS: B Broca speech area is the only region responsible for the motor aspects of speech. Page 457

Cerebral edema is an increase in the fluid content of the brain's: a. Ventricles b. Tissue c. Neurons d. Meninges

ANS: B Cerebral edema is an increase in the fluid content of brain tissue; that is, a net accumulation of water within the brain. Cerebral edema is not noted in the brain's ventricles, neurons, or meninges. Page 557

Cheyne-Stokes respirations are described as a: a. Sustained deep rapid but regular pattern of breathing b. Crescendo-decrescendo pattern of breathing, followed by a period of apnea c. Prolonged inspiratory period, gradually followed by a short expiratory period d. Completely irregular breathing pattern with random shallow, deep breaths and irregular pauses

ANS: B Cheyne-Stokes respiration is an abnormal rhythm of breathing (periodic breathing) that alternates between hyperventilation and apnea. Cheyne-Stokes respirations do not include a sustained deep respiratory rate. Altered inspiratory and expiratory periods are not characteristic of Cheyne-Stokes respirations. Random, irregular breathing patterns are not observed during Cheyne-Stokes respirations. Pages 529-530 | Page 532 | Table 17-4

What type of posturing exists when a person with a severe closed head injury has all four extremities in rigid extension with the forearms in hyperpronation and the legs in plantar extension? a. Decorticate b. Decerebrate c. Spastic d. Cerebellar

ANS: B Decerebrate posturing includes opisthotonos (hyperextension of the vertebral column) with clenching of the teeth; extension, abduction, and hyperpronation of the arms; and extension of the lower extremities including plantar extension. The other options do not describe such posturing. Page 534 | Page 575 | Table 17-6

What is the cause of reflux esophagitis? a. Immune response to gastroesophageal reflux b. Delayed gastric emptying c. Congenital anomaly d. Secretory response to gastroesophageal reflux

ANS: B Delayed gastric emptying contributes to reflux esophagitis by (1) lengthening the period during which reflux is possible and (2) increasing the acid content of chyme. None of the other options are accurate descriptions of the cause of reflux esophagitis. Page 1429

Considering host defense mechanisms, which element in the urine is bacteriostatic? a. High pH (alkaline urine) b. High urea c. High glucose d. High calcium

ANS: B Dilute urine washes out bacteria, and urine with higher urea concentrations (high osmolarity) is more bacteriostatic. Page 1350

Which statement is false regarding the contributing factors of duodenal ulcers? a. Bleeding from duodenal ulcers causes hematemesis or melena. b. Gastric emptying is slowed, causing greater exposure of the mucosa to acid. c. The characteristic pain begins 30 minutes to 2 hours after eating when the stomach is empty. d. Duodenal ulcers occur with greater frequency than other types of peptic ulcers.

ANS: B Duodenal ulcers can be associated with altered mucosal defenses, rapid gastric emptying, elevated serum gastrin levels, or acid production stimulated by smoking. The other options provide correct information regarding duodenal ulcers. Page 1435

Which disorder has clinical manifestations that include decreased consciousness for up to 6 hours, as well as retrograde and posttraumatic amnesia? a. Mild concussion b. Classic concussion c. Cortical contusion d. Acute subdural hematoma

ANS: B Evidence of a classic concussion is the immediate loss of consciousness, which lasts less than 6 hours. Retrograde and anterograde (posttraumatic) amnesia is also present. The other options do not apply. Page 588

Hypothalamic-pituitary-adrenal (HPA) system abnormalities exist in a large percentage of individuals with: a. Schizophrenia b. Major depression c. Mania d. Panic disorder

ANS: B Excessive activation of the HPA system resulting in elevated glucocorticoid secretion is found in a large percentage (30% to 70%) of people with major depression, suggesting that mechanisms responsible for HPA hormone alterations contribute to the pathophysiologic condition of depression. HPA system abnormalities do not necessarily exit in individuals represented by the remaining options. Page 648

What type of seizure starts in the fingers and progressively spreads up the arm and extends to the leg? a. Complex-psychomotor seizure b. Focal (partial) Jacksonian seizure c. Generalized seizures d. Atonic-drop seizure

ANS: B Focal (partial) Jacksonian seizures most often begin in the face and fingers and then progressively spread to other body parts. The other options do not begin and spread in the fashion described. Page 551 | Table 17-16

A criterion for a diagnosis of generalized anxiety disorder (GAD) is a period of excessive worrying that lasts for at least how many months? a. 3 b. 6 c. 9 d. 12

ANS: B GAD is diagnosed when an individual spends at least 6 months worrying excessively and exhibits at least three of the six symptoms. Although 3 months is not sufficient time, the remaining options are excessive. Page 655

What term is used to identify frank bleeding of the rectum? a. Melena b. Hematochezia c. Occult bleeding d. Hematemesis

ANS: B Hematochezia is the only available option that is associated with frank bright red or burgundy blood from the rectum. Page 1428

Hypercalciuria is primarily attributable to which alteration? a. Defective renal calcium reabsorption b. Intestinal hyperabsorption of dietary calcium c. Bone demineralization caused by prolonged immobilization d. Hyperparathyroidism

ANS: B Hypercalciuria is usually attributable to intestinal hyperabsorption of dietary calcium and less commonly to a defect in renal calcium reabsorption. Hyperparathyroidism and bone demineralization associated with prolonged immobilization are also known to cause hypercalciuria but too a much lesser degree. Page 1343

Stage 1 intracranial hypertension is caused by the: a. Loss of autoregulation that normally maintains constant blood flow during changes in cerebral perfusion pressure b. Displacement of cerebrospinal fluid, followed by compression of the cerebral venous system c. Vasoconstriction of the cerebral arterial system with reciprocal increase in systemic blood pressure d. Compression of the medulla oblongata in the brainstem by herniation of the cerebral cortex

ANS: B If intracranial pressure remains high after cerebrospinal fluid (CSF) displacement out of the cranial vault, then cerebral blood volume is altered, resulting in stage 1 intracranial hypertension. Vasoconstriction and external compression of the venous system occur in an attempt to further decrease the intracranial pressure. None of the remaining options accurately describe the cause of stage 1 intracranial hypertension. Page 556

Which type of axon transmits a nerve impulse at the highest rate? a. Large nonmyelinated b. Large myelinated c. Small nonmyelinated d. Small myelinated

ANS: B If the myelin layer is tightly wrapped many times around the axon and is forming the nodes of Ranvier, then conduction velocity increases and the neuron is referred to as myelinated. The increased diameter of the myelinated axons allows for the transmission of impulses at a faster rate. The other options do not affect nervous impulse transmission rates. Pages 448-450

An intestinal obstruction at the pylorus or high in the small intestine causes metabolic alkalosis by causing which outcome? a. Gain of bicarbonate from pancreatic secretions that cannot be absorbed b. Excessive loss of hydrogen ions normally absorbed from gastric juices c. Excessive loss of potassium, promoting atony of the intestinal wall d. Loss of bile acid secretions that cannot be absorbed

ANS: B If the obstruction is at the pylorus or high in the small intestine, then metabolic alkalosis initially develops as a result of excessive loss of hydrogen ions that normally would be reabsorbed from the gastric juices. This selection is the only option that accurately describes the cause of metabolic alkalosis in this situation. Page 1432

What is an effect of the sympathetic nervous system? a. Stimulation of peristalsis b. Increased blood sugar levels c. Reduction in heart rate d. Pupillary constriction

ANS: B In general, sympathetic stimulation promotes responses that are concerned with the protection of the individual, which include increased blood sugar levels, temperature, and blood pressure. The responses noted in the remaining options are not a result of sympathetic stimulation. Page 476

Which statement is true regarding the major difference between male and female sex hormone production? a. Luteinizing hormone has no apparent action in a man. b. In a man, sex hormone production is relatively constant. c. Estradiol is not produced in a man. d. In a man, gonadotropin-releasing hormone does not cause the release of follicle stimulating hormone.

ANS: B In men, sex hormone production is relatively constant with some diurnal variation. The other options are not true statements. Page 789

Which would be considered a positive symptom of schizophrenia? a. Blunted affect b. Auditory hallucinations c. Poverty of speech d. Lack of social interaction

ANS: B Positive symptoms frequently occur during a psychotic episode, when an individual loses touch with reality and experiences something that should be absent (e.g., hallucinations). The remaining options are classified as negative symptoms. Pages 644-646

The edema of the upper cervical cord after spinal cord injury is considered life threatening because of which possible outcome? a. Hypovolemic shock from blood lost during the injury b. Breathing difficulties from an impairment to the diaphragm c. Head injury that likely occurred during the injury d. Spinal shock immediately after the injury

ANS: B In the cervical region, spinal cord swelling may be life threatening because of the possibility of resulting impairment of the diaphragm function (phrenic nerves exit C3-C5). The other options do not appropriately explain the threat. Page 591

Which neurotransmitter is reduced in people with schizophrenia? a. Dopamine b. Gamma-aminobutyric acid c. Acetylcholine d. Serotonin

ANS: B In the dorsal prefrontal cortex of schizophrenic brains, glutamic acid decarboxylase, the major enzyme in gamma-aminobutyric acid GABA biosynthesis, is diminished, which likely impairs synaptic performance and cognitive and behavioral functions associated with this brain region. The other neurotransmitters mentioned in the option choices are not related to the presentation of schizophrenia. Page 643

The process of conjugation of bilirubin in the liver is best described as which transformation? a. Unconjugated (fat-soluble) bilirubin into urobilinogen b. Unconjugated (fat-soluble) bilirubin into conjugated (water-soluble) bilirubin c. Conjugated (water-soluble) bilirubin into unconjugated (fat-soluble) bilirubin d. Conjugated (water-soluble) bilirubin into urobilinogen

ANS: B In the liver, unconjugated bilirubin moves from plasma in the sinusoids into the hepatocytes. Within hepatocytes it joins with glucuronic acid to form conjugated bilirubin, which is water-soluble. This selection is the only option that accurately describes the transformation associated with the conjugation of bilirubin. Pages 1411-1412

What indicates that spinal shock is terminating? a. Voluntary movement below the level of injury b. Reflex emptying of the bladder c. Paresthesia below the level of injury d. Decreased deep tendon reflexes and flaccid paralysis

ANS: B Indications that spinal shock is terminating include the reappearance of reflex activity, hyperreflexia, spasticity, and reflex emptying of the bladder. Termination of a spinal cord injury is not evidenced by any of the other options. Page 592

Infertility is defined as the inability to conceive after how many months of unprotected intercourse with the same partner? a. 6 b. 12 c. 18 d. 24

ANS: B Infertility is defined as the inability to conceive after 1 year of unprotected intercourse with the same partner. Page 835

Which edema is most often observed with noncommunicating hydrocephalus? a. Metabolic b. Interstitial c. Vasogenic d. Ischemic

ANS: B Interstitial edema is observed most often with noncommunicating hydrocephalus. Noncommunicating hydrocephalus is not the cause of any of the other options. Page 558

What theory is used to describe the cause of endometriosis? a. Obstruction within the fallopian tubes prevents the endometrial tissue from adhering to the lining of the uterus. b. Endometrial tissue passes through the fallopian tubes and into the peritoneal cavity and remains responsive to hormones. c. Inflammation of the endometrial tissue develops after recurrent sexually transmitted diseases. d. Endometrial tissue lies dormant in the uterus until the ovaries produce sufficient hormone to stimulate its growth.

ANS: B It has been proposed that endometriosis is caused by the implantation of endometrial cells during retrograde menstruation, during which menstrual fluids move through the fallopian tubes and empty into the pelvic cavity (see Figure 24-16). Similar to normal endometrial tissue, the ectopic (out of place) endometrium responds to the hormonal fluctuations of the menstrual cycle. Of the available options, this answer is the only accepted theory for the cause of endometriosis. Page 823

Kidney stones in the upper part of the ureter would produce pain referred to which anatomical area? a. Vulva or penis b. Umbilicus c. Thighs d. Lower abdomen

ANS: B Kidney stones in the upper part of the ureter would produce pain in the umbilicus. Sensory innervation for the upper part of the ureter arises from the tenth thoracic nerve roots with referred pain to the umbilicus. The other options would not experience such referred pain. Page 1325

Which electrolyte imbalance contributes to lithium toxicity? a. Hypernatremia b. Hyponatremia c. Hyperkalemia d. Hypokalemia

ANS: B Lithium toxicity can be a result of hyponatremia. Lithium is normally removed from the kidneys; however, when the body is sodium depleted, the kidneys reabsorb sodium along with lithium. Lithium toxicity does not result from any of the remaining options. Pages 652-653

How can glaucoma cause blindness? a. Infection of the cornea b. Pressure on the optic nerve c. Opacity of the lens d. Obstruction of the venous return from the retina

ANS: B Loss of visual acuity as a result of pressure on the optic nerve is the only reason glaucoma can result in blindness. Pages 510-511

A surge of which hormone causes the corpus luteum to produce progesterone? a. Follicle stimulating hormone b. Luteinizing hormone c. Gonadotropin-releasing hormone d. Estrogen

ANS: B Luteinizing hormone from the anterior pituitary stimulates the corpus luteum to secrete progesterone, the second major female sex hormone. Page 778

Individuals who eat aged cheese and avocado when taking monoamine oxidase inhibitors (MAOIs) may experience: a. Kidney damage b. Hypertensive crisis c. Orthostatic hypotension d. Weight gain

ANS: B MAOIs also may induce acute and heightened elevations in blood pressure (e.g., hypertensive crisis) after the intake of tyramine-rich foods, such as aged cheeses, sour cream, pods of broad beans, pickled herring, liver, canned figs, raisins, and avocados. The ingestion of tyramine-rich foods is not a trigger for the remaining options. Page 651

What directly causes ovulation during the menstrual cycle? a. Gradual decrease in estrogen levels b. Sudden increase of LH c. Sharp rise in progesterone levels d. Gradual increase in estrogen levels

ANS: B Menstrual cyclicity and regular ovulation are dependent on (1) the activity of the gonadostat (GnRH pulse generator); (2) the pituitary secretion of gonadotropins; and (3) estrogen (estradiol)-positive feedback for the preovulatory LH and FSH surges, oocyte maturation, and corpus luteum formation. Pages 778-780

Which organism is responsible for the development of syphilis? a. Neisseria syphilis b. Treponema pallidum c. Haemophilus ducreyi d. Chlamydia trachomatis

ANS: B T. pallidum is the only cause of syphilis. Page 923

Which laboratory test is considered adequate for an accurate and reliable diagnosis of gonococcal urethritis in a symptomatic man? a. Ligase chain reaction (LCR) b. Gram-stain technique c. Polymerase chain reaction (PCR) d. DNA testing

ANS: B Microscopic evaluation of Gram-stained slides of clinical specimens is deemed positive for Neisseria gonorrhoeae if gram-negative diplococci with the typical "kidney bean" morphologic appearance are found inside polymorphonuclear leukocytes. Such a finding is considered adequate for the diagnosis of gonococcal urethritis in a symptomatic man. The other options are not relevant to the diagnosis of this condition. Page 922

What is the central component of the pathogenic model of multiple sclerosis? a. Myelination of nerve fibers in the peripheral nervous system (PNS) b. Demyelination of nerve fibers in the CNS c. Development of neurofibrillary tangles in the CNS d. Inherited autosomal dominant trait with high penetrance

ANS: B Multiple sclerosis (MS) is an autoimmune disorder diffusely involving the degeneration of CNS myelin and loss of axons. The other options are not central components of the pathogenic model of MS. Pages 618-619

The mutation of which gene is an early event associated with the pathogenetic origin of esophageal cancer? a. K-ras mutation b. TP53 c. myc d. HER2

ANS: B Mutation of the TP53 gene is an early event associated with esophageal cancer. This selection is the only mutation from among the provided options. Page 1466

What is the cause of gastroesophageal reflux disease? a. Excessive production of hydrochloric acid b. Zone of low pressure of the lower esophageal sphincter c. Presence of Helicobacter pylori in the esophagus d. Reverse muscular peristalsis of the esophagus

ANS: B Normally, the resting tone of the lower esophageal sphincter maintains a zone of high pressure that prevents gastroesophageal reflux. In individuals who develop reflux esophagitis, this pressure tends to be lower than normal from either transient relaxation or a weakness of the sphincter. This selection is the only option that accurately describes the cause of gastroesophageal reflux disease. Page 1429

Which structure is lined with columnar epithelial cells? a. Perimetrium b. Endocervical canal c. Myometrium d. Vagina

ANS: B Of the available options, only the endocervical canal does not have an endometrial layer; rather, the layer is lined with columnar epithelial cells. Page 775

Considering the pathophysiologic characteristics of primary amenorrhea, what anatomic structure is involved in compartment IV? a. Vagina b. Hypothalamus c. Ovary d. Anterior pituitary

ANS: B Of the options available, only compartment IV disorders include central nervous system (CNS) conditions, in particular hypothalamic disorders. Page 805

During reactivation (release from latency), herpes virus genomes are transported through which nerves to the dermal surface? a. Somatic b. Peripheral sensory c. Autonomic d. Peripheral motor

ANS: B Only during reactivation are the viral genomes transported through the peripheral sensory nerves back to the dermal surface. Page 933

What unique factor causes adolescent girls to have a high risk for sexually transmitted infections (STIs)? a. They are in an experimental phase with sexual intercourse and believe they are resistant to developing STIs. b. The adolescent cervix is immature and lacks immunity. c. The length of the vaginal canal is short in adolescents, allowing a greater concentration of microorganisms within the internal genitalia. d. In adolescent girls, the anus to the vaginal introitus are in close proximity.

ANS: B Partly, perhaps, because of risk-taking behavior (unprotected intercourse or selection of high-risk partners), many adolescents have an increased risk for STI exposure and infection. The unique factor for adolescent women is that they have a physiologically increased susceptibility to infection because of increased cervical immaturity and lack of immunity. The remaining options are not considered legitimate risk factors for STIs. Pages 918-919

When considering the risk factors for the development of phantom limb pain, the nurse recognizes which as a primary contributing factor? a. Age, with adolescent patients being at a higher risk than adults b. Presence of pain in the limb before amputation c. Patient's previous experience with managing pain d. Cultural views regarding the acceptance of pain

ANS: B Phantom limb pain is pain that an individual feels in an amputated limb after the stump has completely healed. It is more likely to appear in individuals who experienced pain in the limb before amputation. The other options would not be considered a primary contributing factors. Page 494

A decrease in receptor binding for which neurotransmitter is found in individuals with depression? a. Norepinephrine b. Serotonin c. Dopamine d. Acetylcholine

ANS: B Postmortem and/or brain imaging studies of individuals with depression reveal a widespread decrease in serotonin 5-HT1A-receptor subtype binding in frontal, temporal, and limbic cortex, as well as serotonin-transporter binding in cerebral cortex and hippocampus. A decrease in receptor binding is not observed in the other neurotransmitters. Page 650

A person who has cholera would be expected to have which type of diarrhea? a. Osmotic b. Secretory c. Small volume d. Motility

ANS: B Primary causes of secretory diarrhea are bacterial enterotoxins, particularly those released by cholera or strains of Escherichia coli, and neoplasms, such as gastrinoma or thyroid carcinoma. None of the other options are associated with secretory diarrhea. Page 1425

The release of which chemical mediator causes primary dysmenorrhea? a. Leukotrienes b. Prostaglandins c. Bradykinin d. C-reactive protein

ANS: B Primary dysmenorrhea is painful menstruation associated with the release of prostaglandins in ovulatory cycles. This option is the only answer that accurately identifies the chemical mediator associated with dysmenorrhea. Page 804

Which hormone promotes the development of testosterone in both males and females? a. Progesterone b. Prolactin c. Oxytocin d. Estrogen

ANS: B Prolactin, a polypeptide synthesized and secreted from the pituitary, helps maintain biosynthesis of testosterone. Page 791

Rapid eye movement (REM) sleep occurs in cycles approximately every: a. 45 minutes b. 90 minutes c. 120 minutes d. 150 minutes

ANS: B REM sleep accounts for 20% to 25% of sleep time and is characterized by desynchronized, low-voltage, fast activity that occurs for 5 to 60 minutes approximately every 90 minutes, beginning after 1 to 2 hours of non-REM sleep. Page 503

Posthyperventilation apnea (PHVA) ceases and rhythmic breathing is resumed when levels of arterial: a. Carbon dioxide increase b. Carbon dioxide become normal c. Oxygen increase d. Oxygen decrease

ANS: B Rhythmic breathing returns when the PCO2 level returns to normal. None of the remaining options would affect normal rhythmic breathing after PHVA. Page 530 | Table 17-4

Which pancreatic enzyme is responsible for the breakdown of carbohydrates? a. Trypsin b. Amylase c. Lipase d. Chymotrypsin

ANS: B Salivary and pancreatic amylases break down starches to oligosaccharides by splitting -1,4-glucosidic linkages of long-chain molecules. None of the other options is relevant to this process. Page 1402

Which hormone is linked to an increase in appetite during puberty? a. Inhibin b. Leptin c. Activin d. Follistatin

ANS: B Sensitivity to leptin, which regulates appetite and energy metabolism, increases during puberty; in theory, the adolescent consumes more calories to meet the caloric needs of the pubertal growth spurt. The percent of body fat and leptin levels in girls continue to increase, whereas muscle mass increases in boys. No apparent link exists between increased appetite during puberty and any of the other options. Page 778

Which statement is false about the factors that facilitate the ascent of gonococci into the uterus and fallopian tubes? a. Ascent of gonococci is facilitated because the cervical plug disintegrates during menstruation. b. Ascent of gonococci is facilitated because the vaginal pH decreases to 2 or 3. c. Ascent of gonococci is facilitated because the uterine contractions may cause retrograde menstruation into the fallopian tubes. d. Ascent of gonococci is facilitated because the bacteria may adhere to sperm and be transported to the fallopian tubes.

ANS: B Several factors can facilitate the ascent of gonococci into the uterus and the fallopian tubes, where they cause pelvic inflammatory disease (PID). Among these factors are (1) disintegration of the cervical mucous plug and (2) a rise in vaginal pH greater than 4.5 during menstruation. The other options are accurate statements. Page 921

A woman diagnosed with trichomoniasis asks if her sexual partner should be treated as well. What is the appropriate response? a. Sexual partners should be treated only if symptoms are present. b. Sexual partners should be treated even if they are asymptomatic. c. Infections in men are self-limiting; therefore a male sexual partner does not require treatment. d. Sexual partners should be treated to prevent infection.

ANS: B Sexual partners, even if asymptomatic, are also treated and examined for coexisting sexually transmitted infections. Page 938

The most likely rationale for body temperature fluctuations after cervical spinal cord injury is that the person has: a. Developed bilateral pneumonia or a urinary tract infection. b. Sustain sympathetic nervous system damage resulting in disturbed thermal control. c. Sustained a head injury that damaged the hypothalamus's ability to regulate temperature. d. Developed septicemia from posttrauma infection.

ANS: B Spinal cord injuries result in disturbed thermal control because the sympathetic nervous system is damaged. None of the remaining options explain this complication. Page 592

Status epilepticus is considered a medical emergency because of the: a. Loss of consciousness b. Development of cerebral hypoxia c. Possibility of a head injury during seizures d. Decrease in brain metabolism

ANS: B Status epilepticus is a true medical emergency because a single seizure can last longer than 30 minutes, resulting in hypoxia of the brain. The other options are not the criteria used to consider status epilepticus. Page 553

After a cerebrovascular accident, a man is unable to either feel or identify a comb with his eyes closed. This is an example of: a. Agraphia b. Tactile agnosia c. Anosognosia d. Prosopagnosia

ANS: B Tactile agnosia is the inability to recognize objects by touch. None of the other options define the inability to recognize objects by touch. Page 542 | Table 17-9

Which dyskinesia involves involuntary movements of the face, trunk, and extremities? a. Paroxysmal b. Tardive c. Hyperkinesia d. Cardive

ANS: B Tardive dyskinesia is the involuntary movement of the face, trunk, and extremities. The other terms do not describe involuntary movements of the face, trunk, and extremities. Page 562

During a synapse, what change occurs after the neurotransmitter binds to the receptor? a. The permeability of the presynaptic neuron changes; consequently, its membrane potential is changed as well. b. The permeability of the postsynaptic neuron changes; consequently, its membrane potential is changed as well. c. The postsynaptic cell prevents any change in permeability and destroys the action potential. d. The presynaptic cell synthesizes and secretes additional neurotransmitters.

ANS: B The binding of the neurotransmitter at the receptor site changes the permeability of the postsynaptic neuron and, consequently, its membrane potential. The remaining options do not accurately describe the occurrence. Pages 452-453

Blood vessels of the kidneys are innervated by the: a. Vagus nerve b. Sympathetic nervous system c. Somatic nervous system d. Parasympathetic nervous system

ANS: B The blood vessels of the kidney are innervated by the sympathetic noradrenergic fibers that cause arteriolar vasoconstriction and reduce renal blood flow. The other options are not involved in this process. Page 1326

Where are the primary-order pain transmitting neurons located within the spinal cord? a. Lateral root ganglia b. Dorsal root ganglia c. Anterior root ganglia d. Medial root ganglia

ANS: B The cell bodies of the primary-order neurons, or pain-transmitting neurons, reside only in the dorsal root ganglia just lateral to the spine along the sensory pathways that penetrate the posterior part of the cord. Page 487

Which area of the brain assumes the responsibility for conscious and unconscious muscle synergy and for maintaining balance and posture? a. Cerebrum b. Cerebellum c. Diencephalon d. Brainstem

ANS: B The cerebellum is responsible for conscious and unconscious muscle synergy and for maintaining balance and posture. This role is not assumed by any of the remaining options. Page 459

Where is the cerebrospinal fluid produced? a. Arachnoid villi b. Choroid plexuses c. Ependymal cells d. Pia mater

ANS: B The choroid plexuses are the structures that produce CSF; they arise from the pia mater. The remaining options do not have a role in CSF production. Pages 465-466

The area of the kidneys that contains the glomeruli and portions of the tubules is called the: a. Medulla b. Cortex c. Pyramids d. Columns

ANS: B The cortex contains all the glomeruli and portions of the tubules of the kidneys. Although the other options are also located in the kidney, they do not contain the glomeruli. Page 1320

Which statement best describes the gastrointestinal tract? a. The gastrointestinal tract is a muscular tube that transports food from the mouth to the stomach. b. The gastrointestinal tract is a hollow tube that extends from the mouth to the anus. c. The gastrointestinal tract is a baglike structure that propels partially digested food (chyme). d. The structure is 5 m long and consists of three segments.

ANS: B The gastrointestinal tract is a hollow tube that extends from the mouth to the anus. The esophagus is a muscular tube that transports food from the mouth to the stomach. The stomach is a baglike structure that secretes digestive juices, mixes and stores food, and propels partially digested food (chyme) into the duodenum. The small intestine is 5 m long and has three segments. Page 1395

The ileum and jejunum are suspended by folds of the peritoneum that contain an extensive vascular and nervous network. What are these folds called? a. Ligament of Treitz b. Mesentery c. Auerbach folds d. Lamina propria

ANS: B The ileum and jejunum are suspended in loose folds from the posterior abdominal wall by a peritoneal membrane called the mesentery. The mesentery facilitates intestinal motility and supports blood vessels, nerves, and lymphatics. None of the other options accurately fills this description. Page 1401

Innervation of the bladder and internal urethral sphincter is supplied by which nerves? a. Peripheral nerves b. Parasympathetic fibers c. Sympathetic nervous system d. Tenth thoracic nerve roots

ANS: B The innervation of the bladder and internal urethral sphincter is supplied by parasympathetic fibers of the autonomic nervous system. The process is not dependent on any of the other options. Page 1326

Autonomic hyperreflexia-induced bradycardia is a result of stimulation of the: a. Sympathetic nervous system to ß-adrenergic receptors to the sinoatrial node b. Carotid sinus to the vagus nerve to the sinoatrial node c. Parasympathetic nervous system to the glossopharyngeal nerve to the atrioventricular node d. Bundle branches to the -adrenergic receptors to the sinoatrial node

ANS: B The intact autonomic nervous system reflexively responds with an arteriolar spasm that increases blood pressure. Baroreceptors in the cerebral vessels, the carotid sinus, and the aorta sense the hypertension and stimulate the parasympathetic system. The heart rate decreases, but the visceral and peripheral vessels do not dilate because efferent impulses cannot pass through the cord. The process is not appropriately described by the other options. Pages 593-595 | Figure 18-13

When the right kidney is obstructed, how will the glomeruli and tubules in the left kidney compensate? a. Increase in number b. Increase in size c. Develop collateral circulation d. Increase speed of production

ANS: B These processes cause the contralateral (unobstructed) kidney to increase the size of individual glomeruli and tubules. The changes described by the other options are not correct. Page 1342

The presence of chyme in the duodenum stimulates which hormones? a. Motilin and histamine b. Secretin and cholecystokinin c. Enteroglucagon and gastric inhibitory peptide d. Somatostatin and acetylcholine

ANS: B The movement of chyme from the stomach into the duodenum initiates the intestinal phase of secretion. This phase represents a slowdown of the gastric secretory response; however, the presence of digested protein and amino acids in the duodenum continue to stimulate some gastric secretion. The release of secretin and cholecystokinin stimulate pancreatic secretions and inhibit gastric secretions. This selection is the only option that correctly identifies the hormones that are stimulated by the presence of chyme in the duodenum. Page 1400

What is the functional unit of the kidney called? a. Glomerulus b. Nephron c. Collecting duct d. Pyramid

ANS: B The nephron is the functional unit of the kidney. Although the other options are also located in the kidney, they are not its functional units. Page 1320

From which part of the midbrain do cranial nerves V to VIII emerge? a. Midbrain b. Pons c. Medulla oblongata d. Lateral colliculi

ANS: B The nuclei of cranial nerves V through VIII (see Table 15-6 for discussion) are located only in the pons. Pages 459-460

What area of the brain mediates the executive attention functions? a. Limbic b. Prefrontal c. Parietal d. Occipital

ANS: B The prefrontal areas mediate several cognitive functions, called executive attention functions (e.g., planning, problem solving, setting goals). The remaining options are not areas involved with the mediation of executive attention functions. Page 538

What name is given to a large network of neurons within the brainstem that is essential for maintaining wakefulness? a. Midbrain b. Reticular activating system c. Medulla oblongata d. Pons

ANS: B The reticular activating system is essential for maintaining wakefulness. The remaining options are not essential to this function. Page 454

The risk of which cancer is greater if the man has a history of cryptorchidism? a. Penile b. Testicular c. Prostate d. Epididymal

ANS: B The risk of testicular cancer is 35 to 50 times greater in men with cryptorchidism or in those with a history of cryptorchidism than it is for the general male population. This is not true of the other options. Pages 893-894

The ability of the eyes to track moving objects through a visual field is primarily a function of which colliculi? a. Inferior b. Superior c. Mid d. Posterior

ANS: B The superior colliculi are involved with voluntary and involuntary visual motor movements (e.g., the ability of the eyes to track moving objects in the visual field). Tracking moving objects is not the primary function of the remaining options. Page 459

What effect is a result of inhibiting the parasympathetic nervous system with a drug such as atropine? a. Salivation becomes thinner. b. Salivation decreases. c. The pH of saliva changes. d. Digestive enzymes are inhibited.

ANS: B The sympathetic and parasympathetic divisions of the autonomic nervous system control salivation. Because cholinergic parasympathetic fibers stimulate the salivary glands, atropine (an anticholinergic agent) inhibits salivation and makes the mouth dry. None of the other options is a result of drugs such as atropine. Page 1395

Which endogenous opioid is located in the hypothalamus and pituitary and is a strong -receptor agonist? a. Enkephalins b. Endorphins c. Dynorphins d. Endomorphins

ANS: B The synthesis and activity of -endorphin is concentrated in the hypothalamus and the pituitary gland and act as strong µ-receptor agonist. The remaining options do not fulfill this objective. Page 491

Which glomerular lesion is characterized by thickening of the glomerular capillary wall with immune deposition of immunoglobulin G (IgG) and C3? a. Proliferative b. Membranous c. Mesangial d. Crescentic

ANS: B The thickening of the glomerular capillary wall characterizes only membranous lesions. Page 1355 | Table 38-6

How are glucose and insulin used to treat hyperkalemia associated with acute renal failure? a. Glucose has an osmotic effect, which attracts water and sodium, resulting in more dilute blood and a lower potassium concentration. b. When insulin transports glucose into the cell, it also carries potassium with it. c. Potassium attaches to receptors on the cell membrane of glucose and is carried into the cell. d. Increasing insulin causes ketoacidosis, which causes potassium to move into the cell in exchange for hydrogen.

ANS: B This selection is the only option that accurately describes glucose metabolism, causing potassium to move to the intracellular fluid; insulin infusions therefore can be effective in shifting potassium from the extracellular to intracellular space, along with the transport of glucose. Page 1363

What is the most common manifestation of portal hypertension-induced splenomegaly? a. Leukopenia b. Thrombocytopenia c. Erythrocytopenia d. Pancytopenia

ANS: B Thrombocytopenia (decreased platelet count) is the most common manifestation of congestive splenomegaly and can contribute to a tendency of increased bleeding. Page 1452

In acute hypothermia, what physiologic change shunts blood away from the colder skin to the body core in an effort to decrease heat loss? a. Hypotension b. Peripheral vasoconstriction c. Voluntary muscle movements d. Shivering

ANS: B Tissue hypothermia slows the rate of chemical reactions (tissue metabolism), increases the viscosity of the blood, slows blood flow through microcirculation, facilitates blood coagulation, and stimulates profound vasoconstriction. The remaining options do not fulfill this objective. Page 501

Clinical manifestations that include irregular or heavy bleeding, the passage of large clots, and the depletion of iron stores support which diagnosis? a. Premenstrual syndrome b. Dysfunctional uterine bleeding c. Polycystic ovary syndrome d. Primary dysmenorrhea

ANS: B Unpredictable and variable bleeding, in terms of amount and duration, characterize dysfunctional uterine bleeding. Especially during perimenopause, dysfunctional bleeding also may involve flooding and the passage of large clots, which often indicate excessive blood loss. Excessive bleeding can lead to iron-deficiency anemia. This option is the only answer that demonstrates the clinical manifestations described. Page 809

Which statement is true regarding upper motor neurons? a. Upper motor neurons directly influence muscles. b. They modify spinal reflex arcs. c. Upper motor neurons are located in the gray matter of the spinal cord. d. They extend their dendritic processes out of the CNS.

ANS: B Upper motor neurons (i.e., corticospinal tract) are the classification of motor pathways completely contained within the CNS. Their primary roles include directing, influencing, and modifying reflex arcs, lower-level control centers, and motor and some sensory neurons. The remaining options do not accurately describe the characteristic functions of upper motor neurons. Page 462

The nurse recognizes that a patient's diagnosis of a viral infection of the brain's meningeal layer is supported by which diagnostic laboratory result? a. Chloride <110 mEq/L b. Leukocytes >10/mm3 c. Protein <10 mg/dl d. Glucose <40 mg/dl

ANS: B Viral infections causing meningitis will produce a white blood cell (leukocyte) count greater than 10/mm 3. The chloride level is observed in tuberculous meningitis, the protein level has little clinical significance, and the glucose level is observed in patients with acute bacterial meningitis. Page 479 | Table 15-8

A heat stroke is characterized by: a. Core temperatures usually reaching approximately 39.9° C (103.9° F) b. Sweat production on the face occurring even during dehydration c. A rapidly decreasing core temperature as heat loss from the evaporation of sweat ceases d. Symptoms caused by the loss of sodium and prolonged sweating

ANS: B When the core temperature reaches or exceeds 40.5° C (104.9° F), the brain may be preferentially cooled by maximal blood flow through the veins of the head and face, specifically the forehead. Sweat production on the face is maintained even during dehydration. The remaining options do not occur during heat stroke. Page 500

What stimulus causes posthyperventilation apnea (PHVA)? a. Changes in PCO2 levels b. Changes in PaCO2 levels c. Damage to the forebrain d. Any arrhythmic breathing pattern

ANS: B With normal breathing, a neural center in the forebrain (cerebrum) produces a rhythmic breathing pattern. When consciousness decreases, lower brainstem centers regulate the breathing pattern by responding only to changes in PaCO2 levels. This irregular breathing pattern is called PHVA. The other options are not responsible for PHVA. Pages 529-530 | Table 17-4

What are normal characteristics of aging of the male reproductive system? (Select all that apply.) a. Reduced sperm count b. Slower, less forceful ejaculations c. Testicular atrophy and softening d. Longer time to achieve full erection e. Decreased levels of testosterone

ANS: B, C, D, E The described effects on ejaculation, testes, erection, and testosterone are normal characteristics of male aging. Sperm count remains normal with age, although the semen tends to contain more defective and nonmotile sperm. Pages 795-796

Which statements are true regarding renal colic? (Select all that apply.) a. Renal colic results in mild-to-moderate pain. b. Pain originates in the flank area. c. Renal colic indicates the presence of renal stones. d. Pain radiated to the groin. e. Renal colic indicates obstruction of the renal pelvis or proximal ureter.

ANS: B, C, D, E Renal colic is described as moderate-to-severe pain often originating in the flank and radiating to the groin. It usually indicates obstruction by renal stones of the renal pelvis or proximal ureter. Page 1344

Which statements are true concerning the sexually transmitted infection (STI) lymphogranuloma venereum? (Select all that apply.) a. Lymphogranuloma venereum is an STI commonly diagnosed in the United States. b. It begins as a skin infection. c. Lymphogranuloma venereum spreads to lymph tissues. d. Primary and secondary lesions are apparent with this STI. e. A 60-day course of oral erythromycin is the recommended treatment.

ANS: B, C, D Lymphogranuloma venereum is a chronic STI uncommon in the United States. The lesion begins as a skin infection and spreads to the lymph tissue, causing inflammation, necrosis, buboes, and abscesses of the inguinal lymph nodes. Primary lesions appear on the penis and scrotum in men and on the cervix, vaginal wall, and labia in women. Secondary lesions involve inflammation and swelling of the lymph nodes with the formation of large blue buboes that rupture and form draining ulcerative lesions. A 21-day or longer course of oral doxycycline or erythromycin is needed for treatment. Treatment of sexual partners is recommended. Pages 931-932

Which cranial nerves contain parasympathetic nerves? (Select all that apply.) a. I (olfactory) b. III (oculomotor) c. VII (facial) d. IX (glossopharyngeal) e. X (vagus)

ANS: B, C, D, E All but cranial nerve I (olfactory) contain parasympathetic nerves. Page 473

Prerenal injury from poor perfusion can result from which condition? (Select all that apply.) a. Bilateral ureteral obstruction b. Renal vasoconstriction c. Renal artery thrombosis d. Hemorrhage e. Hypotension

ANS: B, C, D, E Poor perfusion can result from renal artery thrombosis, hypotension related to hypovolemia (dehydration, diarrhea, fluid shifts) or hemorrhage, renal vasoconstriction and alterations in renal regional blood flow, microthrombi, or kidney edema that restricts arterial blood flow. Bilateral ureteral obstruction is not associated with prerenal injuries. Page 1360

What are the initial clinical manifestations immediately noted after a spinal cord injury? (Select all that apply.) a. Headache b. Bladder incontinence c. Loss of deep tendon reflexes d. Hypertension e. Flaccid paralysis

ANS: B, C, E A complete loss of reflex function in all segments below the level of the lesion characterizes a spinal cord injury. Severe impairment below the level of the lesion is obvious; it includes paralysis and flaccidity in muscles, absence of sensation, loss of bladder and rectal control, transient drop in blood pressure, and poor venous circulation. Neither headache nor hypertension is an initial clinical manifestation related to a spinal cord injury. Page 592

The sympathetic nervous system primarily serves to protect an individual by doing which of the following? (Select all that apply.) a. Decreasing mucous production b. Increasing blood sugar levels c. Increasing body temperature d. Decreasing sweat excretion e. Increasing blood pressure

ANS: B, C, E In general, sympathetic stimulation promotes responses that are concerned with the protection of the individual, which include increasing blood sugar levels, temperature, and blood pressure. The remaining options are not protective measures resulting from initiatives made by the sympathetic nervous system. Page 476

Which neurotransmitter is inhibited in generalized anxiety disorder (GAD)? (Select all that apply.) a. Acetylcholine b. Serotonin c. Dopamine d. Norepinephrine e. Epinephrine

ANS: B, D Abnormalities in the norepinephrine and serotonin systems were reported in GAD. The other options are more related to depression. Page 655

What requirements are necessary for calcium to be absorbed through the ileum at concentrations of less than 5 mmol/L? (Select all that apply.) a. Receptor site on the ileum b. Vitamin D3 c. Vitamin K d. Carboxypeptidase e. A carrier protein

ANS: B, E At concentrations less than 5 mmol/L, calcium is transported actively across cell membranes, bound to a carrier protein. The carrier formation requires the presence of the active form of vitamin D3 (1,25-dihydroxyvitamin D). These selections are the only options relevant to the transport of calcium. Page 1405

What are the common clinical manifestations of endometriosis? (Select all that apply.) a. Back and flank pain b. Infertility c. Dysuria d. Amenorrhea e. Dysmenorrhea

ANS: B, E Common clinical manifestations primarily include infertility, dysmenorrhea, dyschezia (pain on defecation), and dyspareunia (pain on intercourse). Page 824

Dysfunctional uterine bleeding (DUB), secondary to ovarian dysfunction, is abnormal uterine bleeding resulting from: (Select all that apply.) a. Endometriosis b. Progesterone deficiency c. Sexually transmitted infections d. Congenital abnormalities in the uterine structure e. Estrogen excess

ANS: B, E Of the options available, DUB, secondary to ovarian dysfunction, is a result of either progesterone deficiency or unopposed estrogen excess. Pages 808-809

Which forces create passive transport of water in the proximal tubule? (Select all that apply.) a. Peritubular capillary hydrostatic pressure b. Peritubular capillary oncotic pressure c. Interstitial hydrostatic pressure d. Interstitial osmotic pressure e. Peritubular capillary osmotic pressure

ANS: B, E The osmotic force generated by active sodium transport promotes the passive diffusion of water out of the tubular lumen and into the peritubular capillaries. The elevated oncotic pressure of the blood in the peritubular capillaries further enhances the passive transport of water. The remaining options are not forces that create passive transport of water in the proximal tubule. Page 1330

Considering the mediating factors of premenstrual syndrome (PMS), which medication may be used either continually or only during the menstrual period as a treatment for the condition? a. NSAIDs b. Estrogen c. SSRIs d. Progesterone

ANS: C A selective serotonin reuptake inhibitors (SSRI) (an antidepressant) relieves symptoms in approximately 60% to 90% of women and may be continually administered or only prescribed during the premenstrual period. Oral contraceptive pills that contain estrogen and progesterone also can be continuously used for up to 3 months to decrease the frequency of menstrual periods, PMS, and premenstrual dysphoric disorder (PMDD). Nonsteriodal antiinflammatory drugs (NSAIDs) would not be continually administered. Page 813

The basis of the specificity theory of pain is that: a. Injury to specific organs results in specific types of pain. b. Chronic pain is generally less intense than acute pain. c. The greater the tissue injury, the greater the pain. d. Acute pain is specific only to certain injuries.

ANS: C According to the specificity theory, a direct relationship exists between the intensity of pain and the extent of tissue injury. The remaining options are not accurate statements regarding this pain theory. Page 485

Pricking one's finger with a needle would cause minimal pain, whereas experiencing abdominal surgery would produce more pain. This distinction is an example of which pain theory? a. Gate control theory b. Intensity theory c. Specificity theory d. Pattern theory

ANS: C According to the specificity theory, a direct relationship exists between the intensity of pain and the extent of tissue injury. The remaining options are not related to the intensity of perceived pain. Page 485

The major relay station of sensory information is located in the: a. Basal ganglia b. Midbrain c. Thalamus d. Hypothalamus

ANS: C Although the organization of all of the ascending tracts is complex, the principal target for nociceptive afferents is the thalamus, which, in general, is the major relay station of sensory information. The remaining options do not fulfill this objective. Page 487

Neurofibrillary tangles characterize which neurologic disorder? a. Dementia syndrome b. Delirium c. Alzheimer disease d. Parkinson disease

ANS: C Amyloid plaques, neurofibrillary tangles, as well as neuronal and synaptic losses in the brain, characterize Alzheimer disease. Pages 546-549 | Table 17-13

What characteristic is a medical criterion of brain death? a. Akinetic mutism b. Coma c. Apnea d. Locked-in syndrome

ANS: C Apnea is viewed as a criterion of brainstem death, whereas the remaining options reflect cerebral death. Pages 533-534

The most common primary central nervous system (CNS) tumor is the: a. Microglioma b. Neuroblastoma c. Astrocytoma d. Neuroma

ANS: C Astrocytomas are the most common primary CNS tumors (50% of all brain and spinal cord tumors). The other options do not occur as frequently. Page 629

Which change is a result of puberty and defends the vagina from infection? a. The pH stabilizes between 7 and 8. b. A thin squamous epithelial lining develops. c. Vaginal pH becomes more acidic. d. Estrogen levels are low.

ANS: C At puberty, the pH becomes more acidic (4 to 5) and the squamous epithelial lining thickens. These changes are maintained until menopause (cessation of menstruation), at which time the pH rises again to more alkaline levels and the epithelium thins out. Therefore protection from infection is greatest during the years when a woman is most likely to be sexually active. Estrogen does not play a role in infection protection. Page 773

A man who sustained a cervical spinal cord injury 2 days ago suddenly develops severe hypertension and bradycardia. He reports severe head pain and blurred vision. The most likely explanation for these clinical manifestations is that he is: a. Experiencing acute anxiety b. Developing spinal shock c. Developing autonomic hyperreflexia d. Experiencing parasympathetic areflexia

ANS: C Autonomic hyperreflexia is the only option that is characterized by paroxysmal hypertension (up to 300 mm Hg systolic), a pounding headache, blurred vision, sweating above the level of the lesion with flushing of the skin, nasal congestion, nausea, piloerection caused by pilomotor spasm, and bradycardia (30 to 40 beats per minute). Pages 593-594

Which mineral accounts for the most common type of renal stone? a. Magnesium-ammonium-phosphate b. Uric acid c. Calcium oxalate d. Magnesium phosphate

ANS: C Calcium stones (calcium phosphate or calcium oxalate) account for 70% to 80% of all stones requiring treatment. Page 1343

When comparing the effects of acute and chronic pain on an individual, chronic pain is more often: a. The external event that results in a sense of fear b. Viewed as being meaningful but undesirable c. A factor that contributes to depression d. A sense of internal unease

ANS: C Chronic pain is often associated with a sense of hopelessness and helplessness as relief becomes more elusive and the timeframe more protracted. The pain is perceived as meaningless, and depression is often a concomitant finding, as either a result of the chronic pain state or as a contributor to its development. Individuals often psychologically respond to acute pain with fear (e.g., fear of diagnosis, fear of continued pain), anxiety, and a general sense of unpleasantness or unease. Page 492

Coronary artery disease is most affected by which component of sleep? a. Non-REM b. Light c. REM d. Delta wave

ANS: C Coronary artery disease is most affected during rapid eye movement (REM) sleep. During this component of sleep, dreams may provoke nocturnal angina, increased heart rate, and electrocardiographic (ECG) changes. The other options are not associated with coronary artery disease. Page 506

Which term is used to identify the descent of the posterior bladder and trigone into the vaginal canal? a. Rectocele b. Vaginocele c. Cystocele d. Enterocele

ANS: C Cystocele is the only term used to identify the descent of a portion of the posterior bladder wall and trigone into the vaginal canal; the trauma of childbirth is usually the cause. Page 819

Fluid-filled squishy sacs characterize which breast disorder? a. Paget disease b. Cysts c. Nonproliferative breast lesions d. Lobular carcinoma in situ

ANS: C Cysts (fluid-filled sacs) are a specific type of lump that commonly occurs in women in their 30s, 40s, and early 50s. Cysts feel squishy when they occur close to the surface of the breast; however, when deeply embedded, cysts can feel hard. The other options do not accurately identify the disorder associated with these symptoms. Pages 838-839

Which person is at the greatest risk for developing delirium? a. An individual with diabetes celebrating a 70th birthday b. A depressed Hispanic woman c. An individual on the second day after hip replacement d. A man diagnosed with schizophrenia

ANS: C Delirium is associated with autonomic nervous system overactivity and typically develops in 2 to 3 days, most commonly in critical care units, postsurgically, or during withdrawal from CNS depressants (e.g., alcohol, narcotic agents). Age, gender, and chronic illnesses are not generally associated with delirium triggers. Page 545

During the time that ovulation occurs, which statement concerning basal body temperature (BBT) is true? a. BBT increases. b. BBT decreases. c. BBT fluctuates around 37° C (98° F). d. BBT rises consistently above 37.8° C (100° F).

ANS: C During the follicular phase of ovulation, the BBT fluctuates around 37° C (98° F). Page 782

Enkephalins and endorphins act to relieve pain by which process? a. Inhibiting cells in the substantia gelatinosa b. Stimulating the descending efferent nerve fibers c. Attaching to opiate receptor sites d. Blocking transduction of nociceptors

ANS: C Enkephalins and endorphins are neurohormones that act as neurotransmitters by binding to one or more G protein-coupled opioid receptors and thus relieving pain. The other options are not accurate descriptions of how enkephalins and endorphins relieve pain. Pages 490-491

How does the release (increase) of epinephrine raise body temperature? a. The release of epinephrine causes shivering. b. It affects muscle tone. c. It raises the metabolic rate. d. It increases and strengthens the heart rate.

ANS: C Epinephrine and norepinephrine produce a rapid transient increase in heat production by raising the body's basal metabolic rate. The other options are not correct descriptions of the effects of epinephrine on body heat. Page 496

When does the male body begin to produce sperm? a. Before birth b. Shortly after birth c. At puberty d. When erection is possible

ANS: C Erections begin in utero and continue throughout life, but ejaculation does not occur until sperm production begins at puberty. Page 787

Which hormone is synthesized and secreted by the kidneys? a. Antidiuretic hormone b. Aldosterone c. Erythropoietin d. Angiotensinogen

ANS: C Erythropoietin is produced by the fetal liver and in the adult kidney and is essential for normal erythropoiesis. This statement is not true of the other options. Page 1334

How does an established gonococcal infection usually express itself in newborns? a. Generalized skin rash 4 to 6 days after birth b. Systemic infection with fever c. Bilateral corneal ulceration d. Yellow vaginal or penile discharge approximately 10 days after birth

ANS: C Established infection causes bilateral corneal ulceration. The other options are not usual sites for such an infection. Page 922

What parts of the brain mediate the expression of affect, both emotional and behavioral states? a. Hypothalamus and subthalamus b. Parietal and frontal lobes c. Limbic system and prefrontal cortex d. Basal ganglia and medulla oblongata

ANS: C Extensive connections with the limbic system and prefrontal cortex mediate the expression of affect, both emotional and behavioral states. The remaining options are not involved in these expressions. Page 457

Which hormone stimulates gonads to produce both male and female hormones? a. Gonadotropin-releasing hormone (GnRH) b. Follicle-stimulating hormone (FSH) c. Luteinizing hormone (LH) d. Estrogen

ANS: C Extrahypothalamic factors cause the hypothalamus to secrete GnRH, which stimulates the anterior pituitary to secrete gonadotropins—FSH and LH. These hormones, in turn, stimulate the gonads (ovaries or testes) to secrete female or male sex hormones. Pages 769-771

Meningiomas characteristically compress from: a. Within neural tissues b. Outside spinal nerve roots c. Outside the spinal cord d. Within the subarachnoid space

ANS: C Extramedullary spinal cord tumors produce dysfunction by compression of adjacent tissue, not by direct invasion. The pathologic characteristic of meningiomas is not appropriately described by the other options. Page 633

Which of the following causes condylomata acuminata or genital warts? a. Chlamydia b. Adenovirus c. Human papillomavirus (HPV) d. Herpes simplex virus 1 (HSV-1)

ANS: C Genital warts are quite contagious and are a result of only HPV. Pages 934-935

Having ejected a mature ovum, the ovarian follicle develops into a(n): a. Atretic follicle b. Thecal follicle c. Corpus luteum d. Functional scar

ANS: C Having ejected a mature ovum, the only resulting structure is the corpus luteum. Page 776

Atrial fibrillation, rheumatic heart disease, and valvular prosthetics are risk factors for which type of stroke? a. Hemorrhagic b. Thrombotic c. Embolic d. Lacunar

ANS: C High-risk sources for the onset of embolic stroke are atrial fibrillation (15% to 25% of strokes), left ventricular aneurysm or thrombus, left atrial thrombus, recent myocardial infarction, rheumatic valvular disease, mechanical prosthetic valve, nonbacterial thrombotic endocarditis, bacterial endocarditis, patent foramen ovale, and primary intracardiac tumors. These are not risk factors for the other options provided. Page 599

Persistent symptoms of posttraumatic stress disorder (PTSD) include: a. Depression b. Hypertension c. Nightmares d. Poor nutrition

ANS: C In PTSD, the individual re-experiences the traumatic event as intrusive collections or flashbacks during the day and during persistent nightmares. Nightmares replicate the traumatic experiences and often prevent sleep. The remaining options are not typically attributed to PTSD. Page 656

Which gland produces the associated hormones that are found in high levels in a female fetus? a. Posterior pituitary excretes gonadotropin-releasing hormone (GnRH) and luteinizing hormone (LH). b. Hypothalamus excretes luteinizing hormone (LH) and gonadotropin-releasing hormone (GnRH). c. Anterior pituitaryfollicle-stimulating hormone (FSH) and luteinizing hormone (LH). d. Hypothalamus excretes gonadotropin-releasing hormone (GnRH) and follicle-stimulating hormone (FSH).

ANS: C In the female fetus, the anterior pituitary excretes high levels of two gonadotropins—FSH and LH. The other sequences are not correct. Pages 769-770

What type of cerebral edema occurs when permeability of the capillary endothelium increases after injury to the vascular structure? a. Cytotoxic b. Interstitial c. Vasogenic d. Ischemic

ANS: C Increased permeability of the capillary endothelium of the brain after injury to the vascular structure causes vasogenic edema. The remaining options are not consistent with this description. Pages 557-558

Alterations in which part of the brain are linked to hallucinations, delusions, and thought disorders associated with schizophrenia? a. Parietal lobe b. Limbic system c. Temporal lobe d. Hypothalamus

ANS: C Only temporal lobe alterations may be responsible for the production of positive schizophrenic symptoms, such as hallucinations, delusions, thought disorders, and bizarre behavior. Page 642

What term is used to describe the complication that can result from a spinal cord injury above T6 that is producing paroxysmal hypertension, as well as piloerection and sweating above the spinal cord lesion? a.Craniosacral dysreflexia b. Parasympathetic dysreflexia c. Autonomic hyperreflexia d. Retrograde hyperreflexia

ANS: C Individuals most likely to be affected have lesions at the T6 level or above. Paroxysmal hypertension (up to 300 mm Hg systolic), a pounding headache, blurred vision, sweating above the level of the lesion with flushing of the skin, nasal congestion, nausea, piloerection caused by pilomotor spasm, and bradycardia (30 to 40 beats/minute) characterize autonomic hyperreflexia. No other options appropriately describe this complication. Pages 593-594

What is the primary cause of peptic ulcers? a. Hypersecretion of gastric acid b. Hyposecretion of pepsin c. Helicobacter pylori d. Escherichia coli

ANS: C Infection with H. pylori is a primary cause of peptic ulcers. Page 1435

Which term is used to identify benign uterine tumors that develop from smooth muscle cells in the myometrium and are commonly called uterine fibroids? a. Endometrial polyps b. Myometrial polyps c. Leiomyomas d. Myometriomas

ANS: C Leiomyomas, commonly called myomas or uterine fibroids, are benign smooth muscle tumors in the myometrium (see Figure 24-14). The other terms do not accurately identify the tumors described. Page 821

Which cells have phagocytic properties similar to monocytes and contract like smooth muscles cells, thereby influencing the glomerular filtration rate? a. Principle cells b. Podocin cells c. Mesangial cells d. Intercalated cells

ANS: C Mesangial cells and the mesangial matrix, secreted by mesangial cells, lie between and support the glomerular capillaries. Different mesangial cells contract like smooth muscle cells to regulate glomerular capillary blood flow. They also have phagocytic properties similar to monocytes. The other options are not capable of these functions. Page 1321

The neurotransmitter, norepinephrine, is secreted in the: a. Somatic nervous system b. Parasympathetic preganglion c. Sympathetic postganglion d. Parasympathetic postganglion

ANS: C Most postganglionic sympathetic fibers release norepinephrine (adrenaline). The remaining options do not reflect the correct site of norepinephrine secretion. Pages 473-474

Which nerves are capable of regeneration? a. Nerves within the brain and spinal cord b. Peripheral nerves that are cut or severed c. Myelinated nerves in the peripheral nervous system d. Unmyelinated nerves of the peripheral nervous system

ANS: C Nerve regeneration is limited to only myelinated fibers and generally occurs only in the peripheral nervous system. Pages 450-451

Detrusor hyperreflexia develops from neurologic disorders that originate where? a. Spinal cord between C2 and S1 b. Spinal cord between S2 and S4 c. Above the pontine micturition center d. Below the cauda equina

ANS: C Neurologic disorders that develop above the pontine micturition center result in detrusor hyperreflexia, also known as an uninhibited or reflex bladder. This selection is the only option responsible for detrusor hyperreflexia. Page 1345

Considering the normal frequency of bowel evacuation, how infrequently can evacuation occur and still be considered within normal range? a. Once a day b. Once every 2 days c. Once a week d. Once every 2 weeks

ANS: C Normal bowel habits range from two or three evacuations per day to one per week. Page 1428

Which type of nerve fibers transmits pain impulses? a. A-alpha (A) fibers b. A-beta (A) fibers c. A-delta (A) fibers d. B fibers

ANS: C Of the available options, only medium-sized A fibers transmit pain impulses. Page 486

What is the most common infectious cause of orchitis and one that usually affects postpubertal boys? a. Herpes b. Escherichia coli c. Mumps d. Cytomegalovirus

ANS: C Of the options available, mumps is the most common infectious cause of orchitis and usually affects postpubertal boys. Pages 894-895

What are the cardinal symptoms of small intestinal obstruction? a. Constant, dull pain in the lower abdomen relieved by defecation b. Acute, intermittent pain 30 minutes to 2 hours after eating c. Colicky pain caused by distention, followed by vomiting d. Excruciating pain in the hypogastric area caused by ischemia

ANS: C Of the options available, only colicky pain caused by distention followed by vomiting are considered the cardinal symptoms of a small intestinal obstruction. Pages 1432-1433

Bladder cancer is associated with the gene mutation of which gene? a. c-erbB2 b. Human epidermal growth factor receptor 2 (HER2) c. TP53 d. myc

ANS: C Oncogenes of the ras gene family and tumor-suppressor genes including TP53 mutations and the inactivation of the retinoblastoma gene (pRb) are implicated in bladder cancer. This process is not associated with the other options. Page 1348

What are typical findings on breast palpation of a woman diagnosed with simple fibroadenoma? a. Painful, round, movable, and fluid-filled mass b. Painless, movable, hard, and irregular mass c. Smooth, solid, mobile, and well-circumscribed mass d. Smooth, nonmovable, irregular, and soft mass

ANS: C Only this option accurately identifies the typical palpation results of a simple fibroadenoma. Pages 840-841

What statement concerning the pathogenetic mechanisms of polycystic ovarian syndrome (POS) is true? a. POS causes a decrease in leptin levels; this decrease reduces the hypothalamic pulsatility of gonadotropin-releasing hormone, which reduces the number of follicles that mature. b. POS is a result of a disorder in the anterior pituitary that increases the follicle-stimulating hormone, which reduces the luteinizing hormone released. c. POS is a result of a combination of conditions that include oligo-ovulation or anovulation, elevated levels of androgens, or clinical signs of hyperandrogenism and polycystic ovaries. d. POS inhibits testosterone, which stimulates androgen secretion by the ovarian stroma and indirectly reduces sex hormone-binding globulin.

ANS: C POS has at least two of the following conditions: oligo-ovulation or anovulation, elevated levels of androgens, or clinical signs of hyperandrogenism and polycystic ovaries. Of the options available, only this answer accurately defines the pathogenetic mechanisms of POS. Page 810

The link between major depression and cortisol secretion is that individuals with depression: a. Show suppression of plasma cortisol when given dexamethasone. b. Have a decreased plasma cortisol level, despite the administration of exogenous corticosteroids. c. Show that persistently elevated plasma cortisol levels can result in inflammation that is believed to trigger depression. d. Have normal plasma cortisol levels throughout the day when they take antidepressant medication as prescribed.

ANS: C Persistent elevations in cortisol may also induce immunosuppression that compromises the body's immune systems to contain inflammation and infectious diseases. Increasing evidence suggests that inflammation is another risk factor that triggers the onset of depression. The options related to dexamethasone and exogenous corticosteroids are not true as they apply to depression and cortisol secretion. Page 648

Which term is also used to refer to paradoxic sleep? a. Non-REM b. Light c. REM d. Delta wave

ANS: C REM sleep is also known as paradoxic sleep because the electroencephalographic (EEG) pattern is similar to the normal awake pattern. None of the other terms are used to identify paradoxic sleep. Page 503

What process is capable of increasing both intrathoracic and intraabdominal pressure, thereby facilitating defecation? a. Relaxation of the internal anal sphincter b. Intestinal peristalsis c. Valsalva maneuver d. Ileogastric reflex

ANS: C Squatting or sitting facilitate defecation; these positions straighten the angle between the rectum and anal canal and increase the efficiency of straining (increasing intraabdominal pressure). Initiating the Valsalva maneuver increases intraabdominal pressure. This maneuver consists of inhaling and forcing the diaphragm and chest muscles against the closed glottis and increases both intrathoracic and intraabdominal pressure, which is transmitted to the rectum. None of the other options facilitate defecation as described. Page 1408

Dilated and sluggish pupils, widening pulse pressure, and bradycardia are clinical findings evident of which stage of intracranial hypertension? a. 1 b. 2 c. 3 d. 4

ANS: C Stage 3 intracranial hypertension exhibits clinical manifestations that include decreasing levels of arousal, Cheyne-Stokes respiration or central neurogenic hyperventilation, pupils that become sluggish and constricted, widened pulse pressure, and bradycardia. These responses are not characteristic of any other stage. Page 556

Which statement is false regarding the sources of increased ammonia that contribute to hepatic encephalopathy? a. End products of intestinal protein digestion are sources of increased ammonia. b. Digested blood leaking from ruptured varices is a source of increased ammonia. c. Accumulation of short-chain fatty acids that is attached to ammonia is a source of increased ammonia. d. Ammonia-forming bacteria in the colon are sources of increased ammonia.

ANS: C The accumulation of short-chain fatty acids, serotonin, tryptophan, and false neurotransmitters probably contributes to neural derangement and is not associated with ammonia levels. The other options provide accurate information regarding how the sources of ammonia contribute to hepatic encephalopathy. Pages 1454-1455

The adult intestine processes approximately how many liters of luminal content per day? a. 3 b. 6 c. 9 d. 12

ANS: C The adult intestine processes approximately 9 L of luminal content per day. Of this amount, 2 L is ingested and the remaining 7 L consists of intestinal secretions. Page 1425

Where is the usual site of fertilization of an ovum? a. Trumpet end of the fallopian tubes b. Fimbriae of the fallopian tubes c. Ampulla of the fallopian tubes d. Os of the fallopian tubes

ANS: C The ampulla, or distal third, of the fallopian tube is the usual site of fertilization (see Figure 23-7). Page 775

Which anatomic structure secretes follicle stimulating hormone (FSH) and luteinizing hormone (LH)? a. Hypothalamus b. Ovaries c. Anterior pituitary d. Adrenal cortex

ANS: C The anterior pituitary is the gland that secretes FSH and LH. Pages 780-781

Which statement is false regarding the pathophysiologic process of acute pancreatitis? a. Bile duct or pancreatic duct obstruction blocks the outflow of pancreatic digestive enzymes. b. Acute pancreatitis can also result from direct cellular injury from drugs or viral infection. c. Acute pancreatitis is an autoimmune disease in which immunoglobulin G (IgG) coats the pancreatic acinar cells; consequently, the pancreatic enzymes destroy the cells. d. Acute pancreatitis is usually mild and spontaneously resolves.

ANS: C The backup of pancreatic secretions and the activation and release of enzymes (activated trypsin activates chymotrypsin, lipase, and elastase) within the pancreatic acinar cells cause acute pancreatitis, an obstructive disease. The activated enzymes cause autodigestion (e.g., proteolysis, lipolysis) of the pancreatic cells and tissues, resulting in inflammation. Acute pancreatitis is usually a mild disease and spontaneously resolves; however, approximately 20% of those with the disease develop a severe acute pancreatitis that requires hospitalization. Pancreatitis develops because of a blockage to the outflow of pancreatic digestive enzymes caused by bile duct or pancreatic duct obstruction (e.g., gallstones). Acute pancreatitis can also result from direct cellular injury from drugs or viral infection. Page 1464

Which sexually transmitted infection frequently coexists with gonorrhea? a. Syphilis b. Herpes simplex virus c. Chlamydia d. Chancroid

ANS: C The coexistence of chlamydial infection with gonorrhea frequently occurs. No coexistence exists with the other options. Page 922

A patient exhibits symptoms including hematuria with red blood cell casts and proteinuria exceeding 3 to 5 g/day, with albumin as the major protein. These data suggest the presence of which disorder? a. Cystitis b. Chronic pyelonephritis c. Glomerulonephritis d. Nephrotic syndrome

ANS: C The data suggest the patient has the disorder known as glomerulonephritis. Two major changes distinctive of more severe glomerulonephritis are (1) hematuria with red blood cell casts and (2) proteinuria exceeding 3 to 5 g/day with albumin as the major protein. These symptoms do not support the diagnosis of the other options. Page 1357

Which of the meninges closely adheres to the surface of the brain and spinal cord and follows the sulci and fissures? a. Dura mater b. Arachnoid c. Pia mater d. Inner dura

ANS: C The delicate pia mater closely adheres to the surface of the brain and spinal cord and even follows the sulci and fissures. The remaining options are not represented in the description provided in the stem. Pages 464-465

Why are water and electrolytes transported in both directions through tight junctions and intercellular spaces rather than across cell membranes? a. The intercellular hydrostatic pressure is inadequate to push the water and electrolytes across the cell membranes. b. A balance of cations and ions among the electrolytes on each side of the cell membranes cannot be maintained. c. The epithelial cell membranes are formed of lipids that are hydrophobic and therefore repel water. d. Receptors on those cell membranes are occupied with a diffusion of amino acids and monosaccharides.

ANS: C The epithelial cell membranes of the small intestine are formed of lipids and are hydrophobic or tend to repel water. Therefore water and electrolytes are transported in both directions—toward the capillary blood or toward the intestinal lumen—through the tight junctions and intercellular spaces rather than across cell membranes. This selection is the only option that accurately describes why water and electrolytes are transported in both directions. Pages 1401-1402

What is the first sign of puberty in boys? a. Thickening of the scrotal skin b. Growth of pubic hair c. Enlargement of the testes d. Change in voice

ANS: C The first sign of puberty in boys is an enlargement of the testes and a thinning of the scrotal skin. Page 886

Which neuropeptide promotes wakefulness? a. Prostaglandin D2 b. L-tryptophan c. Hypocretins d. Growth factors

ANS: C The hypothalamus, as a major sleep center, secretes hypocretins (orexins), which are neuropeptides that promote wakefulness and rapid eye movement (REM) sleep, as well as appetite, energy consumption, and pleasure or reward. The remaining options do not fulfill this objective. Page 503

With receptive dysphasia (fluent), the individual is able to: a. Respond in writing, but not in speech. b. Produce verbal speech, but not comprehend language. c. Comprehend speech, but not verbally respond. d. Neither respond verbally nor comprehend speech.

ANS: C The individual experiencing receptive dysphasia may be able to produce verbal language, but language is meaningless because of a disturbance in understanding all language. The remaining options do not describe receptive dysphasia. Page 543 | Table 17-10

On average, what percent of cardiac output do the kidneys receive? a. 10% to 20% b. 15% to 20% c. 20% to 25% d. 30% to 35%

ANS: C The kidneys are highly vascular organs and usually receive 1000 to 1200 ml of blood per minute, or approximately 20% to 25% of the cardiac output. Page 1326

Where in the small intestines are lymphocytes, plasma cells, and macrophages produced? a. Brush border b. Microvilli c. Lamina propria d. Crypts of Lieberkühn

ANS: C The lamina propria, which is a connective tissue layer of the mucous membrane, lies beneath the epithelial cells of the villi and contains lymphocytes, plasma cells, which produce immunoglobulins, and macrophages. None of the other options performs the described function. Page 1401

In which disorder are acetylcholine receptor antibodies (IgG antibodies) produced against acetylcholine receptors? a. Guillain-Barré syndrome b. Multiple sclerosis c. Myasthenia gravis d. Parkinson disease

ANS: C The main defect of myasthenia gravis is the formation of autoantibodies (an immunoglobulin G [IgG] antibody) against receptors at the Ach-binding site on the postsynaptic membrane. This defect is not found in any of the other options. Pages 625-626

From which part of the midbrain do cranial nerves IX to XII emerge? a. Midbrain b. Pons c. Medulla oblongata d. Lateral colliculi

ANS: C The nuclei of cranial nerves IX through XII (see Table 15-6 for discussion) are located only in the medulla oblongata. Page 460

Dilation of the ipsilateral pupil, following uncal herniation, is the result of pressure on which cranial nerve (CN)? a. Optic (CN I) b. Abducens (CN VI) c. Oculomotor (CN III) d. Trochlear (CN IV)

ANS: C The oculomotor CN (III) is involved in this manifestation of pupil dilation. None of the other options would result in pupil dilation when subjected to pressure. Page 557 | Box 17-4

The weakness resulting from the segmental paresis and paralysis characteristic of anterior horn cell injury is difficult to recognize because: a. Upper motor neurons are involved. b. The injury is microscopic. c. Two or more nerve roots supply each muscle. d. The person is unable to feel the involved muscles.

ANS: C The paresis and paralysis associated with anterior horn cell injury are segmental; however, because two or more roots supply each muscle, the segmental character of the weakness may be difficult to recognize. The reason this pathophysiologic condition is difficult to recognize is not appropriately explained by any of the other options. Page 571

Thought and goal-oriented behaviors are functions of which area of the brain? a. Cerebellum b. Limbic system c. Prefrontal lobe d. Occipital lobe

ANS: C The prefrontal area is responsible for goal-oriented behavior (i.e., ability to concentrate), short-term or recall memory, and the elaboration of thought and inhibition on the limbic (emotional) areas of the CNS. The remaining options are not involved in these functions. Page 456

Norepinephrine produces what primary response? a. Increased contractility of the heart b. Release of renin from the kidney c. Vasoconstriction d. Pupillary dilation

ANS: C The primary response from norepinephrine is the stimulation of the 1-adrenergic receptors that cause vasoconstriction. The remaining options do not accurately reflect the primary response of norepinephrine. Page 474

Which description is used when a progressive neoplastic change involves the full epithelial thickness of the cervix? a. Cervical intraepithelial neoplasia b. Cervical dysplasia c. Cervical carcinoma in situ d. Invasive carcinoma of the cervix

ANS: C The progressive neoplastic changes of cervical cells are classified on a continuum from cervical intraepithelial neoplasia (dysplasia) to cervical carcinoma in situ (full epithelial thickness of the cervix is involved), which is generally a precursor of invasive carcinoma of the cervix to invasive carcinoma of the cervical tissue. Page 827

Kupffer cells are best described as: a. Natural killer cells that produce interferon-gamma (IFN-) b. Contractile and therefore capable of regulating the sinusoid blood flow c. Bactericidal and therefore central to innate immunity d. Able to metabolize estrogen, progesterone, and androgens

ANS: C The sinusoids are lined with phagocytic cells, known as Kupffer cells, which are part of the mononuclear phagocyte system and are the largest population of tissue macrophages in the body. They are bactericidal and central to innate immunity. None of the other options accurately describe Kupffer cells. Pages 1409-1410

Compared with a younger individual, how is the specific gravity of urine in older adults affected? a. Specific gravity of urine in older adults is increased. b. Specific gravity of urine in older adults is considered high normal. c. Specific gravity of urine in older adults is considered low normal. d. Specific gravity of urine in older adults is decreased.

ANS: C The specific gravity of the urine in older individuals tends to be on the low side of normal. Page 1336

What is the trigone? a. A smooth muscle that comprises the orifice of the ureter b. The inner mucosal lining of the kidneys c. A smooth triangular area between the openings of the two ureters and the urethra d. One of the three divisions of the loop of Henle

ANS: C The trigone is a smooth triangular area lying between the openings of the two ureters and the urethra. The other options do not accurately identify the trigone. Page 1325

Which statement is incorrect regarding the characteristics of vitamin B12? a. Vitamin B12 is absorbed in the terminal ileum. b. Vitamin B12 is absorbed in its free (unbound) form but only in small amounts. c. Vitamin B12 is necessary for platelet maturation. d. Vitamin B12 binds to intrinsic factor.

ANS: C Vitamin B12 is not necessary for platelet maturation. Most vitamin B12 (cobalamin) is bound to intrinsic factor (making it resistant to digestion) and is absorbed in the terminal ileum, although a small amount of the vitamin is absorbed in its free (unbound) form. Page 1406

How much urine accumulates in the bladder before the mechanoreceptors sense bladder fullness? a. 75 to 100 ml b. 100 to 150 ml c. 250 to 300 ml d. 350 to 400 ml

ANS: C When the bladder accumulates 250 to 300 ml of urine, it contracts and the internal urethral sphincter relaxes through activation of the spinal reflex arc (known as the micturition reflex). Page 1326

Bilirubin is a by-product of the destruction of which aged cells? a. Platelets b. Protein c. Leukocytes d. Erythrocytes

ANS: D Bilirubin is a by-product of only the destruction of aged red blood cells or erythrocytes. Page 1411

Which differentiating sign is required to make the diagnosis of pyelonephritis from that of cystitis? a. Difficulty starting the stream of urine b. Spasmodic pain that radiates to the groin c. Increased glomerular filtration rate d. Urinalysis confirmation of white blood cell casts

ANS: D Clinical assessment, alone, is difficult to differentiate the symptoms of cystitis from those of pyelonephritis. Urine culture, urinalysis, and clinical signs and symptoms establish the specific diagnosis. White blood cell casts indicate pyelonephritis, but they are not always present in the urine. This selection is the only option that is considered a required sign of pyelonephritis. Pages 1351-1352

Which intracerebral disease process is capable of producing diffuse dysfunction? a. Closed head trauma with bleeding b. Subdural pus collections c. Neoplasm d. Infarct emboli

ANS: D Disorders within the brain substance (intracerebral)—bleeding, infarcts emboli, and tumors—primarily functioning as masses may cause diffuse dysfunction. Such localized destructive processes directly impair functioning of the thalamic or hypothalamic activating systems. Disorders outside the brain but within the cranial vault (extracerebral), including neoplasms, closed-head trauma with subsequent bleeding, and subdural empyema (accumulation of pus), can cause similar dysfunction. Page 528

After a partial gastrectomy or pyloroplasty, clinical manifestations that include increased pulse, hypotension, weakness, pallor, sweating, and dizziness are the results of which mechanism? a. Anaphylactic reaction in which chemical mediators, such as histamine, prostaglandins, and leukotrienes, relax vascular smooth muscles, causing shock b. Postoperative hemorrhage during which a large volume of blood is lost, causing hypotension with compensatory tachycardia c. Concentrated bolus that moves from the stomach into the small intestine, causing hyperglycemia and resulting in polyuria and eventually hypovolemic shock d. Rapid gastric emptying and the creation of a high osmotic gradient in the small intestine, causing a sudden shift of fluid from the blood vessels to the intestinal lumen

ANS: D Dumping syndrome occurs with varying severity in 5% to 10% of individuals who have undergone partial gastrectomy or pyloroplasty. Rapid gastric emptying and the creation of a high osmotic gradient in the small intestine cause a sudden shift of fluid from the vascular compartment to the intestinal lumen. Plasma volume decreases, causing vasomotor responses, such as increased pulse rate, hypotension, weakness, pallor, sweating, and dizziness. Rapid distention of the intestine produces a feeling of epigastric fullness, cramping, nausea, vomiting, and diarrhea. This selection is the only option that accurately identifies the mechanism responsible for the described situation. Page 1440

Most dysphasias are associated with cerebrovascular accidents involving which artery? a. Anterior communicating b. Posterior communicating c. Circle of Willis d. Middle cerebral

ANS: D Dysphasias are usually associated with a cerebrovascular accident involving the middle cerebral artery or one of its many branches. Damage to or occlusion of any of the other options does not cause dysphasias. Pages 539-540

Which cytokines are endogenous pyrogens? a. IL-3, IL-10, and IL-18 b. IL-2, IL-8, and IFN- c. IL-4, IL-12, colony-stimulating factor, and IFN- d. IL-1, IL-6, TNF-, and IFN-

ANS: D Endogenous pyrogens include prostaglandin E2 (PGE2), interleukin-1 (IL-1), IL-6, tumor necrosis factor-alpha (TNF-), and interferon-gamma (IFN-). The other options are not endogenous pyrogens. Page 498

It is true that myasthenia gravis: a. Is an acute autoimmune disease. c. May result in adrenergic crisis. b. Affects the nerve roots. d. Causes muscle weakness.

ANS: D Exertional fatigue and weakness that worsens with activity, improves with rest, and recurs with resumption of activity characterizes myasthenia gravis. None of the other options are true of myasthenia gravis. Page 626

A communicating hydrocephalus is caused by an impairment of the: a. Cerebrospinal fluid flow between the ventricles b. Cerebrospinal fluid flow into the subarachnoid space c. Blood flow to the arachnoid villi d. Absorption of cerebrospinal fluid

ANS: D Hydrocephalus from impaired absorption outside the ventricles is called communicating (extraventricular) hydrocephalus. The other options do not accurately describe the cause of a communicating hydrocephalus. Page 558

Maintenance of a constant internal environment and the implementation of behavioral patterns are main functions of which area of the brain? a. Thalamus b. Epithalamus c. Subthalamus d. Hypothalamus

ANS: D Hypothalamic function falls into two major areas: (1) maintenance of a constant internal environment, and (2) implementation of behavioral patterns. The remaining options do not address these functions. Page 459

In the 95% of those with delayed puberty, the problem is caused by which condition? a. Disruption in the hypothalamus b. Disruption of the pituitary c. Deficit in estrogen or testosterone d. Physiologic hormonal delays

ANS: D In 95% of cases, delayed puberty is a physiologic delay; that is, hormonal levels are normal and the hypothalamic-pituitary-gonadal (HPG) axis is intact, but maturation is happening slowly. This selection is the only option that accurately describes 95% of those with delayed puberty. Page 886

In 95% of children of delayed puberty, the problem is caused by: a. Disruption in the hypothalamus b. Disruption of the pituitary c. Deficit in estrogen or testosterone d. Physiologic hormonal delays

ANS: D In 95% of children with delayed puberty, the delay is physiologic; that is, hormonal levels are normal and the hypothalamic-pituitary-gonadal (HPG) axis is intact, but maturation is slowly happening. This option is the only answer that accurately describes the most common cause of delayed puberty. Page 802

The type of vascular malformation that most often results in hemorrhage is: a. Cavernous angioma b. Venous angioma c. Capillary telangiectasia d. Arteriovenous malformation

ANS: D In an arteriovenous malformation (AVM), arteries feed directly into veins through a vascular tangle of malformed vessels, causing venous hemorrhaging. The other options are not as likely to result in a hemorrhage. Pages 604-605

Which vascular malformation is characterized by arteries that feed directly into veins through vascular tangles of abnormal vessels? a. Cavernous angioma b. Capillary telangiectasia c. Arteriovenous angioma d. Arteriovenous malformation

ANS: D In only an arteriovenous malformation (AVM), do arteries feed directly into veins through a vascular tangle of malformed vessels. Pages 604-605

The absence of which major hormone is a determinant of sexual differentiation (wolffian system) in utero? a. Estrogen b. Progesterone c. Growth hormone d. Testosterone

ANS: D In the absence of testosterone, a loss of the wolffian system occurs and the two gonads develop into ovaries at 6 to 8 weeks' gestation. Between 6 and 7 weeks' gestation, the male embryo differentiates under the influence of TDF. The presence of estrogen is a determinating factor. None of the other options function as a major determinant to sexual differentiation in utero. Page 769

What is usually the first clinical manifestation of breast cancer? a. Nipple dimpling b. Nipple discharge c. Enlargement of one breast d. Painless lump

ANS: D Invasive carcinoma of the breast generally exhibits a nontender palpable mass or thickened area. This option is the only answer that accurately describes the first clinical manifestation of breast cancer. Page 873

Which statement provides the most accurate information regarding the transmission of herpes simplex virus (HSV)? a. HSV is transmitted only when vesicles are present. b. HSV is transmitted only while lesions are present. c. The use of condoms prevents the transmission of HSV. d. The risk of transmission is present even during latent periods.

ANS: D Latent infections can become reactivated and cause a recurrent infection with similar manifestations. The other options are not accurate statements regarding the transmission of HSV. Pages 933-934

Which characteristic is the most critical index of nervous system dysfunction? a. Size and reactivity of pupils b. Pattern of breathing c. Motor response d. Level of consciousness

ANS: D Level of consciousness is the most critical clinical index of nervous system function or dysfunction. An alteration in consciousness indicates either improvement or deterioration of a person's condition. No other option is used as the critical index of nervous system. Page 529

Antiemetic agents, such as domperidone and haloperidol, are antagonists for which receptors? a. 5-Hydroxytryptamine (5-HT) serotonin b. Histamine-2 c. Acetylcholine d. Dopamine

ANS: D Metoclopramide, domperidone, and haloperidol are dopamine antagonists, making them effective antiemetic agents. This selection is the only option that identifies a receptor that is involved in the process of vomiting. Page 1424

Which sexually transmitted disease occasionally causes clinical manifestations of scant intermittent penile discharge, slight pruritus, and mild dysuria? a. Campylobacter enteritis b. Shigellosis c. Cytomegalovirus enteritis d. Trichomoniasis

ANS: D Most men with trichomoniasis are asymptomatic but may have scant intermittent discharge, slight pruritus, and mild dysuria. The other options do not demonstration these symptoms. Page 937

What is the end-product of protein metabolism that is excreted in urine? a. Glucose b. Ketones c. Bile d. Urea

ANS: D Of the options available, only urea is an end-product of protein metabolism and is the major constituent of urine along with water. Page 1332

Prolonged high environmental temperatures that produce dehydration, decreased plasma volumes, hypotension, decreased cardiac output, and tachycardia cause which disorder of temperature regulation? a. Heat cramps hyperthermia b. Heat stroke c. Malignant d. Heat exhaustion

ANS: D Of the options presented, only heat exhaustion, or collapse, is a result of prolonged high core or environmental temperatures resulting in dehydration, decreased plasma volumes, hypotension, decreased cardiac output, and tachycardia. Page 500

Which statement regarding pelvic inflammatory disease (PID) is true? a. An episode of mild PID can decrease the possibility of a successful pregnancy by 80%. b. Such an inflammation results in temporary changes to the ciliated epithelium of the fallopian tubes. c. PID has not been associated with an increased risk of an ectopic pregnancy. d. Contracting this infection increases the risk of uterine cancer.

ANS: D PID infection results in permanent changes to the ciliated epithelium of the fallopian or uterine tubes. A recent study has found that one episode of mild, subclinical PID resulted in a 40% decrease in later pregnancy rates, and multiple episodes of PID further increase the risk of infertility. Scarring caused by PID greatly increases the risk of a later ectopic pregnancy by up to tenfold. Scarring and adhesions also can result in chronic pelvic pain and, potentially, an increased risk of later uterine cancer. Pages 813-814

Which neurotransmitter is inhibited in panic disorders? a. Norepinephrine b. Serotonin c. Dopamine d. Gamma-aminobutyric acid (GABA)

ANS: D Panic disorder also may involve the GABA-benzodiazepine (BZ) receptor system. The other options are more related to depression. Page 654

What substance stimulates renal hydroxylation in the process of producing vitamin D? a. Erythropoietin b. Thyroid hormone c. Calcitonin d. Parathyroid hormone

ANS: D Parathyroid hormone stimulates renal hydroxylation in the process of producing vitamin D. The first step occurs in the liver with hydroxylation at the 25th carbon (calcifediol); the second step in hydroxylation occurs at the first carbon position in the kidneys. The other options are not involved in this process. Page 1334

Parkinson disease is a degenerative disorder of the brain's: a. Hyothalamus b. Anterior pituitary c. Frontal lobe d. Basal ganglia

ANS: D Parkinson disease is a commonly occurring degenerative disorder of the basal ganglia and not of any of the other brain structures. Pages 564-565

What term is used to identify a condition in which the foreskin cannot be retracted over the glans penis? a. Paraphimosis b. Priapism c. Prephimosis d. Phimosis

ANS: D Phimosis is the only term used to identify the condition in which the foreskin cannot be retracted back over the glans. Page 888

Which midbrain dysfunction causes pupils to be pinpoint size and fixed in position? a. Diencephalon dysfunction b. Oculomotor cranial nerve dysfunction c. Dysfunction of the tectum d. Pontine dysfunction

ANS: D Pinpoint fixed pupils are a result of pontine dysfunction. The diencephalon, oculomotor cranial nerve, and tectum are not involved in such a pupil reaction. Page 532

What is the leading cause of infertility in women? a. Pelvic inflammatory disease b. Endometriosis c. Salpingitis d. Polycystic ovary syndrome

ANS: D Polycystic ovary syndrome remains one of the most common endocrine disturbances affecting women, especially young women, and is a leading cause of infertility in the United States. Page 810

Which statement is false concerning the accumulation of fluid in the peritoneal cavity? a. Impaired excretion of sodium by the kidneys promotes water retention. b. Decreased oncotic pressure and increased hepatic sinusoidal hydrostatic pressure cause the movement of fluid into the peritoneal cavity. c. Decreased blood flow to the kidneys activates aldosterone, which retains sodium. d. Circulating nitric oxide causes vasoconstriction, which forces fluid from the capillaries into the peritoneal cavity.

ANS: D The arterial vasodilation theory proposes that circulating nitric oxide or the release of endotoxin from translocation of intestinal bacteria triggers arterial vasodilation of the splanchnic organs early in the course of cirrhosis and stimulates renal sodium retention through the renin-angiotensin-aldosterone system, increased sympathetic tone, and changes in the intrarenal blood flow. The other options provide accurate information regarding the accumulation of fluid in the peritoneal cavity. Page 1453

Which elements in saliva protect against tooth decay? a. Salivary -amylase and ptyalin b. Secretin and IgA c. Endogenous enamelin and salivary -amylase d. Exogenous fluoride and a pH of 7.4

ANS: D The bicarbonate concentration of saliva sustains a pH of approximately 7.4, which neutralizes bacterial acids and prevents tooth decay. Exogenous fluoride (e.g., fluoride in drinking water) is absorbed and then secreted in the saliva, providing additional protection against tooth decay. Decay prevention is not associated with any of the other options. Page 1395

Antipsychotic drugs block which neurotransmitter receptor? a. Norepinephrine b. Gamma-aminobutyric acid c. Serotonin d. Dopamine

ANS: D The dopamine hypothesis initially suggested that abnormal elevation in dopaminergic transmission contributes to the onset of schizophrenia. This hypothesis was based on pharmacologic studies showing that antipsychotic drugs are potent blockers of brain dopamine receptors; therefore the other options are incorrect. Page 643

How many days does it take for the entire epithelial population of the small intestines to be replaced? a. 30 to 45 b. 15 to 25 c. 7 to 15 d. 4 to 7

ANS: D The entire epithelial population is replaced approximately every 4 to 7 days. Page 1401

Which statement is true regarding the gate control theory (GCT) of pain? a. The pain gate is located in the brain. b. A closed gate increases pain perception. c. The brain primarily controls the pain gate. d. An open gate facilitates the brain in processing the pain.

ANS: D The open gate in the spinal cord regulates the transmission of pain impulses that ascend to the brain for further processing and interpretation, thus leading to the management of pain. The remaining statements are not true when discussing the GCT of pain. Page 485

What is the action of urodilatin? a. Urodilatin causes vasoconstriction of afferent arterioles. b. It causes vasodilation of the efferent arterioles. c. Urodilatin inhibits antidiuretic hormone secretion. d. It inhibits salt and water reabsorption.

ANS: D Urodilatin (a natriuretic peptide) inhibits sodium and water reabsorption from the medullary part of collecting duct, thereby producing diuresis. It is not involved in the actions described by the other options. Pages 1327-1328 | Table 37-1

Spinal cord injuries most likely occur in which region? a. Cervical and thoracic b. Thoracic and lumbar c. Lumbar and sacral d. Cervical and thoracic-lumbar

ANS: D Vertebral injuries most often occur at vertebrae C1-C2 (cervical), C4-C7, and T1-L2 (thoracic lumbar). None of the other options are applicable. Pages 590-591

When renin is released, it is capable of which action? a. Inactivation of autoregulation b. Direct activation of angiotensin II c. Direct release of antidiuretic hormone (ADH) d. Formation of angiotensin I

ANS: D When renin is released, it cleaves an -globulin (angiotensinogen produced by liver hepatocytes) in the plasma to form angiotensin I. Page 1327

Describe the pathophysiology of acute pancreatitis?

Acute pancreatitis also knows as acute hemorrhagic pancreatitis is initiated by intrapancreatic activation of proteases. It usually is a mild disease, but about 20% of those afflicted develop a severe pancreatic inflammation requiring hospital care. Although the precise mechanism or sequence of events of ten is unknown, alcoholism and biliary tract obstruction are commonly associated. Bile reflux into the pancreas occurs if gallstones obstruct the common bile duct and bile contributes to attacks of acute pancreatitis. The most common theory is that pancreatitis develops d/t and injury or disruption of pancreatic acinar cells, which permit leakage of pancreatic enzymes (trypsin, chymotrypsin, elatase) into pancreatic tissue.

Entamoeba histolytica

Amebiasis

Condition is characterized by an inability to have emotional experiences.

Anhedonia

Considering the pathophysiology of primary amenorrhea, what anatomic structure is involved in compartment IV?

Compartment IV disorders include CNS disorders, in particular hypothalamic disorders. In some of the congenital syndromes that cause primary amenorrhea, the hypothalamic-pituitary-ovarian (HPO) axis is dysfunctional. The hypothalamus is unable to synthesize GnRH, so the pituitary fails to secrete LH and FSH. Therefore, the ovary doesn't receive the hormonal signals that normally initiate the ovarian and endometrial changes of the menstrual cycle, and ovulation and menstruation doesn't occur. Because ovarian hormones are absent, estrogen-dependent sex characteristic don't develop.

Brain abscess

Complication of mastoiditis

Glossopharyngeal nerve

Controls motor functions to the pharynx and salivary glands and sensory functions from the pharynx and tongue.

Alterations in immunoglobulin G (IgA) production have been found in individuals with this disorder.

Crohn disease

Casts

Cylindric with distinct borders

Meningitis

CNS manifestation of tuberculosis

Spinal accessory nerve

Carries sensory and motor fibers to the pharynx and larynx.

Hydrocele

Collection of fluid in the tunica vaginalis

Describe the pathogenesis of polycystic ovarian syndrome.

Cause is unknown but a genetic basis is suspected. A hyperandrogenic state is a cardinal feature in the pathogenesis of PCOS. Glucose intolerance/insulin resistance often run parallel and markedly aggravate the hyperandrogenic state, thus contributing to the s/s of PCOS. Obesity adds/worsens PCOS. 50% of normal weight women w/PCOS have IR, all obese women w/PCOS do. (Hyperandrogenic state is an excessive secretion of androgens by the adrenal cortex, ovaries, or testes. The common manifestations in women are hirsutism and virilism. Hyperandrogenism is often caused by either ovarian or adrenal diseases). IR and hyperinsulinemia causes androgen release by the ovaries and a reduction in serum sex hormone binding globulin (SHBG), the net effect of this is increased free testosterone levels. Excessive androgen affects follicular growth and insulin effects follicular decline by suppressing apoptosis and enabling follicles, which would normally disintegrate, to survive. In PCOS there is dysfunction of the follicle development. Inappropriate gonadotropin secretion triggers the beginning of a vicious cycle that perpetuates anovulation. Typically FSH levels are below normal and LH levels are elevated. Persistent LH elevation causes an increase in androgens from the adrenal gland and testosterone, androstenedione, and CHEA from the ovary. Androgens are converted to estrogen in peripheral tissues, and increased testosterone levels cause a significant reduction (50%) in SHBG, which in turn causes increased levels of free estradiol. Elevated estrogen levels trigger a positive feedback response in LH and a negative feedback response in FSH. Because FSH isn't depressed new follicular growth is continuously stimulated but not to full maturation and constant production of steroids in response to gonadotropin stimulation. Thus PCIS is characterized by excessive production of both androgen and estrogen. Increased androgen secretion by the ovaries contributes to premature follicular failure (atresia) and persistent anovulation. In turn, persistent anovulation causes enlarges polycystic ovaries characterized by a smooth, pearly white capsule. This characteristic appearance is caused by an increase in surface area and volume of up to 2.8 times, doubling of growing and atretic follicles, thickening of the tunica (outer area) by 50%, increasing cortical stromal thickening by 1/3rd and a 5 fold increase in subcortical stroma, and escalating hyperplasia. With advancing age, menstrual irregularities may improve while metabolic syndrome and type 2 diabetes increases.

Haemophilus ducreyi

Chancroid

Stimulates the gallbladder to eject bile and the pancreas to secrete alkaline fluid.

Cholecystokinin

Pain that results from tumor infiltration of nerve tissue, from trauma or chemical injury to the nerve, or from damage from radiation, chemotherapy, or surgical sectioning of the nerve

Deafferentation pain

Dysmenorrhea is caused by the release of the chemical mediator: __________.

It's primarily the result of the effects of excessive endometrial prostaglandin production, enhanced by progesterone. Elevated levels of prostaglandins are found in endometrial. Primary dysmenorrhea is painful menstruation associated with release of prostaglandins in ovulatory cycles, but not with pelvic disease.

Clinical manifestations including increased pulse, hypotension, weakness, pallor, sweating, and dizziness following a partial gastrectomy or pyloroplasty are the result of what pathophysiological problem?

Early dumping syndrome. Dumping syndrome, which is the rapid emptying and creation of a high osmotic gradient within the small intestine cause a sudden shift of fluid from the vascular compartment to the intestinal lumen, causing the vasomotor responses mentioned above.

Differentiate encephalitis, cerebral aneurysm and cerebral vascular accident (CVA).

Encephalitis: is an acute febrile illness, usually of viral origin, with nervous system involvement. The most common encephalitides are caused by arthropod-corne viruses and herpes simplex. Intracranial aneurysm: may result from arteriosclerosis, congenital abnormality, trauma, inflammation, or infection. CVA: are classified according to pathophysiology and thus are ischemic, global hypoperfusion, or hemorrhagic. 50% of CVAs occur in persons older than 70 years. Strokes, however, do occur in a 3:10 ration in individuals younger than 65 years. Stroke tends to run families. The incidence of stroke is 2.5 times higher in blacks that whites. Stroke prevalence in 2005 for black men was 2.3 million compared to 3.4 million in black women. The risk of first ever stroke in blacks is almost twice that of whites.

Fluent speech is difficult to comprehend.

Formal thought disorder

Creatinine clearance

Good estimate of glomerular filtration rate

Calymmatobacterium granulomatis

Granuloma inguinale

Which virus is a necessary precursor for developing cervical intraepithelial carcinoma (CIN) and cervical cancer?

HPV, 16 & 18 are high-risk oncogenic types

Perceptions are experienced without external stimulation of the sense organs.

Hallucinations

Erythrocytes

Hematuria

What type of the Hepatitis virus is sexually transmitted?

Hep B is transmitted through contact with infected blood, body fluids, or contaminated needles. Hep B is also a sexually transmitted disease. Although Hep C isn't recognized as a STI, the CDC has listed sexual exposure as an HCV risk factor.

Subarachnoid hemorrhage causes hydrocephalus by what mechanism?

Hydrocephalus from impaired absorption outside the ventricles is called communicating (extraventricular) hydrocephalus. SAH occurs when blood escapes from defective or injured vasculature into the subarachnoid space. When a vessel tears, blood under pressure is pumped into the subarachnoid space. The blood produces an inflammatory reaction in these tissues. Clinical manifestations of an SAH include HA, AMS, transient motor weakness, and numbness and tingling. Vasospasm and delayed cerebral ischemia are serious complications. Treatment of vasospasm includes use of CCB to prevent or reverse vasospasm and augmentation cerebral perfusion by volume expansion and hemodilution.

Define post-traumatic stress disorder (PTSD).

In PTSD, the individual re-experiences the traumatic event as intrusive collections or flashbacks during the day and during persistent nightmares. Nightmares replicate the traumatic experiences and often prevent further sleep. Symptoms of intrusive memory may include: Flashbacks, or reliving the traumatic event for minutes or even days at a time. Upsetting dreams about the traumatic event. Symptoms of avoidance and emotional numbing may include: Trying to avoid thinking or talking about the traumatic event. Feeling emotionally numb, avoiding activities you once enjoyed, hopelessness about the future, memory problems, trouble concentrating, and difficulty maintaining close relationships. Symptoms of anxiety and increased emotional arousal may include: irritability or anger, overwhelming guilt or shame, self-destructive behavior (such as drinking to much), trouble sleeping, being easily startled or frightened, hearing or seeing things that aren't there. Treatment- drug medications and CBT, individual learns to control the anxiety symptoms and memories of the event during therapy. Paroxetine and sertraline 1st line SSRI for chronic. TCA's 2nd line. Chronic TCAs 2nd line. Exposure to a terrifying or life-threatening trauma (serious accidents, natural disaster, child abuse, kidnapping, violent attacks), may produce post-traumatic stress disorder. The disturbance has caused intense fear, threat of death or helplessness. PTSD may develop within hours, several months or over a number of years. Lifetime prevalence rate of PTSD is 7-8%. PTSD may involve several neural structures and neurotransmitter systems. Combat related PTSD shows a smaller hippocampus (the brain structure sensitive to the effects of cortisol stress hormone and excitatory amino acids). Trauma related PTSD shows increased activation in the amygdala and decreased activity in some prefrontal cortical areas. Pediatric PTSD shows the generalized effect of trauma on reducing total brain volume. The amygdala, hippocampus and prefrontal cortex are important in PTSD, as these are areas help in how fearful memories are stored, retrieved or forgotten. The hypothesis is than an altered fear-learning process may involve amygdala hyperresponsiveness, lack of prefrontal cortical inhibition, hippocampal dysfunction and/or susceptibility to the adverse effects of stress.

Define intussusception.

Intussusception is an infolding or prolapse of a segment of the small intestine into the adjacent but more distal segment of the intestine. It usually causes strangulation of the blood supply. Its rare in adults compared with the more frequent occurrence in infants. Intussusception is the telescoping or invagination of one portion of the intestine into another. It's the most common cause of acquired intestinal obstruction in infants. Usually, the ileum invaginates the cecum and part of the ascending colon by collapsing through the ilocecal valve.

How does acute unilateral renal obstruction predispose people to hypertension?

It causes activation of the RAAS because of the decreased perfusion to the affected side. This system causes vasoconstriction of the vessels, which leads to increased BP. Early treatment and correction of obstruction is very important in the care of such a patient.

Delays gastric and small bowel emptying.

Peptide YY

List the most common cause of Hyperpituitarism.

Pituitary adenomas are generally associated with hyperpituitarism. Pituitary adenomas are benign slow growing tumors arising from cells of the anterior pituitary, most commonly those secreting GH and prolactin.

Differentiate the types of headaches: migraine, cluster, and tension.

Migraine: is defined as repeated, episodic HA lasting 4-72 hours. Symptoms are unilateral head pain, worsening with movement, accompanied by photophobia or phonophobia, and presence of any one of the following throbbing quality, moderate to severe nausea or vomiting. Familial episodic disorder/HA, with or w/o an aura, precipitated by triggering event (lack of sleep, overexertion, weather, skipping meals). More prevalent in women. Classic photophobia, triggers, need cool room, sleep, n/v common. With and w/o aura, often genetic. Cortical spreading depression. Neurotransmitters: substance P, calcitonin. Over activity of trigeminovascular system of the brain. Dysfunction of brainstem pathways. Abnormal sensory modulation. The clinical phases of a migraine attack are the premonitory phase, the migraine aura, the HA phase and the recovery phase. Exact mechanism is not known. Trigeminal nerve- brain stem, pain impulses to thalamus to sensory cortex. Activity worsens pain. Goal is to eliminate nocioception to trigeminal nucleus. Monitor diet, decreases stress, improves lifestyle. Migraine HA likely involves: serotonergic and other neurotransmitter alterations. Cluster: The HA attack usually begins w/o warning and is characterized by severe, unilateral, tearing, burning, periorbital, and retrobulbar or temporal pain lasting 30 minutes to 2 hours. Acute and chronic occurs in episodes several times daily for period of days. Unilateral pain, intense tearing and burning. s/s: ptosis, lacrimation, eye redness, and nausea. Sympathetic under activity and parasympathetic over activity, trigeminal activation cause unknown. Chronic paroxysmal heicrania are cluster HA with more frequent daily attacks (4-12 times/day but with shorter duration). They occur primarily in women and respond to treatment with indomethacin. Tension: Its mild-moderate bilateral HA with sensation of a tight band or pressure around the head. Tension-type is the most common. Common in men and women. Due to stress and anxiety mild to moderate pain. Acute and chronic, both a central and peripheral etiology. The HA is bilateral, with the sensation of tight band around the head. Onset is usually gradual. The pain may last for hours or days. Temporal arteritis-HA across temples

What are the clinical manifestations of a urinary tract infection in an older adult?

Older adults with cystitis may be asymptomatic or demonstrate confusion or vague abd discomfort. Older adults with recurrent UTI and other concurrent illness have a higher risk of mortality.

Spermatocele

Painless diverticulum of the epididymis located between the head of the epididymis and the testis

What effect do pyrophosphate, potassium citrate, and magnesium have on crystal formation?

Potassium citrate, pyrophosphate and magnesium are capable of crystal growth inhibition thereby reducing the risk of calcium phosphate or calcium oxalate precipitation in the urine and preventing subsequent stone formation.

Dilation and relaxation of the ureter with hydroureter and hydronephrosis

Pregnancy

Trigeminal nerve

Provides motor and sensory functions to the face, mouth, nose, and eyes.

Which tubule is the only surface inside the nephron where cells are covered with microvilli to increase the reabsorptive surface area?

Proximal tubules

Leukocytes

Pyuria

Testicular torsion

Rotation of a testis, which twists blood vessels of the spermatic cord

Stimulates the pancreas to secrete alkaline pancreatic juices.

Secretin

Which stimulus inhibits gastric motility?

Secretin- intestinal hormone, inhibitory and make threshold potential more negative. Cholecystokinin- inhibits gastric emptying so that fats are not emptied into the duodenum at a rate that exceeds the rate of bile and enzyme secretion.

The most common manifestation of portal hypertension induced splenomegaly is _________.

Splenomegaly is an enlargement of the spleen. Portal HTN contributes to congestive splenomegaly by increasing intrasplenic BP. Thrombocytopenia is the most common manifestation of congestive splenomegaly and can contribute to an increased bleeding tendency.

Pain that occurs after peripheral nerve injury and is described as continuous with severe sensations and a burning quality

Sympathetically maintained pain

Treponema pallidum

Syphilis

The mutation of which gene occurs in cancers of the stomach, colon, liver, gallbladder, and pancreas?

TP53 and K-ras. Stomach-Non-environmental risk factors, family hx, blood type, type A atrophic gastritis, pernicious anemia. Most important risk factor H. pylori infection that carries the CagA gene product cytoxin associated antigen A, salt added to foods, food additives in pickled or salted foods, and low intake of fruits and vegetables. Colon-genetic an environmental factor is associated with the development of CRC. Alterations in the tumor-suppressor TP53 gene are present in 75-85% of patients. Genetic mutations involving DCC and TP53. Liver-5th most common cancer. 3rd leading cause of death worldwide. Increasing d/t increased Hep B and C infection. Usually cause by metastatic spread. Gallbladder- rare in US. Infection with HBV, HCV, and HDV, chronic liver disease, exposure to mycotoxins. Most significant mytotoxins are the aflatoxins particularly those produced by the Aspergillus falvus, a mold found in spoiled corn, peanuts, and grain. Aflaxtoxins cause a mutation of the TP53 suppressor gene and activation of WNT signal transduction pathway. Heavy smoking, heavy drinking, presence of liver flukes. Pancreas-A Kras mutation is the most common genetic alteration, tumor suppressor gene alterations are also found including, TP53, p16, and DCC.

What part of the kidney controls renal blood flow, glomerular filtration, and renin secretion?

The JG apparatus, which is composed of the JG cells and macula densa cells control renal blood flow, glomerular filtration and renin secretion. The JG cells are located around the afferent arteriole where it enters the glomerulus. Between the afferent and the efferent arterioles is a portion of the distal convoluted tubule with specialized Na and Cl sensing cells known as the macula densa.

13. Differentiate parasympathetic, autonomic, sympathetic nervous system.

The PNS functions to conserve and restore energy. In general, sympathetic stimulation promotes responses that are concerned with the protection of the individual that would include increased blood sugar levels and temperature and BP. The responses noted in the remaining options are not a result of sympathetic stimulation. Functions below the level of consciousness to control visceral functions (regulation of cardiac muscle, smooth muscle and glands of the body, considered involuntary). Partially located in CNS, but mostly looked at as the "efferent" (carrying nerve impulses away from CNS) division of the PNS. Motor component is a 2 neuron system consisting of preganglionic neurons (myelinated) and postganglionic neurons (not myelinated). Separated structurally and functionally into 2 divisions: PNS and SNS. PNS: functions to conserve and restore energy (rest and repose). Is the craniosacral division as its nerve cell bodies are found in the cranial nerve nuclei and sacral region. Preganglionic fibers travel to ganglia close to the organs that they innervate before forming synapses with the short postganglionic neurons. Increased parasympathetic activity leads to a reduced HR, enhanced visceral functions including digestion (stimulation of vagus nerve), increases peristalsis including secretion and relaxation of sphincters, constriction of pupils. Lacks the generalized and widespread response of the sympathetic system. SNS: functions to mobilize energy stores in times of need (fight or flight). Called the thoracilumbar division from synapses after leaving the cord in the sympathetic (paravertebral) ganglia. Its here where the impulses can choose to travel by: 1) directly across the same ganglion level and form a synapse with the cell bodies of the postganglionic neuron. 2) Up or down the sympathetic chain before forming synapses with a higher or lower postganglionic neuron. 3) Through the chain ganglion w/o synapsing. Some preganglionic axons form pathways called splanchnic nerves leads to collateral ganglia surrounding the abdominal aorta. Collateral ganglia are named according to the branches of the aorta closes to them. Postganglionic neurons leave the collateral ganglia and innervate the viscera below the diaphragm. Because the preganglionic sympathetic neurons are myelinated, they travel via the splanchnic nerve and innervated the adrenal medulla quickly to release epi and norepi. Neurotransmitter that is released is acetylcholine, characterized by cholinergic transmission, most postganglionic sympathetic fibers release norepi and are considered to be functioning by adrenergic transmission. Adrenergic receptors include alpha and beta-adrenergic receptors. Alpha 1 activity: excitation and stimulation. Alpha 2 activity: relaxation or inhibition. Beta 1 activity: facilitate increased HR & contractility and cause the release of renin from the kidney. Beta 2 activity: facilitate all the remaining effects attributed to beta-adrenergic receptors. Norepi stimulates all alpha-adrenergic and beta1 adrenergic receptors and certain beta 2 adrenergic receptors. The primary response from norepi is stimulation of the alpha 1 adrenergic receptors that cause vasoconstriction.

How is abdominal pain produced?

The causal mechanisms of abd pain are mechanical, chemical mediators of inflammation, or ischemic. Generally the abd organs are not sensitive to mechanical stimuli, such as cutting, tearing, or crushing. These organs are, however, sensitive to stretching and distention, which activate nerve endings in hollow as well as solid structures. The onset of pain is associated with rapid distention causes little pain. Biochemical mediators of inflammatory response, such as histamine, bradykinin and serotonin stimulate pain nerve endings and produce abd pain. The edema and vascular congestion that accompany chemical, bacterial, or viral inflammation also cause painful stretching. Obstruction of blood flow from the distention of bowel obstruction or mesenteric vessel thrombosis produces the pain of ischemia and increased concentrations of tissue metabolites stimulate pain receptors. Abd pain may be generalized to the abd or localized to a particular abd quadrant. The pain is often described as sharp, dull or colicky. The casual mechanisms of abd pain are mechanical, chemical mediators or inflammation of ischemic. Generally, the abd organs are not sensitive to mechanical stimuli, such as cutting, tearing or crushing. These organs are sensitive to stretching and distention, which activate nerve endings in hollow as well as solid structure. The onset of pain is associated with rapid distention, gradual distention causes little pain. Traction on the peritoneum caused by adhesions, distention of the common bile duct or forceful peristalsis resulting from intestinal obstruction causes pain because of increased tension. Capsules that surround the solid organs, such as the liver and gallbladder, contain pain fibers that are stimulated by stretching if these organs swell. Biochemical mediators of the inflammatory response, such as histamine, bradykinin and serotonin, stimulate pain nerve endings that produce abd pain. The edema and vascular congestion that accompany chemical, bacterial or viral inflammation also cause painful stretching. Obstruction of blood flow from the distention of bowel obstruction or mesenteric vessel thrombosis produces the pain of ischemia, and increased concentration of tissue metabolites that stimulate pain receptors.

Where is the chemoreceptor trigger zone (CTZ) for vomiting located?

The chemoreceptor trigger zone (CTZ) is located in the area postrema in the 4th ventricle of the medulla oblongata. Stimulation of receptors, such as dopamine, serotonin, opioid, acetylcholine and substance P, in the CTZ leads to vomiting.

Define the relationship between sweating and a spinal cord injury.

The condition also results in disturbed thermal control because the hypothalamus is unable to regulate a damaged sympathetic nervous system. This damage causes faulty control of sweating and radiation through capillary dilation.

Which abnormal lab value is found in glomerular disorders?

The diagnosis of glomerular disease is confirmed by the progressive development of clinical manifestations and laboratory findings of abnormal UA and proteinuria, RBC, WBC, and casts. Microscopic evaluation from renal biopsy form renal biopsy provides a specific determination of renal injury and type of pathologic condition.

What organ regulates eating behavior, energy metabolism, and body fat mass?

The hypothalamus is where appetite and metabolism are regulated. Hormones that circulate in the blood at concentrations proportional to body fat mass serve as peripheral signals to the arcuate nucleus (ARC) in the hypothalamus. Regulation of appetite and satiety occurs through neuroendocrine regulation of eating behavior, energy metabolism, and body fat mass. This is a complex system is controlled by a circuit of signaling molecules from the periphery acting on central controls including the brainstem, hypothalamus, and ANS. The arcuate nucleus in the hypothalamus has 2 sets of neurons with opposing effects that interact to regulate and balance food intake and energy metabolism. Many different hormones control appetite and body weight. The sources of these include insulin from the beta cells of the pancreas, ghrelin from the stomach, peptide YY from the intestines, and leptin, adiponectin, and resistin from the the adipocytes of adipose tissue. These hormones circulate in the blood in levels proportional to body fat mass and serve as peripheral signals to the arcuate nucleas of the hypothalamus where appetite and metabolism are regulate.

Innervation of the bladder and internal urethral sphincter is supplied by which nerves?

The innervation of the bladder and internal urethral sphincter is supplied by parasympathetic fibers of the autonomic nervous system. They primarily pass with the arteries to and from the sacral levels of the spinal cord. Sensory fibers may extend as high as the T6 position of the spinal cord. Motor fibers from the pudendal nerve supply the external urethral sphincter. The reflex arc required for micturition is stimulated by mechanoreceptors that respond to stretching of tissue.

List the sequence of events that lead to hyperreflexia induced bradycardia.

The intact ANS reflexively responds with an arteriolar spasm that increases BP. Baroreceptors in the cerebral vessels, the carotid sinus, and the aorta sense the HTN and stimulate the PNS. The HR decreases, but the visceral and peripheral vessels don't dilate because efferent impulses cannot pass through the cord. Stimulation of the carotid sinus to the vagus nerve to the SA node. The intact ANS reflexively responds with an arteriolar spasm that increases BP. Baroreceptors in the cerebral vessels, the carotid sinus, and the aorta sense the HTN and stimulate the PNS. The HR decreases, but the visceral and peripheral vessels don't dilate because efferent impulses cannot pass through the cord. Sudden dangerous increase in BP. Occurs anytime after spinal shock resolves. Massive, uncompensated cardiovascular response to stimulation of SNS. Individuals most at risk have lesions T6 or higher. Characterized by paroxysmal HTN up to 300, HA, blurred vision, sweating above level of the lesion with flushing of skin, nasal congestion, piloerection and bradycardia often in combination with distended bladder or rectum. The ANS reflexively responds with an arterial spasm that increases BP. Baroreceptors in the cerebral vessels, carotid sinus, and the aorta sense the HTN and stimulate the PNS.

How much urine accumulates in the bladder before the mechanoreceptors sense bladder fullness?

The mechanoreceptors sense bladder fullness and send impulses to the sacral level of the cord with bladder filling. When the bladder accumulates 250-300mL of urine, the bladder contracts and the internal urethral sphincter relaxes through activation of the spinal reflex arc. At this time a person feels the urge to void. In older adults, the reflex can be initiated or facilitated by impulses coming from the brain resulting in voluntary control of micturition.

The most common type of renal stone is?

The most common stone types include calcium oxalate or phosphate, struvite (magnesium, ammonium, and phosphate), and uric acid.

Differentiate craniosacral and parasympathetic dysreflexia.

The nerve cell bodies of this division are located in the cranial nerve nuclei and in the sacral region of the spinal cord and therefore constitute the craniosacral division. Parasympathetic dysreflexia, also known as autonomic hyperreflexia or autonomic dysreflexia. The craniosacral division is from the cranial nerve nuclei and in the sacral region of the spinal cord. Parasympathetic is from the sacral region of the spinal cord run either separately or together with some spinal nerves.

Define the function of the temporal lobe, parietal lobe, cerebral cortex and the hypothalamus.

The primary motor area of the frontal lobe is located along the precentral gyrus forming the primary voluntary motor area. Electrical stimulation of specific areas of this cortex causes specific muscles of the body to move. The parietal lobe lies within the borders of the central, parieto-occipital, and lateral sulci. The lobe contains the major area for somatic sensory input. The temporal lobe lies inferior to the lateral sulcus and is composed of the superior, middle and inferior temporal gyri. The primary auditory cortex and its related association area lie deep within the lateral sulcus on the superior temporal gyrus. The occipital lobe lies caudal to the parietooccipital sulci and superior to the cerebellum. The primary visual cortex is located in this region and receives input from the retinas. Much of the remainder of this lobe is involved in visual association. Temporal: This area is responsible for reception and interpretation of speech, and dysfunction may result in aphasia or dysphasia. Temporal lobe also involved as a major area for long-term memory and secondary functions such as balance, taste, and smell. The primary auditory cortex and its related area lie deep within the lateral sulcus on the superior temporal gyrus. Wernicke is located on superior temporal gyrus. The 8 principle symptoms of temporal lobe damage include: disturbances of the auditory sensation and perception, disturbances of selective attention of auditory and visual input, disorders of visual perception, impaired organization and categorizing of verbal material, disturbances of language comprehension, impaired long term memory, altered personality and affective behavior, and altered sexual behavior. Parietal: (body sensory) this lobe contains the major area for somatic sensory input, located primarily along the post-central gyrus, which is adjacent the primary motor area. Much of this region is involved in sensory association (storage, analysis, and interpretation of stimuli). Injury may affect the ability to locate parts on your body, and the ability to recognize parts of your body. Cerebral cortex: The cerebral cortex contains the cell bodies and dendrites of neurons, often are referred to as gray matter. Gray matter is organized into columns perpendicular to the surface that receive, integrate, store, and transmit information. White matter lies beneath the cerebral cortex. Cerebral cortex is part of telencephalon in the forebrain. Hypothalamus: Hypothalamus makes up diencephalon, with epithalamus, thalamus, and subthalamus, and forms the roof of the 3rd ventricle. Functions include: visceral and somatic responses, effectual responses, hormone synthesis, sympathetic and parasympathetic activity, temperature regulation, feeding responses, physical expression of emotion, sexual behavior, pleasure punishment centers, level of arousal or wakefulness.

How is the herpes virus genomes are transported to the dermal surface?

The virus is spread by skin-to-skin mucous membrane contact during viral shedding. After penetrating the skin, HSV, is established in the sensory nerve ganglion innervating the primary site where it can cause recurrence.

Which of the following statements provides the most accurate information regarding the transmission of HSV?

There are 8 types HSV (1, 2, CMV, varicella-zoster 3, EBV, herpes virus 6, 7 and 8. However, types 1 and 2 are most common and type 2 is most among the 2, 80% if initial and 98% of recurrent infections). Regarding transmission: the virus is spread from skin-to-skin mucous membrane contact during viral shedding. Its transmitted through intimate contact with a person who is shedding the virus in a secretion OR from a peripheral lesion OR mucosal surface. Persons w/o symptoms probably transmit most infections. A woman has 80-90% risk of developing HSV after exposure to an infected man. The likelihood of nonsexual aerosolized secretions is quite rare and unlikely. Neonatal infections can occur during utero, intrapartum or post partum periods. Perinatal transmission can cause extensive morbidity and mortality.

What are the results of a primary adenoma.

Those are usually benign slow growing tumors that arise from cells of the anterior pituitary, most commonly those that secrete GH and prolactin. HA and fatigue, visual disturbances because of pressure of the tumor on the optic chasm, it causes thyroid and adrenal hypo function because the tumor has a paradoxical effect on adjacent cells which results in hypo secretion of other anterior pituitary hormones.

Lyme disease

Tick-borne bacterial infection

Vestibulocochlear nerve

Transmits impulses for the sense of hearing.

Trichomonas vaginalis

Trichomoniasis

Inflammation develops in crypts of Lieberkühn in the large intestine.

Ulcerative colitis

The most common clinical manifestation of portal hypertension is _____ bleeding.

Upper gastrointestinal vericeal bleeding. Vomiting of blood from bleeding esophageal varicies is the most common clinical manifestation of portal HTN. Splenomegaly is the most common sign of portal HTN in children.

What is the usual cause of Pyelonephritis?

Urinary obstruction and reflux of urine from the bladder are common underlying risk factors. Both kidneys can be involved. Mostly occur in women. Responsible microorganisms responsible are E. Coli or pseudomonas. Other common causes that can contribute to an infection of the upper urinary tract can consist of kidney stones, vesicouretral reflux, pregnancy, neurogenic bladder, and female sexual trauma. An ascending infection by E. Coli. Urinary obstruction and reflux of urine from the bladder are common underlying risk factors. Both kidneys can be involved. Mostly occur in women. Responsible microorganisms responsible are E. Coli or pseudomonas. Other common causes that can contribute to an infection of the upper urinary tract can consist of kidney stones, vesicouretral reflux, pregnancy, neurogenic bladder, and female sexual trauma.


Conjuntos de estudio relacionados

Psychology Test 3 (Chapters 6, 7)

View Set

AP Macro prgress check unit 2: MCQ

View Set

Foundations of Nursing - Unit 6 Exam

View Set

Chapter 27/35- Disorders of the Bladder and Lower Urinary Tract

View Set

Chem: Unit 3: Atomic Concepts - castle learning

View Set